Buy BOOKS at Discounted Price

Linear Programming

Class 12th Mathematics RD Sharma Volume 2 Solution
Exercise 30.1
  1. A small manufacturing firm produces two types of gadgets A and B, which are…
  2. A company is making two products A and B. The cost of producing one unit of…
  3. A firm manufactures 3 products A, B and C. The profits are Rs. 3, Rs. 2 and Rs.…
  4. A firm manufactures two types of products A and B and sells them at a profit of…
  5. A rubber company is engaged in producing three types of tyres A, B and C. Each…
  6. A company sells two different products A and B. The two products are produced…
  7. To maintain his health a person must fulfil certain minimum daily requirements…
  8. A manufacturer can produce two products, A and B, during a given time period.…
  9. Vitamins A and B are found in two different foods F1 and F2. One unit of food…
  10. An automobile manufacturer makes automobiles and trucks in a factory that is…
  11. Two tailors A and B earn Rs 150 and Rs 200 per day respectively. A can stitch…
  12. An airline agrees to charter planes for a group. The group needs at least 160…
  13. Amit’s mathematics teacher has given him three long lists of problems with the…
  14. A farmer has a 100 - acre farm. He can sell the tomatoes, lettuce, or radishes…
  15. A firm has to transport at least 1200 packages daily using large vans which…
  16. A firm manufactures two products, each of which must be processed through two…
Exercise 30.2
  1. Maximize Z = 5x + 3y Subject to : 3x + 5y ≤ 15 5x + 2y ≤ 10 x, y ≥ 0 Solve each…
  2. Maximize Z = 9x + 3y Subject to : 2x + 3y ≤ 13 3x + y ≤ 5 x, y ≥ 0 Solve each…
  3. Minimize Z = 18x + 10y Subject to : 4x + y ≥ 20 2x + 3y ≥ 30 x, y ≥ 0 Solve…
  4. Maximize Z = 50x + 30y Subject to : 2x + y ≤ 18 3x + 2y ≤ 34 x, y ≥ 0 Solve…
  5. Maximize Z = 4x + 3y Subject to : 8x + 6y ≤ 48 3x + 4y ≤ 24 x ≤ 5 , y ≤ 6 x, y…
  6. Maximize Z = 15x + 10y Subject to : 2x + 3y ≤ 70 3x + 2y ≤ 80 x, y ≥ 0 Solve…
  7. Maximize Z = 10x + 6y Subject to : 2x + 5y ≤ 34 3x + y ≤ 12 x, y ≥ 0 Solve each…
  8. Maximize Z = 3x + 4y Subject to : 2x + 2y ≤ 80 2x + 4y ≤ 120 Solve each of the…
  9. Maximize Z = 7x + 10y Subject to : x + y ≤ 30000 y ≤ 12000 x ≥ 6000 x, y ≥ 0 x…
  10. Maximize Z = 2x + 4y Subject to : x + y ≥ 8 x + 4y ≥ 12 x ≥ 3 y ≥ 2 Solve each…
  11. Minimize Z = 5x + 3y Subject to : 2x + y ≥ 10 x + 3y ≥ 15 x ≤ 10 y ≤ 8 x, y ≥…
  12. Minimize Z = 30x + 20y Subject to : x + y ≤ 8 x + 4y ≥ 12 5x + 8y = 20 x, y ≥…
  13. Maximize Z = 4x + 3y Subject to : 8x + 6y ≤ 48 3x + 4y ≤ 24 x ≤ 5 , y ≤ 6 x, y…
  14. Minimize Z = x - 5y + 20 Subject to : x - y ≥ 0 - x + 2y ≥ 2 x ≥ 3 y ≤ 4 x, y…
  15. Maximize Z = 3x + 5y Subject to : x + 2y ≤ 20 x + y ≤ 15 y ≤ 5 X, y ≥ 0 Solve…
  16. Minimize Z = 3x1 + 5x2 Subject to : x1 + 3x2 ≥ 3 x1 + x2 ≥ 2 x1, x2 ≥ 0 Solve…
  17. Maximize Z = 2x + 3y Subject to : x + y ≥ 1 10x + y ≥ 5 x + 10y ≥ 1 x, y ≥ 0…
  18. Maximize Z = - x1 + 2x2 Subject to : - x1 + 3x2 ≤ 10 x1 + x2 ≤ 6 x1 - x2 ≤ 2…
  19. Maximize Z = x + y Subject to : - 2x + y ≤ 1 x ≤ 2 x + y ≤ 3 x, y ≥ 0 Solve…
  20. Maximize Z = - x1 + 2x2 Subject to : - x1 + 3x2 ≤ 10 x1 + x2 ≤ 6 x1 - x2 ≤ 2…
  21. Maximize Z = 3x + 4y Subject to : x - y ≤ 1 x + y ≥ 3 x, y ≥ 0 Solve each of…
  22. Show the solution zone of the following inequalities on a graph paper: 5x + y…
  23. Find the maximum and minimum value of 2x + y subject to the constraints : x +…
  24. Find the minimum value of 3x + 5y subject to the constraints - 2x + y ≤ 4 x +…
  25. Solved the following linear programming problem graphically : Maximize Z = 60x…
  26. Find graphically, the maximum value of Z = 2x + 5y, subject to constraints…
  27. Solve the following LPP graphically : Maximize Z = 20x + 10y Subject to the…
  28. Solve the following linear programming problem graphically : Minimize z = 6x +…
Exercise 30.3
  1. A diet of two foods F1 and F2 contains nutrients thiamine, phosphorous and…
  2. A diet for a sick person must contain at least 4000 units of vitamins, 50 units…
  3. To maintain one’s health, a person must fulfill certain minimum daily…
  4. A hospital dietician wishes to find the cheapest combination of two foods, A…
  5. A dietician mixes together two kinds of food in such a way that the mixture…
  6. A diet is to contain at least 80 units of vitamin A and 100 units of minerals.…
  7. Kellogg is a new cereal formed of a mixture of bran and rice that contains at…
  8. A wholesale dealer deals in two kinds, A and B (say) of mixture of nuts. Each…
  9. One kind of cake requires 300 gm of flour and 15 gm of fat, another kind of…
  10. Reshma wishes to mix two types of food P and Q in such a way that the vitamin…
  11. One kind of cake requires 200 g of flour and 25 g of fat, and another kind of…
  12. A dietician has to develop a special diet using two foods P and Q. Each packet…
  13. A farmer mixes two brands P and Q of cattle feed. Brand P, costing ₹ 250 per…
  14. A dietician wishes to mix together two kinds of food X and Y in such a way…
  15. A fruit grower can use two types of fertilizer in his garden, brand P and Q.…
Exercise 30.4
  1. If a young man drives his scooter at a speed of 25 km/hr, he has to spend Rs2…
  2. A manufacturer has three machines installed in his factory. Machines I and II…
  3. Two tailors, A and B earn ₹ 15 and ₹ 20 per day respectively. A can stitch 6…
  4. A factory manufactures two types of screws, A and B, each type requiring the…
  5. A company produces two types of leather belts, say type A and B. Belt A is a…
  6. A small manufacturer has employed 5 skilled men and 10 semi - skilled men and…
  7. A manufacturer makes two types A and B of tea - cups. Three machines are needed…
  8. A factory owner purchases two types of machines, A and B, for his factory. The…
  9. A company produces two types of goods, A and B, that require gold and silver.…
  10. A manufacturer of Furniture makes two products : chairs and tables. Processing…
  11. A furniture manufacturing company plans to make two products : chairs and…
  12. A firm manufactures two products A and B. Each product is processed on two…
  13. A firm manufacturing two type of electric items, A and B, can make a profit of…
  14. A factory uses three different resources for the manufacture of two different…
  15. A publisher sells a hard cover edition of a text book for ₹ 72.00 and a…
  16. A firm manufactures headache pills in two sizes A and b. Size A contains 2…
  17. A chemical company produces two compounds, A and B. The following table gives…
  18. A company manufactures two type of novelty Souvenirs made of plywood.…
  19. A manufacturer makes two products A and B. Product A sells at 200 each and…
  20. A manufacturer produces two type of steel trunks. He has two machines A and B.…
  21. A manufacturer of patent medicines is preparing a production plan on…
  22. An aeroplane can carry a maximum of 200 passengers. A profit of ₹ 400 is made…
  23. A gardener has a supply of fertilizer of type I which consists of 10% nitrogen…
  24. Anil wants to invest at most ₹ 12000 in Saving Certificates and National…
  25. A man owns a field of area 1000 sq.m. He wants to plant fruit trees in it. He…
Exercise 30.5
  1. Two godowns, A and B, have grain storage capacity of 100 quintals and 50…
  2. A medical company has factories at two places, A and B. From these places,…
Mcq
  1. The solution set of the inequation 2x + y 5 is
  2. Objective function of a LPP is
  3. Which of the following sets are convex?
  4. Let X1 and X2 are optimal solutions of a LPP, then
  5. The maximum value of Z = 4x + 2y subjected to the constraints 2x + 3y ≤ 18, x + y ≥ 10;…
  6. The optimal value of the objective function is attained at the points…
  7. The maximum value of Z = 4x + 3y subjected to the constraints 3x + 2y ≥ 160, 5x + 2y ≥…
  8. Consider a LPP given byMinimum Z = 6x + 10ySubjected to x ≥ 6; y ≥ 2; 2x + y ≥ 10; x, y…
  9. The objective function Z = 4x + 3y can be maximized subjected to the constraints 3x +…
  10. If the constraints in a linear programming problem are changed
  11. Which of the following statements is correct?
  12. Which of the following is not a convex set?
  13. By graphical method, the solution of linear programming problemMaximize Z = 3x1 +…
  14. The region represented by the inequation system x, y ≥ 0, y ≤ 6, x + y ≤ 3 is…
  15. The point at which the maximum value of x + y, subject to the constraints x + 2y ≤ 70,…
  16. The value of objective function is maximum under linear constraints…
  17. The corner points of the feasible region determined by the following system of linear…

Exercise 30.1
Question 1.

A small manufacturing firm produces two types of gadgets A and B, which are first processed in the foundry, then sent to the machine shop for finishing. The number of man - hours of labour required in each shop for the production of each unit of A and B, and the number of man - hours the firm has available per week are as follows:


The profit on the sale of A is Rs 30 per unit as compared with ₹ 20 per unit of B. The problem is to determine the weekly production of gadgets A and B, so that the total profit is maximized. Formulate this problem as a LPP.


Answer:

The given data can be shown in a table as follows:



Now, let the required weekly production of gadgets A and B be x and y respectively


As it is given that profit on each gadget A is Rs. 30 and that on B is Rs. 20, so profit on x and y number of gadgets A and B respectively are 30x and 20y.


If z = total profit then we have,


z = 30x + 20y


Further, it is also given that the production of A and B requires 10 hours per week and 6 hours per week in the foundry. Also, the maximum capacity of the foundry is given as 1000 hours.


Now, x units of A and y units of B will require 10x + 6y hours


So, we have


10x + 6y ≤ 1000


This is our first constraint.


Given, production of one unit gadget A requires 5x hours per week and y units of gadget B requires 4y hours per week, but the maximum capacity of the machine shop is 600 hours per week.


So, 5x + 4y ≤ 600


This is our second constraint.


Hence, the mathematical formulation of LPP is:


Find x and y which will maximize z = 30x + 20y


Subject to constraints,


10x + 6y ≤ 1000


5x + 4y ≤ 600


and also, as production cannot be less than zero, so x, y ≥ 0



Question 2.

A company is making two products A and B. The cost of producing one unit of products A and B are Rs 60 and Rs 80 respectively. As per the agreement, the company has to supply at least 200 units of product B to its regular customers. One unit of product A requires one machine hour whereas product B has machine hours available abundantly within the company. Total machine hours available for product A are 400 hours. One unit of each product A and B requires one labour hour each and total of 500 labour hours are available. The company wants to minimize the cost of production by satisfying the given requirements. Formulate the problem as a LPP.


Answer:

The given data can be shown in a table as follows:



Let production of product A be x units and of be B y units.


Given,


Profit on 1 unit of product A = Rs.60


Profit on 1 unit of product B = Rs.80


So, profit on x units of A and y units of B is 60x and 80y respectively.


Let z = total profit,


So, we have


z = 60x + 80y


Given, a minimum supply of product B is 200


So, y ≥ 200 (First constraint)


Given that, production of one unit of product A requires 1 hour of machine hours, so x units of product A require x hours but total machine time available for product A is 400 hours


So, x ≤ 400 (Second constraint)


Given, each unit of product A and B requires one hour of labour hour, so x units of product A require x hours and y units of product B require y hours of labour hours, but total labour hours available is 500 so


x + y ≤ 500 (Third constraint)


Hence, mathematical formulation of LPP is,


Find x and y which


Minimize z = 60x + 80y


Subject to constraints,


y ≥ 200


x ≤ 400


x + y ≤ 500


and also, as production cannot be less than zero, so x, y ≥ 0



Question 3.

A firm manufactures 3 products A, B and C. The profits are Rs. 3, Rs. 2 and Rs. 4 respectively. The firm has 2 machines and below is the required processing time in minutes for each machine on each product.



Machines M1 and M2 have 2000 machine minutes respectively. The firm must manufacture 100 A’s, 200 B’s and 50 C’s but not more than 150 A’s. Set up a LPP to maximize the profit.


Answer:

The given data can be formulated in a table as below.



Let, required production of product A, B and C be x, y and z units respectively.


Given, profit on one unit of product A, B and C are Rs 3, and Rs 2, Rs 4.


So, profit on x, y, z units of A, B, C Rs 3x, Rs 2y, Rs 4z.


Let U be the total profit, so


U = 3x + 2y + 4z


Given, one unit of product A, B and C requires 4, 3 and 5 minutes on machine M1. So, x units of A, y units of B and z units of C need 4x, 3y and 5z minutes. Maximum capacity on machine M1 is 2000 minutes, so,


4x + 3y + 5z ≤ 200 0 (First constraint)


Given, one unit of product A, B and C requires 2, 2 and 4 minutes on machine M2. So, x units of A, y units of B and z units of C require 2x, 2y and 4z minutes. Maximum capacity on machine M2 is 2500 minutes, so,


2x + 2y + 4z ≤ 250 0 (Second constraint)


Also, given that firm must manufacture more than 100 A’s, 200 B’s, 50 C’s also not more than 150 A’s, so,


100 ≤ x ≤ 150,


y ≥ 200 (Other constraints)


z ≥ 50


Hence, mathematical formulation of LPP is:


Find x, y and z which maximize U = 3x + 2y + 4z


Subject of constraints,


4x + 3y + 5z ≤ 2000


2x + 2y + 4z ≤ 2500


100 ≤ x ≤ 150,


y ≥ 200


z ≥ 50


and also, as production cannot be less than zero, so x, y ≥ 0



Question 4.

A firm manufactures two types of products A and B and sells them at a profit of Rs 2 on type A and Rs 3 on type B. Each product is processed on two machines M1 and M2. Type A requires one minute of processing time on M1 and two minutes of M2; type B requires one minute on M1 and one minute on M2. The machine M1 is available for not more than 6 hours 40 minutes while machine M2 is available for 10 hours during any working day. Formulate the problem as a LPP.


Answer:

Given equation can be written in tabular form as



Let required production of product A be x units and product B be y units.


Given, profit on one unit of product A and B are Rs 2 and Rs 3 respectively, so profits on x units of product A and y units of product B will be Rs 2x and Rs 3y respectively.


Let total profit be Z, so Z = 2x + 3y


Given, production of one unit of product A and B require 1 and 1 minute on machine M1 respectively, so production of x units of product A and y units of product B require x minutes and y minutes on machine M1 but total time available on machine M1 is 600 minutes, so


x + y ≤ 400 (First constraint)


Given, production of one unit of product A and B require 2 minutes and 1 minutes on machine M2 respectively. So, production of x units of product A and y units of product B require 2x minutes and y minutes respectively on machine M2, but machine M2 is available for 600 minutes, so


2x + y ≤ 600 (Second constraint)


Hence, the mathematical formulation of LPP is:


Find x and y which


maximize Z = 2x + 3y


Subject to constraints,


x + y ≤ 400


2x + y ≤ 600


and, x, y ≥ 0 [Since production of the product cannot be less than zero]



Question 5.

A rubber company is engaged in producing three types of tyres A, B and C. Each type requires processing in two plants, Plant I and Plant II. The capacities of the two plants, in the number of tyres per day, are as follows:



The monthly demand for tyre A, B and C is 2500, 3000 and 7000 respectively. If plant I costs Rs 2500 per day, and plant II costs Rs 3500 per day to operate, how many days should each be run per month to minimize cost while meeting the demand? Formulate the problem as LPP.


Answer:


Let plant 1 requires x days, and plant II requires y days per month to minimize cost.


Given, plant I and II costs Rs 2500 per day and Rs 3500 per day respectively, so cost to run plant I and II are Rs 2500x and Rs 3500y per month.


Let Z be the total cost per month,


So, Z = 2500x + 3500y


Given, production of tyre A from plant I and II is 50 and 60 respectively, so production of tyre A from plant I and II will be 50x and 60y respectively per month but the maximum demand of tyre A is 2500 per month so,


100x + 60y ≥ 2500 [First constraint]


Given, production of tyre B from plant I and II is 100 and 60 respectively, so production of tyre B from plant I and II will be 100x and 60y per month respectively but the maximum demand of tyre B is 3000 per month, so


100x + 200y ≥ 3000 [Second constraint]


Given, production of tyre C from plant I and II is 100 and 200 respectively. So, production of tyre C from plant I and II will be 100x and 200y per month respectively but the maximum demand of tyre C is 7000 per day, so


100x + 200y ≥ 7000 [Third constraint]


Hence, mathematical formulation of LPP is,


Find x and y which


Minimize Z = 2500x + 3500y


Subject to constraint,


50x + 60y ≥ 2500


100x + 60y ≥ 3000


100x + 200y ≥ 7000


And, x, y ≥ 0 [Since number of days cannot be less than zero]



Question 6.

A company sells two different products A and B. The two products are produced in a common production process and are sold in two different markets. The production process has a total capacity of 45000 man - hours. It takes 5 hours to produce a unit of A and B hours to produce a unit of B. The market has been surveyed and company officials feel that the maximum number of units of A that can be sold is 7000 and that of B is 10, 000. If the profit is Rs 60 per unit for the product A and Rs 40 per unit for the product B, how many units of each product should be sold to maximize profit? Formulate the problem as LPP.


Answer:


Let required production of product A be x units and production of product B be y units.


Given, profits on one unit of product A and B are Rs 60 and Rs 40 respectively, so profits on x units of product A and y units of product B are Rs 60x and Rs 40y.


Let Z be the total profit, so
Z = 60x + 40y


Given, production of one unit of product A and B require 5 hours and 3 hours respectively man hours, so x unit of product A and y units of product B require 5x hours and 3y hours of man hours respectively but total man hours available are 45000 hours, so


5x + 3y ≤ 45000 (First constraint)


Given, demand for product A is maximum 7000, so


x ≤ 7000 (Second constraint)


Hence, mathematical formulation of LPP:


Find x and y which


maximize Z = 60x + 40y


Subject to constraints,


5x + 3y ≤ 45000


x ≤ 7000


y ≤ 10000


x, y ≤ 0 [Since production cannot be less than zero]



Question 7.

To maintain his health a person must fulfil certain minimum daily requirements for several kinds of nutrients. Assuming that there are only three kinds of nutrients – calcium, protein and calories and the person’s diet consists of only two food items, I and II, whose price and nutrient contents are shown in the table below:



What combination of two food items will satisfy the daily requirement and entail the least cost? Formulate this as a LPP.


Answer:

Let x and y be the packets of 25 gm of Food I and Food II purchased. Let Z be the price paid. Obviously, price has to be minimized.


Take a mass balance on the nutrients from Food I and II,


Calcium 10x + 4y ≥ 20


5x + 2y ≥ 10 (i)


Protein 5x + 5 y ≥ 20


x + y ≥ 4 (ii)


Calories 2x + 6y > 13 (iii)


These become the constraints for the cost function, Z to be minimized i.e., 0.6x + y = 7, given cost of Food I is Rs 0.6/ - and Rs 1/ - per lb


From (i), (ii) & (iii) we get points on the X & Y - axis as [0, 5] & [2, 0] ; [0, 4] & [ 4, 0] ; [0, 13/6] & [6.5, 0] Plotting these



The smallest value of Z is 2.9 at the point (2.75, 1.25). We cannot say that the minimum value of z is 2.9 as the feasible region is unbounded.


Therefore, we have to draw the graph of the inequality 0.6x + y <2.9


Plotting this to see if the resulting line (in green) has any point common with the feasible region. Since there are no common points this is the minimum value of the function Z and the mix is


Food I = 2.75 lb; Food II = 1.25 lb; Price = Rs 2.9


When Z has an optimal value (maximum or minimum), where the variables x and y are subject to constraints described by linear inequalities., this optimal value must occur at a corner point (vertex) of the feasible region.


Here the feasible region is the unbounded region A - B - C - D


Computing the value of 7 at the corner points of the feasible region ABHG




Question 8.

A manufacturer can produce two products, A and B, during a given time period. Each of these products requires four different manufacturing operations: grinding, turning, assembling and testing. The manufacturing requirements in hours per unit of products A and B are given below.



The available capacities of these operations in hours for the given time period are: grinding 30; turning 60, assembling 200; testing 200. The contribution to profit is Rs 20 for each unit of A and Rs 30 for each unit of B. The firm can sell all that it produces at the prevailing market price. Determine the optimum amount of A and B to produce during the given time period. Formulate this as a LPP.


Answer:


Let required production of product A and B be x and y respectively


Given, profits on one unit of product A and B are Rs 2 and Rs 3 respectively, so profits on x units of product A and y units of product B are given by 2x and 3y respectively. Let Z be total profit, so


Z = 2x + 3y


Given, production of 1 unit of product A and B require 1 hour and 2 hours of grinding respectively, so, production of x units of product A and y units of product B require x hours and 2y hours of grinding respectively but the maximum time available for grinding is 3 hours, so


x + 2y ≤ 30 (First constraint)


Given, production of 1 unit of product A and B require 3 hours and 1 hours of turning respectively, so x units of product A and y units of product B require 3x hours and y hours of turning respectively but total time available for turning is 60 hours, so


3x + y ≤ 60 (Second constraint)


Given, production of 1 unit of product A and B require 6 hour and 3 hours of assembling respectively, so production of x units of product A and y units of product B require 6x hours and 3y hours of assembling respectively but total time available for assembling is 200 hours, so


6x + 3y ≤ 200 (Third constraint)


Given, production of 1 unit of product A and B require 5 hours and 4 hours of testing respectively, so production of x units of product A and y units of product B require 5x hours and 4y hours of testing respectively but total time available for testing is 200 hours, so


5x + 4y ≤ 200 (Fourth constraint)


Hence, mathematical formulation of LPP is,


Find x and y which


maximize Z = 2x + 3y


Subject to constraints,


x + 2y ≤ 30


3x + y ≤ 60


6x + 3y ≤ 200


5x + 4y ≤ 200


and, x, y ≥ 0 [Since production cannot be negative]



Question 9.

Vitamins A and B are found in two different foods F1 and F2. One unit of food F1 contains 2 units of vitamin A and 3 units of vitamin B. One unit of food F2 contains 4 units of vitamin A and 2 units of vitamin B. One unit of food F1 and F2 cost ₹ 50 and 25 respectively. The minimum daily requirements for a person of vitamin A and B is 40 and 50 units respectively. Assuming that anything in excess of daily minimum requirement of vitamin A and B is not harmful, find out the optimum mixture of food F1 and F2 at the minimum cost which meets the daily minimum requirement of vitamin A and B. Formulate this as a LPP.


Answer:

Given information can be tabulated as below:



Let required quantity of food F1 be x units and quantity of food F2 be y units.


Given, costs of one unit of food F1 and F2 are Rs 5 and Rs 2.5 respectively, so costs of x units of food F1 and y units of food F2 are Rs 5x and Rs 2.5y respectively.


Let Z be the total cost, so


Z = 5x + 2.5y


Given, one unit of food F1 and food F2 contain 2 and 4 units of vitamin A respectively, so x unit of Food F1 and y units of food F2 contain 2x and 4y units of vitamin A respectively, but minimum requirement of vitamin A is 40 unit, so


2x + 4y ≥ 40 (First constraint)


Given, one unit of food F1 and food F2 contain 3 and 2 units of vitamin B respectively, so x unit of Food F1 and y units of food F2 contain 3x and 2y units of vitamin B respectively, but minimum daily requirement of vitamin B is 40 unit, so


3x + 2y ≥ 50 (Second constraint)


Hence, mathematical formulation of LPP is,


Find x and y which


Minimize Z = 5x + 2.5y


Subject to constraint,


2x + 4y ≥ 40


3x + 2y ≥ 50


x, y ≥ 0 [Since requirement of food F1 and F2 cannot be less zero.]



Question 10.

An automobile manufacturer makes automobiles and trucks in a factory that is divided into two shops. Shop A, which performs the basic assembly operation, must work 5 man - days on each truck but only 2 man - days on each automobile. Shop B which performs finishing operations, must work 3 man - days for each automobile or truck that it produces. Because of men and machine limitations, shop A has 180 man - days per week available while shop B has 135 man - days per week. If the manufacturer makes a profit of Rs 30000 on each truck and Rs 2000 on each automobile, how many of each should he produce to maximize his profit? Formulate this as a LPP.


Answer:

Let number of automobiles produces be x and let the number of trucks


Produced be y.


Let Z be the profit function to be maximized.


Z = 2000x + 30000y


The constraints are on the man hours worked


Shop A 2x + 5y ≤ 180 (i) assembly


Shop B 3x + 3y ≤ 135 (ii) finishing


x, y ≥ 0


Corner points can ve obtained from


2x = 3y + 5y = 180 ⇒ x = 0; y = 36 and x = 90; y = 0


3x + 3y ≤ 135 ⇒ x = 0; y = 45 and x = 45; y = 0


Solving (i) and (ii) gives x = 15 and y = 30



Thus 0 automobiles and 36 trucks give max. profit of Rs 10, 80, 000/ -



Question 11.

Two tailors A and B earn Rs 150 and Rs 200 per day respectively. A can stitch 6 shirts and 4 pants per day while B can stitch 10 shirts and 4 pants per day. Form a linear programming problem to minimize the labour cost to produce at least 60 shirts and 32 pants.


Answer:

The data can be represented in a table below



To minimize labour cost means to assume to minimize the earnings i.e,


Min Z = 150x + 200y


With constraints


x, y ≥ 0 at least 1 shirt and 1 pant is required


6x + 10y ≥ 60 require at least 60 shirts


4x + 4y ≥ 32 require at least 32 pants


On solving the above inequalities as equations, we get,


x = 5 and y = 3


other corner points obtained are [0, 6], [10, 0], [0, 8] and [8, 0]



The feasible region is the upper unbounded region A - E - D


Point E(5, 3) may not be minimal value. So, plot 150x + 200y < 1350 to see


If there is a common region with A - E - D.


The green line has no common point, therefore



Thus, stitching 5 shirts and 3 pants minimizes labour cost to Rs 1350/ -



Question 12.

An airline agrees to charter planes for a group. The group needs at least 160 first class seats and at least 300 tourist class seats. The airlines must use at least two of its model 314 planes which have 20 first class and 30 tourist class seats. The airline will also use some of its model 535 planes which have 20 first class seats and 60 tourist class seats. Each flight of a model 314 plane costs the company Rs 100, 000 and each flight of a model 535 plane costs Rs 150, 000. How many of each type of plane should be used to minimize the flight cost? Formulate this as a LPP.


Answer:


The above LPP can be presented in a table above.


The flight cost is to be minimized i.e; Min Z = x + 1.5y


the constraints


x ≥ 2 at least 2 planes of model 314 must be used


y ≥ 0 at least 1 plane of model .53.5 must be used


20x + 20y ≥ 160 require at least 160 F class seats


30x + 60y ≥ 300 require at least 300 T class seats


Solving the above inequalities as equations we get,


When x = 0, y = 8 and when y = 0, x = 8


When x = 0, y = 5 and when y = 0, x = 10


We get an unbounded region 8 - E - 10 as a feasible solution. Plotting the corner points and evaluating we have,



Since we obtained an unbounded region as the feasible solution a plot of Z (x + 1.5y< 9) is plotted.


Since there are no common points point E is the point that gives a minimum value.


Using 6 planes of model 314 & 2 of model 535 gives minimum cost of 9 lakh rupees.




Question 13.

Amit’s mathematics teacher has given him three long lists of problems with the instruction to submit not more than 100 of them (correctly solved) for credit. The problem in the first set are worth 5 points each, those in the second set are worth 4 points each, and those in the third set are worth 6 points each. Amit knows from experience that he requires on the average 3 minutes to solve a 5 point problem, 2 minutes to solve a 4 point problem, and 4 minutes to solve a 6 point problem. Because he has other subjects to worry about, he cannot afford to devote more than hours altogether to his mathematics assignment. Moreover, the first two sets of problems involve numerical calculations and he knows that he cannot stand more than hours work on this type of problem. Under these circumstances, how many problems in each of these categories shall he do in order to get maximum possible credit for his efforts? Formulate this as a LPP.


Answer:


Let he should solve x, y, z questions from set I, II and III respectively.


Given, each question from set I, II, III earn 5, 4, 6 points respectively, so x questions of set I, y questions of set II and z questions of set III earn 5x, 4y and 6z points, let total point credit be U


So, U = 5x + 4y + 6z


Given, each question of set I, II and III require 3, 2 and 4 minutes respectively, so, x questions of set I, y questions of set II and z questions of set III require 3x, 2y and 4z minutes respectively but given that total time to devote in all three sets is


hours = 210 minutes and first two sets is hours = 150 minutes


So,


3x + 2y + 4z ≤ 210 (First constraint)


3x + 2y ≤ 150 (Second constraint)


Given, total number of questions cannot exceed 100


So, x + y + z ≤ 100 (Third constraint)


Hence, mathematical formulation of LPP is


Find x and y which


maximize U = 5x + 4y + 6z


Subject to constraint,


3x + 2y + 4z ≤ 210


3x + 2y ≤ 150


x + y + z ≤ 100


x, y, z ≤ 0 [Since number of questions to solve from each set


cannot be less than zero.]



Question 14.

A farmer has a 100 - acre farm. He can sell the tomatoes, lettuce, or radishes he can raise. The price he can obtain is Rs 1 per kilogram for tomatoes, Rs 0.75 a head for lettuce and Rs 2 per kilogram for radishes. The average yield per acre is 2000 kgs for radishes, 3000 heads of lettuce and 1000 kilograms of radishes. Fertilizer is available at Rs 0.50 per kg and the amount required per acre is 100 kgs each for tomatoes and lettuce and 50 kilograms for radishes. Labour required for sowing, cultivating and harvesting per acre is 5 man - days for tomatoes and radishes and 6 man – days for lettuce. A total of 400 man - days of labour are available at ₹ 20 per man - day. Formulate this problem as a LPP to maximize the farmer’s total profit.


Answer:

Given information can be tabulated below



Let required quantity of field for tomatoes, lettuce and radishes be x, y and z


acre respectively.


Given, costs of cultivation and harvesting of tomatoes, lettuce and radishes are 5 x20 = Rs 100, 6 x20 = Rs 120, 5 x20 = Rs 100 respectively per acre. Cost of fertilizers for tomatoes, lettuce and radishes 100 x 0.50 = Rs 50, 100 x 0.50 = Rs 50 and 50 x0.50 = Rs 25 respectively per acre.


So, total costs of production of tomatoes, lettuce and radishes are (Rs 100 + 50)x = Rs 150x, (Rs 120 + 50)y = Rs 170y and radishes are (Rs 100 + 25)z = Rs 125z respectively total selling price of tomatoes, lettuce and radishes, according to yield are (Rs2000x1)x = Rs 2000x, (Rs3000 x0.75)y = Rs 2250y and (Rs1000x2)z = Rs 2000z respectively.


Let U be the total profit,


So,


U = (2000x - 150x) + (2250y - 170y) + (2000z - 125z)


U = 1850x + 2080y + 1875z


Given, farmer has 100 - acre farm


So, x + y + z ≤ 100 (First constraint)


Number of cultivation and harvesting days are 400 So, 5x + 6y + 5z ≤ 400


Hence, mathematical formulation of LPP is,


Find x, y, z which


maximize U = 1850x + 2080y + 1875z


Subject to constraint,


x + y + z ≤ 100


5x + 6y + 5z ≤ 400


x, y, z ≥ 0 [Since farm used for cultivation cannot be less than zero.]



Question 15.

A firm has to transport at least 1200 packages daily using large vans which carry 200 packages each and small vans which can take 80 packages each. The cost of engaging each large van is Rs 400 and each small van is Rs 200. Not more than Rs 3000 is to be spent daily on the job and the number of large vans cannot exceed the number of small vans. Formulate this problem as a LPP given that the objective is to minimize cost.


Answer:

Given information can be tabulated as below:



Let the required product of product A and B be x and y units respectively.


Given, labour cost and raw material cost of one unit of product A is Rs 16 and Rs 4, so total cost of product A is Rs 16 + Rs 4 = Rs 20


And given selling price of 1 unit of product A is Rs 25,


So, profit on one unit of product


A = 25 - 20 = Rs 5


Again, given labour cost and raw material cost of one unit of product B is Rs 20 and Rs 4 So, that cost of product B is Rs 20 + Rs 4 = Rs 24


And given selling price of 1 unit of product B is Rs 30


So, profit on one unit of product B = 30 - 24 = Rs 6


Hence, profits on x unit of product A and y units of product B are Rs 5x and Rs 6y respectively.


Let Z be the total profit, so,


Z = 5x + 6y


Given, production of one unit of product A and B need to process for 3 and 4 hours respectively in department 1, so production of x units of product A and y units of product B need to process for 3x and 4y hours respectively in Department 1. But total capacity of Department 1 is 130 hours,


So, 3x + 2y ≤ 130 (First constraint)


Given, production of one unit of product. A and B need to process for 4 and 6 hours respectively in department 2, so production of x units of product A and y units of product B need to process for 4x and 6y hours respectively in Department 2 but total capacity of Department 2 is 260 hours


So, 4x + 6y ≤ 260 (Second constraint)


Hence, mathematical formulation of LPP is,


Find x and y which


Maximize Z = 5x + 6y


Subject to constraint,


3x + 2y ≤ 130,


4x + 6y ≤ 260


x, y ≥ 0 [Since production cannot be less than zero]



Question 16.

A firm manufactures two products, each of which must be processed through two departments, 1 and 2. The hourly requirements per unit for each product in each department, the weekly capacities in each department, selling price per unit, labour cost per unit, and raw material cost per unit are summarized as follows :



The problem is to determine the number of units to produce each product so as to maximize total contribution to profit. Formulate this as a LPP.


Answer:

We have to maximize the profit by calculating the number of units needed to produce each product.


For Product A,


Cost Price per unit = Labour Cost + Raw material cost per unit


Cost Price per unit = 16 + 4 = Rs. 20


Selling Price per unit = Rs. 25


Profit per unit = S.P – C.P = 25 – 20 = Rs. 5


For Product B,


Cost Price per unit = Labour Cost + Raw material cost per unit


Cost Price per unit = 20 + 4 = Rs. 24


Selling Price per unit = Rs. 30


Profit per unit = S.P – C.P = 30 – 24 = Rs. 6


Let number of units produced of Product A be x and number of units produced of Product B be y.


Hence, Total Profit = 5 x + 6 y


To Maximize : z = 5 x + 6 y


For Department 1,


3 x + 2 y ≤ 130


For Department 2,


4 x + 6 y ≤ 260


Hence,


Z = 5 x + 6 y


3 x + 2 y ≤ 130


4 x + 6 y ≤ 260


x, y ≥ 0 [Since production cannot be less than zero]




Exercise 30.2
Question 1.

Solve each of the following linear programming problems by graphical method.

Maximize Z = 5x + 3y

Subject to :

3x + 5y ≤ 15

5x + 2y ≤ 10

x, y ≥ 0


Answer:

Given,


Objective function is: Z = 5x + 3y


Constraints are:


3x + 5y ≤ 15


5x + 2y ≤ 10


x, y ≥ 0


First convert the given inequations into corresponding equations and plot them:


3x + 5y≤ 15 → 3x + 5y = 15 (corresponding equation)


Two coordinates required to plot the equation are obtained as:


Put, x = 0 ⇒ y = 3 (0,3) - - - - first coordinate.


Put, y = 0 ⇒ x = 5 (5,0) - - - - second coordinate


Join them to get the line.


As we know, Linear inequation will be a region in the plane, and we observe that the equation divides the XY plane into 2 halves only, so we need to check which region represents the given inequation,


If the given line does not pass through origin the just put (0,0) to check whether inequation is satisfied or not. If it satisfies the inequation origin side is the required region else the other side is the solution.


Similarly, we repeat the steps for other inequation also and find the common region.


5x + 2y≤ 10 → 5x + 2y = 10 (corresponding equation)


Two coordinates required to plot the equation are obtained as:


Put, x = 0 ⇒ y = 5 (0,5) - - - - first coordinate.


Put, y = 0 ⇒ x = 2 (2,0) - - - - second coordinate


x = 0 is the y - axis and y = 0 is the x - axis


Hence we obtain a plot as shown in figure:



The shaded region in the above figure represents the region of a feasible solution.


Now to maximize our objective function, we need to find the coordinates of the corner points of the shaded region.


We can determine the coordinates graphically our by solving equations.


But choose only those equations to solve which gives one of the corner coordinates of the feasible region.


Solving 3x + 5y = 15 and 5x + 2y = 10 gives


Similarly solving 3x + 5y = 15 and x = 0 gives (0, 3)


And solving 5x + 2y = 10 and y = 0 gives (2,0)


And solving x = 0 and y = 0 gives (0,0)


Now we have coordinates of the corner points so we will put them one by one to our objective function and will find at which point it is maximum.


∵ Z = 5x + 3y


∴ Z at


Z at (0,3) = 5× 0 + 3× 3 = 9


Z at (2,0) = 5× 2 + 3× 0 = 10


Z at (0,0) = 0


We can see that Z is maximum at and max. value is


∴ x = ; y = and maximum the value of Z is



Question 2.

Solve each of the following linear programming problems by graphical method.

Maximize Z = 9x + 3y

Subject to :

2x + 3y ≤ 13

3x + y ≤ 5

x, y ≥ 0


Answer:

Given,


Objective function is: Z = 9x + 3y


Constraints are:


2x + 3y ≤ 13


3x + y ≤ 5


x, y ≥ 0


First convert the given inequations into corresponding equations and plot them:


2x + 3y ≤ 13 → 2x + 3y = 13 (corresponding equation)


Two coordinates required to plot the equation are obtained as:


Put, x = 0 ⇒ y = 3 (0,13/3) - - - - first coordinate.


Put, y = 0 ⇒ x = 13/2 (13/2,0) - - - - second coordinate


Join them to get the line.


As we know, Linear inequation will be a region in the plane, and we observe that the equation divides the XY plane into 2 halves only, so we need to check which region represents the given inequation,


If the given line does not pass through origin the just put (0,0) to check whether inequation is satisfied or not. If it satisfies the inequation origin side is the required region else the other side is the solution.


Similarly, we repeat the steps for other inequation also and find the common region.


3x + y≤ 5 → 3x + y = 5 (corresponding equation)


Two coordinates required to plot the equation are obtained as:


Put, x = 0 ⇒ y = 5 (0,5) - - - - first coordinate.


Put, y = 0 ⇒ x = 5/3 (5/3,0) - - - - second coordinate


x = 0 is the y - axis and y = 0 is the x – axis



Hence we obtain a plot as shown in figure:


The shaded region in the above figure represents the region of a feasible solution.


Now to maximize our objective function, we need to find the coordinates of the corner points of the shaded region.


We can determine the coordinates graphically our by solving equations. But choose only those equations to solve which gives one of the corner coordinates of the feasible region.


Solving 2x + 3y = 13 and 3x + y = 5 gives


Similarly solving 2x + 3y = 13 and x = 0 gives (0, 13/3)


And solving 3x + y = 5 and y = 0 gives (5/3,0)


And solving x = 0 and y = 0 gives (0,0)


Now we have coordinates of the corner points so we will put them one by one to our objective function and will find at which point it is maximum.


∵ Z = 9x + 3y


∴ Z at


Z at (0,13/3) = 9× 0 + 3× 13/3 = 13


Z at (5/3,0) = 9× (5/3) + 3× 0 = 15


Z at (0,0) = 0


We can see that Z is maximum at And max. value is 15


Z is also maximum at (5/3,0) with value = 15


This illustrates that 9x + 3y overlaps with 3x + y = 5 .


∴ Z is maximum at all the points on 3x + y = 5, and max value is 15.



Question 3.

Solve each of the following linear programming problems by graphical method.

Minimize Z = 18x + 10y

Subject to :

4x + y ≥ 20

2x + 3y ≥ 30

x, y ≥ 0


Answer:

Given,


Objective function is: Z = 18x + 10y


Constraints are:


4x + y ≥ 20


2x + 3y ≥ 30


x, y ≥ 0


First convert the given inequations into corresponding equations and plot them:


4x + y ≥ 20 → 4x + y = 20 (corresponding equation)


Two coordinates required to plot the equation are obtained as:


Put, x = 0 ⇒ y = 20 (0,20) - - - - first coordinate.


Put, y = 0 ⇒ x = 5 (5,0) - - - - second coordinate


Join them to get the line.


As we know, Linear inequation will be a region in the plane, and we observe that the equation divides the XY plane into 2 halves only, so we need to check which region represents the given inequation,


If the given line does not pass through origin the just put (0,0) to check whether inequation is satisfied or not. If it satisfies the inequation origin side is the required region else the other side is the solution.


Similarly, we repeat the steps for other inequation also and find the common region.


2x + 3y ≥ 30 → 2x + 3y = 30 (corresponding equation)


Two coordinates required to plot the equation are obtained as:


Put, x = 0 ⇒ y = 10 (0,10) - - - - first coordinate.


Put, y = 0 ⇒ x = 15 (15,0) - - - - second coordinate


x = 0 is the y - axis and y = 0 is the x – axis



Hence we obtain a plot as shown in figure:


The shaded region in the above figure represents the region of a feasible solution.


Now to maximize our objective function, we need to find the coordinates of the corner points of the shaded region.


We can determine the coordinates graphically our by solving equations. But choose only those equations to solve which gives one of the corner coordinates of the feasible region.


Solving 2x + 3y = 30 and 4x + y = 20 gives (3, 8)


Similarly solving 4x + y = 20 and x = 0 gives (0, 20)


And solving 2x + 3y = 30 and y = 0 gives (15,0)


Now we have coordinates of the corner points so we will put them one by one to our objective function and will find at which point it is maximum.


∵ Z = 18x + 10y


∴ Z at (3, 8) = 18 × 3 + 8 × 10 = 134


Z at (0,20) = 18 × 0 + 10 × 20 = 200


Z at (15,0) = 18 × (15) + 10 × 0 = 270


Note: It is unbounded so we can’t say directly by seeing that z = 134 is minimum because there might be possibility that some other points from feasible region may result a smaller number.


In such a case minima will not be possible.


So we check this by setting inequation corresponding to the minimum value of Z.



∴ inequation is 18x + 10y < 134


As (0,0) satisfies the inequation, so it will bound its region towards origin and hence will not overlap with the feasible region.


∴ We can say that minima are possible.


We can see that Z is minimum at (3, 8) and min. value is 134


∴ Z is minimum at x = 3 and y = 8 ; and min value is 134.



Question 4.

Solve each of the following linear programming problems by graphical method.

Maximize Z = 50x + 30y

Subject to :

2x + y ≤ 18

3x + 2y ≤ 34

x, y ≥ 0


Answer:

Given,


Objective function is: Z = 50x + 30y


Constraints are:


2x + y ≤ 18


3x + 2y ≤ 34


x, y ≥ 0


First convert the given inequations into corresponding equations and plot them:


2x + y ≤ 18 → 2x + y = 18 (corresponding equation)


Two coordinates required to plot the equation are obtained as:


Put, x = 0 ⇒ y = 18 (0,18) - - - - first coordinate.


Put, y = 0 ⇒ x = 9 (9,0) - - - - second coordinate


Join them to get the line.


As we know, Linear inequation will be a region in the plane, and we observe that the equation divides the XY plane into 2 halves only, so we need to check which region represents the given inequation,


If the given line does not pass through origin the just put (0,0) to check whether inequation is satisfied or not. If it satisfies the inequation origin side is the required region else the other side is the solution.


Similarly, we repeat the steps for other inequation also and find the common region.


3x + 2y ≤ 34 → 3x + 2y = 34 (corresponding equation)


Two coordinates required to plot the equation are obtained as:


Put, x = 0 ⇒ y = 17 (0, 17) - - - - first coordinate.


Put, y = 0 ⇒ x = 34/3 (34/3, 0) - - - - second coordinate


x = 0 is the y - axis and y = 0 is the x – axis



Hence we obtain a plot as shown in figure:


The shaded region in the above figure represents the region of a feasible solution.


Now to maximize our objective function, we need to find the coordinates of the corner points of the shaded region.


We can determine the coordinates graphically our by solving equations. But choose only those equations to solve which gives one of the corner coordinates of the feasible region.


Solving 3x + 2y = 34 and 2x + y = 18 gives (2,14)


Similarly solving 3x + 2y = 34 and x = 0 gives (0, 17)


And solving 2x + y = 18 and y = 0 gives (9,0)


And solving x = 0 and y = 0 gives (0,0)


Now we have coordinates of the corner points so we will put them one by one to our objective function and will find at which point it is maximum.


∵ Z = 50x + 30y


∴ Z at (2,14) = 50 × 2 + 30 × 14 = 520


Z at (0,17) = 50 × 0 + 30 × 17 = 510


Z at (9,0) = 50 × (9) + 30 × 0 = 450


Z at (0,0) = 0


We can see that Z is maximum at (2, 14) and max. value is 520


∴ Z is maximum at x = 2 and y = 14 ; and max value is 520.



Question 5.

Solve each of the following linear programming problems by graphical method.

Maximize Z = 4x + 3y

Subject to :

8x + 6y ≤ 48

3x + 4y ≤ 24

x ≤ 5 , y ≤ 6

x, y ≥ 0


Answer:

Given,


Objective function is: Z = 4x + 3y


Constraints are:


8x + 6y ≤ 48


3x + 4y ≤ 24


x ≤ 5


y ≤ 6


x, y ≥ 0


First convert the given inequations into corresponding equations and plot them:


8x + 6y ≤ 48 → 8x + 6y = 48 (corresponding equation)


Two coordinates required to plot the equation are obtained as:


Put, x = 0 ⇒ y = 8 (0,8) - - - - first coordinate.


Put, y = 0 ⇒ x = 6 (6,0) - - - - second coordinate


Join them to get the line.


As we know, Linear inequation will be a region in the plane, and we observe that the equation divides the XY plane into 2 halves only, so we need to check which region represents the given inequation,


If the given line does not pass through origin then just put (0,0) to check whether inequation is satisfied or not. If it satisfies the inequation origin side is the required region else the other side is the solution.


Similarly, we repeat the steps for other inequation also and find the common region.


3x + 4y ≤ 24 → 3x + 4y = 24 (corresponding equation)


Two coordinates required to plot the equation are obtained as:


Put, x = 0 ⇒ y = 6 (0,6) - - - - first coordinate.


Put, y = 0 ⇒ x = 8 (8,0) - - - - second coordinate


x = 0 is the y - axis and y = 0 is the x - axis


x = 5 and y = 6 are lines parallel to y - axis and x - axis respectively.



Hence we obtain a plot as shown in figure:


The shaded region in the above figure represents the region of a feasible solution.


Now to maximize our objective function, we need to find the coordinates of the corner points of the shaded region.


We can determine the coordinates graphically our by solving equations. But choose only those equations to solve which gives one of the corner coordinates of the feasible region.


Solving 3x + 4y = 24 and 8x + 6y = 48 gives


Similarly, solve other combinations by observing graph to get other coordinates.


From the figure we have obtained coordinates of corners as:


(0,0) (5,0) (0,6), ,


Now we have coordinates of the corner points so we will put them one by one to our objective function and will find at which point it is maximum.


∵ Z = 4x + 3y


∴ Z at


Z at (0,6) = 4 × 0 + 3 × 6 = 18


Z at (5,0) = 4 × (5) + 3 × 0 = 20


Z at (0,0) = 0


Z at


We can see that Z is maximum at and max. value is 24


∴ Z is maximum at x = 24/7 and y = 24/7 ; and max value is 24


Also it has a maximum at x = 5 and y = 4/3 with max value = 24



Question 6.

Solve each of the following linear programming problems by graphical method.

Maximize Z = 15x + 10y

Subject to :

2x + 3y ≤ 70

3x + 2y ≤ 80

x, y ≥ 0


Answer:

Given,


Objective function is: Z = 15x + 10y


Constraints are:


2x + 3y ≤ 70


3x + 2y ≤ 80


x, y ≥ 0


First convert the given inequations into corresponding equations and plot them:


2x + 3y ≤ 70 → 2x + 3y = 70 (corresponding equation)


Two coordinates required to plot the equation are obtained as:


Put, x = 0 ⇒ y = 70/3 (0,70/3) - - - - first coordinate.


Put, y = 0 ⇒ x = 35 (35,0) - - - - second coordinate


Join them to get the line.


As we know, Linear inequation will be a region in the plane, and we observe that the equation divides the XY plane into 2 halves only, so we need to check which region represents the given inequation,


If the given line does not pass through origin then just put (0,0) to check whether inequation is satisfied or not. If it satisfies the inequation origin side is the required region else the other side is the solution.


Similarly, we repeat the steps for other inequation also and find the common region.


3x + 2y ≤ 80 → 3x + 2y = 80 (corresponding equation)


Two coordinates required to plot the equation are obtained as:


Put, x = 0 ⇒ y = 40 (0,40) - - - - first coordinate.


Put, y = 0 ⇒ x = 80/3 (80/3,0) - - - - second coordinate


x = 0 is the y-axis and y = 0 is the x-axis



Hence, we obtain a plot as shown in figure:


The shaded region in the above figure represents the region of a feasible solution.


Now to maximize our objective function, we need to find the coordinates of the corner points of the shaded region.


We can determine the coordinates graphically our by solving equations. But choose only those equations to solve which gives one of the corner coordinates of the feasible region.


Solving 3x + 2y = 80 and 2x + 3y = 70 gives (20, 10)


Similarly solve other combinations by observing graph to get other coordinates.


From figure we have obtained coordinates of corners as:


(0,0) (80/3 ,0) (0,70/3),(20,10)


Now we have coordinates of the corner points so we will put them one by one to our objective function and will find at which point it is maximum.


∵ Z = 15x + 10y


∴ Z at (20, 10) = 15 × (20) + 10 × (10) = 400


Z at (0,70/3) = 15 × (70/3) + 10 × 0 = 350


Z at (80/3,0) = 15 × (80/3) + 10 × 0 = 400


Z at (0,0) = 0


Clearly, we can see that Z is maximum at and max. value is 400


∴ Z is maximum at x = 20 and y = 10 ; and max value is 400


Also it has a maximum at x = 80/3 and y = 0 with max value = 400



Question 7.

Solve each of the following linear programming problems by graphical method.

Maximize Z = 10x + 6y

Subject to :

2x + 5y ≤ 34

3x + y ≤ 12

x, y ≥ 0


Answer:

Given,


Objective function is: Z = 10x + 6y


Constraints are:


2x + 5y ≤ 34


3x + y ≤ 12


x, y ≥ 0


First convert the given inequations into corresponding equations and plot them:


2x + 5y ≤ 34 → 2x + 5y = 34 (corresponding equation)


Two coordinates required to plot the equation are obtained as:


Put, x = 0 ⇒ y = 34/5 (0,34/5) - - - - first coordinate.


Put, y = 0 ⇒ x = 17 (17,0) - - - - second coordinate


Join them to get the line.


As we know, Linear inequation will be a region in the plane, and we observe that the equation divides the XY plane into 2 halves only, so we need to check which region represents the given inequation,


If the given line does not pass through origin then just put (0,0) to check whether inequation is satisfied or not. If it satisfies the inequation origin side is the required region else the other side is the solution.


Similarly, we repeat the steps for other inequation also and find the common region.


3x + y ≤ 12 → 3x + y = 12 (corresponding equation)


Two coordinates required to plot the equation are obtained as:


Put, x = 0 ⇒ y = 12 (0,12) - - - - first coordinate.


Put, y = 0 ⇒ x = 12/3 = 4 (4,0) - - - - second coordinate


x = 0 is the y-axis and y = 0 is the x-axis



Hence, we obtain a plot as shown in figure:


The shaded region in the above figure represents the region of a feasible solution.


Now to maximize our objective function, we need to find the coordinates of the corner points of the shaded region.


We can determine the coordinates graphically our by solving equations. But choose only those equations to solve which gives one of the corner coordinates of the feasible region.


Solving 3x + y = 12 and 2x + 5y = 34 gives (2, 6)


Similarly solve other combinations by observing graph to get other coordinates.


From figure we have obtained coordinates of corners as:


(0, 0), (4, 0), (0, 34/5), (2, 6)


Now we have coordinates of the corner points so we will put them one by one to our objective function and will find at which point it is maximum.


∵ Z = 10x + 6y


∴ Z at (2, 6) = 10 × (2) + 6 × (6) = 56


Z at (0,34/5) = 10 × (0) + 6 × (34/5) = 204/5


Z at (4,0) = 10 × (4) + 10 × 0 = 40


Z at (0,0) = 0


We can see that Z is maximum at (2,6) and max. value is 56


∴ Z is maximum at x = 2 and y = 16 ; and max value is 56



Question 8.

Solve each of the following linear programming problems by graphical method.

Maximize Z = 3x + 4y

Subject to :

2x + 2y ≤ 80

2x + 4y ≤ 120


Answer:

Given,


Objective function is: Z = 3x + 4y


Constraints are:


2x + 2y ≤ 80


2x + 4y ≤ 120


First convert the given inequations into corresponding equations and plot them:


2x + 2y ≤ 80 → 2x + 2y = 80 (corresponding equation)


Two coordinates required to plot the equation are obtained as:


Put, x = 0 ⇒ y = 40 (0,40) - - - - first coordinate.


Put, y = 0 ⇒ x = 40 (40,0) - - - - second coordinate


Join them to get the line.


As we know, Linear inequation will be a region in a plane, and we observe that the equation divides the XY plane into 2 halves only, so we need to check which region represents the given inequation,


If the given line does not pass through origin then just put (0,0) to check whether inequation is satisfied or not. If it satisfies the inequation origin side is the required region else the other side is the solution.


Similarly, we repeat the steps for other inequation also and find the common region.


2x + 4y ≤ 120 → 2x + 4y = 120 (corresponding equation)


Two coordinates required to plot the equation are obtained as:


Put, x = 0 ⇒ y = 30 (0,30) - - - - first coordinate.


Put, y = 0 ⇒ x = 60 (60,0) - - - - second coordinate


x = 0 is the y-axis and y = 0 is the x-axis


Hence, we obtain a plot as shown in figure:



The shaded region in the above figure represents the region of a feasible solution.


Now to maximize our objective function, we need to find the coordinates of the corner points of the shaded region.


We can determine the coordinates graphically our by solving equations. But choose only those equations to solve which gives one of the corner coordinates of the feasible region.


Solving 2x + 4y = 120 and 2x + 2y = 80 gives (20, 20)


There are no other corners in the region obtained.


So maxima will occur at (20,20)


Now we have coordinates of the corner points so we will put them one by one to our objective function and will find at which point it is maximum.


∵ Z = 3x + 4y


∴ Z at (20,20) = 3×(20) + 4×(20) = 140


Note: As the region is unbounded, so we need to check whether maxima occurs or not.


For this we define inequation using the optimal function if the solution region of the inequation does not coincide with the feasible region, it means it has a maximum.



∴ inequation is : 3x + 4y > 140


Clearly, from the graph we observe that 3x + 4y>140 does not overlap with the feasible region


∴ Z is maximum at (20,20) and max. value is 140


∴ Z is maximum at x = 20 and y = 20 ; and max value is 140



Question 9.

Solve each of the following linear programming problems by graphical method.

Maximize Z = 7x + 10y

Subject to :

x + y ≤ 30000

y ≤ 12000

x ≥ 6000

x, y ≥ 0

x ≥ y


Answer:

Given,


Objective function is: Z = 7x + 10y


Constraints are:


x + y ≤ 30000


y ≤ 12000


x ≥ 6000


x, y ≥ 0


x ≥ y


First convert the given inequations into corresponding equations and plot them:


x + y ≤ 30000 → x + y = 30000 (corresponding equation)


Two coordinates required to plot the equation are obtained as:


Put, x = 0 ⇒ y = 30000 (0,30000) - - - - first coordinate.


Put, y = 0 ⇒ x = 30000 (30000,0) - - - - second coordinate


Join them to get the line.


As we know, Linear inequation will be a region in the plane, and we observe that the equation divides the XY plane into 2 halves only, so we need to check which region represents the given inequation,


If the given line does not pass through origin then just put (0,0) to check whether inequation is satisfied or not. If it satisfies the inequation origin side is the required region else the other side is the solution.


Similarly, we repeat the steps for other inequation also and find the common region.


For a line passing through origin, we put any other coordinate to check the region.


x ≥ y → x - y = 0 (corresponding equation)


Line passing through origin (0,0) and (1,1)


x = 0 is the y - axis and y = 0 is the x - axis


y = 12000 (line parallel to x - axis passing through (0,12000))


x = 6000 (line parallel to x – axis passing through (6000,0))


Hence, we obtain a plot as shown in figure:



The shaded region in the above figure represents the region of feasible solution.


Now to maximize our objective function, we need to find the coordinates of the corner points of the shaded region.


We can determine the coordinates graphically our by solving equations. But choose only those equations to solve which gives one of the corner coordinates of the feasible region.


Solving x + y = 30000 and y = 12000 gives (18000, 12000)


Similarly, solve other combinations by observing graph to get other coordinates.


From figure we have obtained coordinates of corners as:


(18000,12000) , (12000,12000) , (6000,0) , (6000,6000) and (30000,0)


Now we have coordinates of the corner points so we will put them one by one to our objective function and will find at which point it is maximum.


∵ Z = 7x + 10y


∴ Z at


Z at (12000,12000) = 7 × (12000) + 10 × (12000) = 204000


Z at (6000,0) = 7 × (6000) + 10 × 0 = 42000


Z at (6000,6000) = 7×6000 + 10×6000 = 67000


Z at (30000,0) = 7×30000 + 10×0 = 210000


Z at (0,0) = 0


We can see that Z is maximum at (18000, 20000) and max. value is 246000


∴ Z is maximum at x = 18000 and y = 20000 ; and max value is 246000



Question 10.

Solve each of the following linear programming problems by graphical method.

Maximize Z = 2x + 4y

Subject to :

x + y ≥ 8

x + 4y ≥ 12

x ≥ 3

y ≥ 2


Answer:

Given,


Objective function is: Z = 2x + 4y


Constraints are:


x + y ≥ 8


x + 4y ≥ 12


x ≥ 3


y ≥ 2


First convert the given inequations into corresponding equations and plot them:


x + y ≥ 8 → x + y = 8 (corresponding equation)


Two coordinates required to plot the equation are obtained as:


Put, x = 0 ⇒ y = 8 (0,8) - - - - first coordinate.


Put, y = 0 ⇒ x = 8 (8,0) - - - - second coordinate


Join them to get the line.


As we know, Linear inequation will be a region in the plane, and we observe that the equation divides the XY plane into 2 halves only, so we need to check which region represents the given inequation,


If the given line does not pass through origin then just put (0,0) to check whether inequation is satisfied or not. If it satisfies the inequation origin side is the required region else the other side is the solution.


Similarly, we repeat the steps for other inequation also and find the common region.


As, x + 4y ≥ 12 → x + 4y = 12


Put x = 0 ⇒ y = 3 coordinate - - - - - (0,3)


Put y = 0 ⇒ x = 12 coordinate - - - - - - (12,0)


y = 2 (line parallel to x - axis passing through (0,2))


x = 3 (line parallel to x – axis passing through (3,0))


Hence, we obtain a plot as shown in figure:



The shaded region in the above figure represents the region of feasible solution.


Now to maximize our objective function, we need to find the coordinates of the corner points of the shaded region.


We can determine the coordinates graphically our by solving equations. But choose only those equations to solve which gives one of the corner coordinates of the feasible region.


Solving x + y = 8 and x = 3 gives (3, 5)


Similarly, solve other combinations by observing graph to get other coordinates.


From the figure we have obtained coordinates of corners as:


(3,5) and (6,2)


Now we have coordinates of the corner points so we will put them one by one to our objective function and will find at which point it is maximum.


∵ Z = 2x + 4y


∴ Z at (3,5) = 2×(3) + 4×(5) = 26


Z at (6,2) = 2 × (6) + 4 × (8) = 20


Note: As the region is unbounded as we can’t say blindly that Z = 24 is maximum because there might be other points in the feasible region that may Make Z even greater.


So we need to check whether Z is maximum or not or Z greater than 24 or not.


For this we define inequation using the optimal function if the solution region of the inequation does not coincide with the feasible region, it means it has a maxima


Inequation : 2x + 4y > 24 or x + 2y > 12



Now we again plot the graph with the constraints and the above inequation


Clearly, x + 2y > 24 has solutions in feasible region.


This proves that the values of Z greater than 24 are possible.


∴ Optimal value of Z is not possible.



Question 11.

Solve each of the following linear programming problems by graphical method.

Minimize Z = 5x + 3y

Subject to :

2x + y ≥ 10

x + 3y ≥ 15

x ≤ 10

y ≤ 8

x, y ≥ 0


Answer:

Given,


Objective function is: Z = 5x + 3y


Constraints are:


2x + y ≥ 10


x + 3y ≥ 15


x ≤ 10


y ≤ 8


x, y ≥ 0


First convert the given inequations into corresponding equations and plot them:


2x + y ≥ 10 → 2x + y = 10 (corresponding equation)


Two coordinates required to plot the equation are obtained as:


Put, x = 0 ⇒ y = 10 (0,10) - - - - first coordinate.


Put, y = 0 ⇒ x = 5 (5,0) - - - - second coordinate


Join them to get the line.


As we know, Linear inequation will be a region in the plane, and we observe that the equation divides the XY plane into 2 halves only, so we need to check which region represents the given inequation,


If the given line does not pass through origin then just put (0,0) to check whether inequation is satisfied or not. If it satisfies the inequation origin side is the required region else the other side is the solution.


Similarly, we repeat the steps for other inequation also and find the common region.


For a line passing through origin, we put any other coordinate to check the region.


x + 3y ≥ 15 → x + 3y = 15 (corresponding equation)


Put x = 0 ⇒ y = 5 coordinate - - - - (0,5)


Put y = 0 ⇒ x = 15 coordinate - - - - (15,0)


x = 0 is the y - axis and y = 0 is the x - axis


y = 8 (line parallel to x - axis passing through (0,8))


x = 10 (line parallel to x – axis passing through (10,0))



Hence, we obtain a plot as shown in figure:


The shaded region in the above figure represents the region of feasible solution.


Now to maximize our objective function, we need to find the coordinates of the corner points of the shaded region.


We can determine the coordinates graphically our by solving equations. But choose only those equations to solve which gives one of the corner coordinates of the feasible region.


Solving x + 3y = 15 and 2x + y = 10 gives (3,4)


Similarly solve other combinations by observing graph to get other coordinates.


From figure we have obtained coordinates of corners as:


(3,4) , (10,8), (1,8) and (10, 5/3)


Now we have coordinates of the corner points so we will put them one by one to our objective function and will find at which point it is maximum.


∵ Z = 5x + 3y


∴ Z at (3,4) = 5×(3) + 3×(4) = 27


Z at (10,8) = 5 × (10) + 3 × (8) = 74


Z at (1,8) = 5 × (1) + 3 × 8 = 29


Z at (10,5/3) = 5 × 10 + 3 × 5/3 = 55


We can see that Z is minimum at (3,4) and min. value is 27


∴ Z is minimum at x = 3 and y = 4 ; and min value is 27



Question 12.

Solve each of the following linear programming problems by graphical method.

Minimize Z = 30x + 20y

Subject to :

x + y ≤ 8

x + 4y ≥ 12

5x + 8y = 20

x, y ≥ 0


Answer:

Given,


Objective function is: Z = 30x + 20y


Constraints are:


x + y ≤ 8


x + 4y ≥ 12


5x + 8y = 20


x, y ≥ 0


First convert the given inequations into corresponding equations and plot them:


x + y ≤ 8 → x + y = 8 (corresponding equation)


Two coordinates required to plot the equation are obtained as:


Put, x = 0 ⇒ y = 8 (0,8) - - - - first coordinate.


Put, y = 0 ⇒ x = 8 (8,0) - - - - second coordinate


Join them to get the line.


As we know, Linear inequation will be a region in the plane, and we observe that the equation divides the XY plane into 2 halves only, so we need to check which region represents the given inequation,


If the given line does not pass through origin then just put (0,0) to check whether inequation is satisfied or not. If it satisfies the inequation origin side is the required region else the other side is the solution.


Similarly, we repeat the steps for other inequation also and find the common region.


For a line passing through origin, we put any other coordinate to check the region.


x + 4y ≥ 12 → x + 4y = 12 (corresponding equation)


Put x = 0 ⇒ y = 3 coordinate - - - - (0,3)


Put y = 0 ⇒ x = 12 coordinate - - - - (12,0)


x = 0 is the y - axis and y = 0 is the x - axis


5x + 8y = 20


On putting x = 0 , y = 20/8 (0,2.5)


Putting y = 0 , x = 4 (4,0)



Hence, we obtain a plot as shown in figure:


The shaded region in the above figure represents the region of feasible solution.


Now to maximize our objective function, we need to find the coordinates of the corner points of the shaded region.


We can determine the coordinates graphically our by solving equations. But choose only those equations to solve which gives one of the corner coordinates of the feasible region.


Solving x + y = 8 and x + 4y = 12 gives (20/3,4/3)


Similarly, solve other combinations by observing graph to get other coordinates.


From figure we have obtained coordinates of corners as:


(0,3) , (0,8) and (20/3 , 4/3)


Now we have coordinates of the corner points so we will put them one by one to our objective function and will find at which point it is maximum.


∵ Z = 30x + 20y


∴ Z at (0,8) = 30×(0) + 20×(8) = 160


Z at (0,3) = 30 × (0) + 20 × (3) = 60


Z at (20/3,4/3) = 30 × (20/3) + 20 × 4/3 = 226.666We can see that Z is minimum at (0,3) and min. value is 60


∴ Z is minimum at x = 0 and y = 3 ; and min value is 60


Maximum value is 226.66 and it occurs at x = 20/3 and y = 4/3



Question 13.

Solve each of the following linear programming problems by graphical method.

Maximize Z = 4x + 3y

Subject to :

8x + 6y ≤ 48

3x + 4y ≤ 24

x ≤ 5 , y ≤ 6

x, y ≥ 0


Answer:

Ideas required to solve the problem:


• Fundamentals of plotting a linear equation. 2 coordinates are sufficient to plot a straight line.


• A linear inequation represents a region of XY plane when plotted.


• For linear programming we define various linear constraints and combining them we get a region in the XY plane which represents a region of feasible operation subject to various constraints.


• But our objective in the linear programming problem is to optimize (maximize or minimize) our objective function and it will be optimal only at one of the corner points of the feasible region. So we check the value of the objective function at every corner points and hence find maximum or minimum


Given,


Objective function is: Z = 4x + 3y


Constraints are:


8x + 6y ≤ 48


3x + 4y ≤ 24


x ≤ 5


y ≤ 6


x, y ≥ 0


First convert the given inequations into corresponding equations and plot them:


8x + 6y ≤ 48 → 8x + 6y = 48 (corresponding equation)


Two coordinates required to plot the equation are obtained as:


Put, x = 0 ⇒ y = 8 (0,8) - - - - first coordinate.


Put, y = 0 ⇒ x = 6 (6,0) - - - - second coordinate


Join them to get the line.


As we know, Linear inequation will be a region in the plane, and we observe that the equation divides the XY plane into 2 halves only, so we need to check which region represents the given inequation,


If the given line does not pass through origin then just put (0,0) to check whether inequation is satisfied or not. If it satisfies the inequation origin side is the required region else the other side is the solution.


Similarly, we repeat the steps for other inequation also and find the common region.


3x + 4y ≤ 24 → 3x + 4y = 24 (corresponding equation)


Two coordinates required to plot the equation are obtained as:


Put, x = 0 ⇒ y = 6 (0,6) - - - - first coordinate.


Put, y = 0 ⇒ x = 8 (8,0) - - - - second coordinate


x = 0 is the y - axis and y = 0 is the x - axis


x = 5 and y = 6 are lines parallel to y - axis and x - axis respectively.



Hence we obtain a plot as shown in figure:


The shaded region in the above figure represents the region of a feasible solution.


Now to maximize our objective function, we need to find the coordinates of the corner points of the shaded region.


We can determine the coordinates graphically our by solving equations. But choose only those equations to solve which gives one of the corner coordinates of the feasible region.


Solving 3x + 4y = 24 and 8x + 6y = 48 gives


Similarly, solve other combinations by observing graph to get other coordinates.


From the figure we have obtained coordinates of corners as:


(0,0) (5,0) (0,6), ,


Now we have coordinates of the corner points so we will put them one by one to our objective function and will find at which point it is maximum.


∵ Z = 4x + 3y


∴ Z at


Z at (0,6) = 4 × 0 + 3 × 6 = 18


Z at (5,0) = 4 × (5) + 3 × 0 = 20


Z at (0,0) = 0


Z at


We can see that Z is maximum at and max. value is 24


∴ Z is maximum at x = 24/7 and y = 24/7 ; and max value is 24


Also it has a maximum at x = 5 and y = 4/3 with max value = 24



Question 14.

Solve each of the following linear programming problems by graphical method.

Minimize Z = x - 5y + 20

Subject to :

x - y ≥ 0

- x + 2y ≥ 2

x ≥ 3

y ≤ 4

x, y ≥ 0


Answer:

Given,


Objective function is: Z = x - 5y + 20


Constraints are:


x - y ≥ 0


- x + 2y ≥ 2


x ≥ 3


y ≤ 4


x, y ≥ 0


First convert the given inequations into corresponding equations and plot them:


x - y ≥ 0 → x = y (corresponding equation)


Two coordinates required to plot the equation are obtained as:


Put, x = 0 ⇒ y = 0 (0,0) - - - - first coordinate.


Put, y = 1 ⇒ x = 1 (1,1) - - - - second coordinate


Join them to get the line.


As we know, Linear inequation will be a region in the plane, and we observe that the equation divides the XY plane into 2 halves only, so we need to check which region represents the given inequation,


If the given line does not pass through origin then just put (0,0) to check whether inequation is satisfied or not. If it satisfies the inequation origin side is the required region else the other side is the solution.


Similarly, we repeat the steps for other inequation also and find the common region.


- x + 2y ≥ 2 → - x + 2y = 2 (corresponding equation)


Two coordinates required to plot the equation are obtained as:


Put, x = 0 ⇒ y = 1 (0,1) - - - - first coordinate.


Put, y = 0 ⇒ x = - 2 ( - 2,0) - - - - second coordinate


x = 0 is the y - axis and y = 0 is the x - axis


x = 3 and y = 4 are lines parallel to y - axis and x - axis respectively.



Hence we obtain a plot as shown in figure:


The shaded region in the above figure represents the region of a feasible solution.


Now to maximize our objective function, we need to find the coordinates of the corner points of the shaded region.


We can determine the coordinates graphically our by solving equations. But choose only those equations to solve which gives one of the corner coordinates of the feasible region.


Solving x - y = 0 and x = 3 gives (3,3)


Similarly, solve other combinations by observing graph to get other coordinates.


From the figure we have obtained coordinates of corners as:


(3,3) (4,4) (6,4),


Now we have coordinates of the corner points so we will put them one by one to our objective function and will find at which point it is maximum.


∵ Z = x - 5y + 20


∴ Z at


Z at (3,3) = 3 – 5×3 + 20 = 8


Z at (4,4) = 4 - 5×4 + 20 = 4


Z at (6,4) = 6 – 5×4 + 20 = 6


We can see that Z is minimum at (4,4) and min. value is 4


∴ Z is minimum at x = 4 and y = 4 ; and min value is 4



Question 15.

Solve each of the following linear programming problems by graphical method.

Maximize Z = 3x + 5y

Subject to :

x + 2y ≤ 20

x + y ≤ 15

y ≤ 5

X, y ≥ 0


Answer:

Given,


Objective function is: Z = 3x + 5y


Constraints are:


x + 2y ≤ 20


x + y ≤ 15


y ≤ 5


x, y ≥ 0


First convert the given inequations into corresponding equations and plot them:


x + 2y ≤ 20 → x + 2y = 20 (corresponding equation)


Two coordinates required to plot the equation are obtained as:


Put, x = 0 ⇒ y = 10 (0,10) - - - - first coordinate.


Put, y = 0 ⇒ x = 20 (20,0) - - - - second coordinate


Join them to get the line.


As we know, Linear inequation will be a region in the plane, and we observe that the equation divides the XY plane into 2 halves only, so we need to check which region represents the given inequation,


If the given line does not pass through origin then just put (0,0) to check whether inequation is satisfied or not. If it satisfies the inequation origin side is the required region else the other side is the solution.


Similarly, we repeat the steps for other inequation also and find the common region.


x + y ≤ 15 → x + y = 15 (corresponding equation)


Two coordinates required to plot the equation are obtained as:


Put, x = 0 ⇒ y = 15 (0,15) - - - - first coordinate.


Put, y = 0 ⇒ x = 15 (15,0) - - - - second coordinate


x = 0 is the y - axis and y = 0 is the x - axis


y = 5 is line parallel to x - axis passing through (0,5).



Hence we obtain a plot as shown in figure:


The shaded region in the above figure represents the region of a feasible solution.


Now to maximize our objective function, we need to find the coordinates of the corner points of the shaded region.


We can determine the coordinates graphically our by solving equations. But choose only those equations to solve which gives one of the corner coordinates of the feasible region.


Solving x + y = 15 and x + 2y = 20 gives (10,5)


Similarly solve other combinations by observing graph to get other coordinates.


From figure we have obtained coordinates of corners as:


(0,0) (10,5) (15,0), (0,5)


Now we have coordinates of the corner points so we will put them one by one to our objective function and will find at which point it is maximum.


∵ Z = 3x + 5y


∴ Z at (0,0) = 0


Z at (10,5) = 3× 10 + 5×5 = 55


Z at (15,0) = 3×15 + 5× 0 = 45


Z at (0,5) = 3×0 + 5×5 = 25


We can see that Z is maximum at (10,5) and max. value is 55


∴ Z is maximum at x = 10 and y = 5 ; and max value is 55



Question 16.

Solve each of the following linear programming problems by graphical method.

Minimize Z = 3x1 + 5x2

Subject to :

x1 + 3x2 ≥ 3

x1 + x2 ≥ 2

x1, x2 ≥ 0


Answer:

Given,


Objective function is: Z = 3x1 + 5x2


Constraints are:


x1 + 3x2 ≥ 3


x1 + x2 ≥ 2


x1, x2 ≥ 0


First convert the given inequations into corresponding equations and plot them:


x1 + 3x2 ≥ 3 → x1 + 3x2 = 3 (corresponding equation)


Two coordinates required to plot the equation are obtained as:


Put, x1 = 0 ⇒ x2 = 1 (0,1) - - - - first coordinate.


Put, x2 = 0 ⇒ x1 = 3 (3,0) - - - - second coordinate


Join them to get the line.


As we know, Linear inequation will be a region in the plane, and we observe that the equation divides the XY plane into 2 halves only, so we need to check which region represents the given inequation,


If the given line does not pass through origin then just put (0,0) to check whether inequation is satisfied or not. If it satisfies the inequation origin side is the required region else the other side is the solution.


Similarly, we repeat the steps for other inequation also and find the common region.


x1 + x2 ≥ 2 → x1 + x2 = 2 (corresponding equation)


Two coordinates required to plot the equation are obtained as:


Put, x1 = 0 ⇒ x2 = 2 (0,2) - - - - first coordinate.


Put, x2 = 0 ⇒ x1 = 2 (2,0) - - - - second coordinate


x1 = 0 is the y-axis and x2 = 0 is the x-axis



Hence we obtain a plot as shown in figure:


The shaded region in the above figure represents the region of a feasible solution.


Now to maximize our objective function, we need to find the coordinates of the corner points of the shaded region.


We can determine the coordinates graphically our by solving equations. But choose only those equations to solve which gives one of the corner coordinates of the feasible region.


Solving x1 + x2 = 2 and x1 + 3x2 = 3 gives (3/2,1/2)


Similarly solve other combinations by observing graph to get other coordinates.


From the figure we have obtained coordinates of corners as:


(3/2,1/2) ,(3,0) and (0,2)


Now we have coordinates of the corner points so we will put them one by one to our objective function and will find at which point it is maximum.


∵ Z = 3x1 + 5x2


∴ Z at (3,0) = 3× 3 + 5× 0 = 9


Z at (0,2) = 3× 0 + 5×2 = 10


Z at (3/2,1/2) = 3×(3/2) + 5×(1/2) = 7


Note: As the region is unbounded, so we need to check whether any value less than 7 is possible for Z or not. If it is unique, we will say that under given constraints we found the minimum Z


For this we define inequation as: 3x + 2y < 7



As (0,0) satisfies the inequation, so this is the region specified by above inequation. As our feasible region does not coincide with the region specified by 3x + 2y < 7.


∴ Z can be minimized


We can see that Z is minimum at (3/2,1/2) and min. value is 7


∴ Z is minimum at x = 3/2 and y = 1/2 ; and min value is 7



Question 17.

Solve each of the following linear programming problems by graphical method.

Maximize Z = 2x + 3y

Subject to :

x + y ≥ 1

10x + y ≥ 5

x + 10y ≥ 1

x, y ≥ 0


Answer:

Given,


Objective function is: Z = 2x + 3y


Constraints are:


x + y ≥ 1


10x + y ≥ 5


x + 10y ≥ 1


x, y ≥ 0


First convert the given inequations into corresponding equations and plot them:


x + y ≥ 1 → x + y = 1 (corresponding equation)


Two coordinates required to plot the equation are obtained as:


Put, x = 0 ⇒ y = 1 (0,1) - - - - first coordinate.


Put, y = 0 ⇒ x = 1 (1,0) - - - - second coordinate


Join them to get the line.


As we know, Linear inequation will be a region in the plane, and we observe that the equation divides the XY plane into 2 halves only, so we need to check which region represents the given inequation,


If the given line does not pass through origin then just put (0,0) to check whether inequation is satisfied or not. If it satisfies the inequation origin side is the required region else the other side is the solution.


Similarly, we repeat the steps for other inequation also and find the common region.


10x + y ≥ 5 → 10x + y = 5 (corresponding equation)


Two coordinates required to plot the equation are obtained as:


Put, x = 0 ⇒ y = 5 (0,5) - - - - first coordinate.


Put, y = 0 ⇒ x = 1/2 (1/2,0) - - - - second coordinate


x + 10y ≥ 1 → x + 10y = 1 (corresponding equation)


Two coordinates required to plot the equation are obtained as:


Put, x = 0 ⇒ y = 1/10 (0,0.1) - - - - first coordinate.


Put, y = 0 ⇒ x = 1 (1,0) - - - - second coordinate


x = 0 is the y - axis and y = 0 is the x - axis



Hence, we obtain a plot as shown in figure:


The shaded region in the above figure represents the region of a feasible solution.


Now to maximize our objective function, we need to find the coordinates of the corner points of the shaded region.


We can determine the coordinates graphically our by solving equations. But choose only those equations to solve which gives one of the corner coordinates of the feasible region.


Solving x + y = 1 and 10x + y = 5 gives (4/9,5/9)


Similarly solve other combinations by observing graph to get other coordinates.


From the figure we have obtained coordinates of corners as:


(4/9,5/9) (1,0) and (0,5)


Now we have coordinates of the corner points so we will put them one by one to our objective function and will find at which point it is maximum.


∵ Z = 2x + 3y


∴ Z at (1,0) = 2


Z at (0,5) = 2× 0 + 3×5 = 15


Z at (4/9,5/9) = 2×(4/9) + 3× (5/9) = 23/9


Note: As the region is unbounded as we can’t say blindly that Z = 15 is maximum because there might be other points in the feasible region that may Make Z even greater.


So we need to check whether Z is maximum or not or Z greater than 15 or not.


For this we define inequation using the optimal function if the solution region of the inequation does not coincide with the feasible region, it means it has a maxima


Inequation : 2x + 3y > 15



We can see that the inequation 2x + 3y > 15 overlaps with a feasible region. So there are no maxima possible


If we want to check regarding a minimum of Z, from the corner we have a min of Z = 2



∴ required inequation is : 2x + 3y<2


Clearly no overlap with the feasible region is there for 2x + 3y < 2


∴ Z has minimum at x = 1 and y = 0 and min value of Z is 2



Question 18.

Solve each of the following linear programming problems by graphical method.

Maximize Z = - x1 + 2x2

Subject to :

- x1 + 3x2 ≤ 10

x1 + x2 ≤ 6

x1 - x2 ≤ 2

x1, x2 ≥ 0


Answer:

Given,


Objective function is: Z = - x1 + 2x2


Constraints are:


- x1 + 3x2 ≤ 10


x1 + x2 ≤ 6


x1 - x2 ≤ 2


x1, x2 ≥ 0


First convert the given inequations into corresponding equations and plot them:


- x1 + 3x2 ≤ 10 → - x1 + 3x2 = 10 (corresponding equation)


Two coordinates required to plot the equation are obtained as:


Put, x1 = 0 ⇒ x2 = 10/3 (0,10/3) - - - - first coordinate.


Put, x2 = 0 ⇒ x1 = - 10 ( - 10,0) - - - - second coordinate


x1 + x2 ≤ 6 → x1 + x2 = 6 (corresponding equation)


Two coordinates required to plot the equation are obtained as:


Put, x1 = 0 ⇒ x2 = 6 (0,6) - - - - first coordinate.


Put, x2 = 0 ⇒ x1 = 6 (6,0) - - - - second coordinate


x1 - x2 ≤ 2 → x1 - x2 = 2 (corresponding equation)


Two coordinates required to plot the equation are obtained as:


Put, x1 = 0 ⇒ x2 = - 2 (0, - 2) - - - - first coordinate.


Put, x2 = 0 ⇒ x1 = 2 (2,0) - - - - second coordinate


x1 ≥ 0 and x2 ≥ 0


Join them to get the line.


As we know, Linear inequation will be a region in the plane, and we observe that the equation divides the XY plane into 2 halves only, so we need to check which region represents the given inequation,


If the given line does not pass through origin then just put (0,0) to check whether inequation is satisfied or not. If it satisfies the inequation origin side is the required region else the other side is the solution.


Similarly, we repeat the steps for other inequation also and find the common region.


Hence, we obtain the following plot:



The shaded region in the above figure represents the region of a feasible solution.


Now to maximize our objective function, we need to find the coordinates of the corner points of the shaded region.


We can determine the coordinates graphically our by solving equations. But choose only those equations to solve which gives one of the corner coordinates of the feasible region.


Solving x1 - x2 = 2 and x1 + x2 = 6 gives (6,6)


Similarly solve other combinations by observing graph to get other coordinates.


From figure we have obtained coordinates of corners as:


(0,0),(2,0),(4,2),(2,4) and (0,10/3)


Now we have coordinates of the corner points so we will put them one by one to our objective function and will find at which point it is maximum.


∵ Z = - x1 + 2x2


∴ Z at (0,0) = 0


Z at (2,0) = - 2 + 2×0 = - 2


Z at (4,2) = - 4 + 2× 2 = 0


Z at (2,4) = - 2 + 2×4 = 6


Z at (0,10/3) = - 0 + 2× (10/3) = 20/3


We can see that Z is maximum at (0,10/3) and max. value is 20/3


∴ Z is maximum at x = 0 and y = 10/3 ; and max value is 20/3



Question 19.

Solve each of the following linear programming problems by graphical method.

Maximize Z = x + y

Subject to :

- 2x + y ≤ 1

x ≤ 2

x + y ≤ 3

x, y ≥ 0


Answer:

Given,


Objective function is: Z = x + y


Constraints are:


- 2x + y ≤ 1


x ≤ 2


x + y ≤ 3


x, y ≥ 0


First convert the given inequations into corresponding equations and plot them:


- 2x + y ≤ 1 → - 2x + y = 1 (corresponding equation)


Two coordinates required to plot the equation are obtained as:


Put, x = 0 ⇒ y = 1 (0,1) - - - - first coordinate.


Put, y = 0 ⇒ x = - 1/2 ( - 1/2,0) - - - - second coordinate


Join them to get the line.


As we know, Linear inequation will be a region in the plane, and we observe that the equation divides the XY plane into 2 halves only, so we need to check which region represents the given inequation,


If the given line does not pass through origin then just put (0,0) to check whether inequation is satisfied or not. If it satisfies the inequation origin side is the required region else the other side is the solution.


Similarly, we repeat the steps for other inequation also and find the common region.


x + y ≤ 3 → x + y = 3 (corresponding equation)


Two coordinates required to plot the equation are obtained as:


Put, x = 0 ⇒ y = 3 (0,3) - - - - first coordinate.


Put, y = 0 ⇒ x = 3 (3,0) - - - - second coordinate


x = 0 is the y - axis and y = 0 is the x - axis


x = 2 is line parallel to y - axis passing through (2,0).


Hence we obtain a plot as shown in figure:



The shaded region in the above figure represents the region of a feasible solution.


Now to maximize our objective function, we need to find the coordinates of the corner points of the shaded region.


We can determine the coordinates graphically our by solving equations. But choose only those equations to solve which gives one of the corner coordinates of the feasible region.


Solving - 2x + y = 1 and x + y = 3 gives (2/3,7/3)


Similarly, solve other combinations by observing graph to get other coordinates.


From the figure we have obtained coordinates of corners as:


(2/3,7/3), (0,1), (0,0), (2,0) and (2,1)


Now we have coordinates of the corner points so we will put them one by one to our objective function and will find at which point it is maximum.


∵ Z = x + y


∴ Z at (0,0) = 0


Z at (2,0) = 2 + 0 = 2


Z at (2,1) = 2 + 1 = 3


Z at (0,1) = 0 + 1 = 1


Z at (2/3,7/3) = 2/3 + 7/3 = 3


Clearly, we can see that Z is maximum at (2,1) and max. value is 3


∴ Z is maximum at x = 2 and y = 1 also at x = 2/3 and y = 7/3; and max value is 3



Question 20.

Solve each of the following linear programming problems by graphical method.

Maximize Z = - x1 + 2x2

Subject to :

- x1 + 3x2 ≤ 10

x1 + x2 ≤ 6

x1 - x2 ≤ 2

x1, x2 ≥ 0


Answer:

Given,


Objective function is: Z = 3x1 + 4x2


Constraints are:


x1 - x2 ≤ - 1


- x1 + x2 ≤ 0


x1 , x2 ≥ 0


First convert the given inequations into corresponding equations and plot them:


x1 - x2 ≤ - 1 → x1 - x2 = - 1 (corresponding equation)


Two coordinates required to plot the equation are obtained as:


Put, x1 = 0 ⇒ x2 = 1 (0,1) - - - - first coordinate.


Put, x2 = 0 ⇒ x1 = - 1 ( - 1,0) - - - - second coordinate


- x1 + x2 ≤ 0 → - x1 + x2 = 0 (corresponding equation)


Two coordinates required to plot the equation are obtained as:


Put, x1 = 0 ⇒ x2 = 0 (0,0) - - - - first coordinate.


Put, x2 = 1 ⇒ x1 = - 1 ( - 1,1) - - - - second coordinate


x1 ≥ 0 and x2 ≥ 0


Join them to get the line.


As we know, Linear inequation will be a region in the plane, and we observe that the equation divides the XY plane into 2 halves only, so we need to check which region represents the given inequation,


If the given line does not pass through origin then just put (0,0) to check whether inequation is satisfied or not. If it satisfies the inequation origin side is the required region else the other side is the solution.


Similarly, we repeat the steps for other inequation also and find the common region.



Hence, we obtain the following plot:


The given constraints don’t enclose any feasible region. No common shaded region so no maxima exists.



Question 21.

Solve each of the following linear programming problems by graphical method.

Maximize Z = 3x + 4y

Subject to :

x – y ≤ 1

x + y ≥ 3

x, y ≥ 0


Answer:

Given,


Objective function is: Z = 3x + 4y


Constraints are:


x – y ≤ 1


x + y ≥ 3


x, y ≥ 0


First convert the given inequations into corresponding equations and plot them:


x – y ≤ 1 → x – y = 1 (corresponding equation)


Two coordinates required to plot the equation are obtained as:


Put, x = 0 ⇒ y = - 1 (0, - 1) - - - - first coordinate.


Put, y = 0 ⇒ x = 1 (1,0) - - - - second coordinate


Join them to get the line.


As we know, Linear inequation will be a region in the plane, and we observe that the equation divides the XY plane into 2 halves only, so we need to check which region represents the given inequation,


If the given line does not pass through origin then just put (0,0) to check whether inequation is satisfied or not. If it satisfies the inequation origin side is the required region else the other side is the solution.


Similarly, we repeat the steps for other inequation also and find the common region.


x + y ≤ 3 → x + y = 3 (corresponding equation)


Two coordinates required to plot the equation are obtained as:


Put, x = 0 ⇒ y = 3 (0,3) - - - - first coordinate.


Put, y = 0 ⇒ x = 3 (3,0) - - - - second coordinate


x = 0 is the y-axis and y = 0 is the x-axis



Hence, we obtain a plot as shown in figure:


The shaded region in the above figure represents the region of a feasible solution.


Now to maximize our objective function, we need to find the coordinates of the corner points of the shaded region.


We can determine the coordinates graphically our by solving equations. But choose only those equations to solve which gives one of the corner coordinates of the feasible region.


Solving x + y = 3 and x = 0 gives (0,3)


Similarly solve other combinations by observing graph to get other coordinates.


From the figure we have obtained coordinates of corners as:


(0,3) and (3,0)


Now we have coordinates of the corner points so we will put them one by one to our objective function and will find at which point it is maximum.


∵ Z = 3x + 4y


∴ Z at (0,3) = 4×3 = 12


Z at (3,0) = 3×3 + 4×0 = 9


As the region is unbounded as we can’t say blindly that Z = 12 is maximum because there might be other points in the feasible region that may Make Z even greater.


So we need to check whether Z is maximum or not or Z greater than 12 or not.


For this we define inequation using the optimal function if the solution region of the inequation does not coincide with the feasible region, it means it has a maxima


Inequation: 3x + 4y > 12


It will be away from the origin side so that it will overlap with the feasible region.


∴ No maxima are possible.



Question 22.

Solve each of the following linear programming problems by graphical method.

Show the solution zone of the following inequalities on a graph paper:

5x + y ≥ 10

x + y ≥ 6

x + 4y ≥ 12

x ≥ 0, y ≥ 0

Find x and y for which 3x + 2y is minimum subject to these inequalities. Use a graphical method.


Answer:

Given,


Z = 3x + 2y


Constraints:


5x + y ≥ 10


x + y ≥ 6


x + 4y ≥ 12


x ≥ 0, y ≥ 0


First convert the given inequations into corresponding equations and plot them:


5x + y ≥ 10 → 5x + y = 10(corresponding equation)


Two coordinates required to plot the equation are obtained as:


Put, x = 0 ⇒ y = 10 (0,10) - - - - first coordinate.


Put, y = 0 ⇒ x = 2 (2,0) - - - - second coordinate


Join them to get the line.


As we know, Linear inequation will be a region in the plane, and we observe that the equation divides the XY plane into 2 halves only, so we need to check which region represents the given inequation,


If the given line does not pass through origin then just put (0,0) to check whether inequation is satisfied or not. If it satisfies the inequation origin side is the required region else the other side is the solution.


Similarly, we repeat the steps for other inequation also and find the common region.


x + y ≥ 6 → x + y = 6 (corresponding equation)


Two coordinates required to plot the equation are obtained as:


Put, x = 0 ⇒ y = 6 (0,6) - - - - first coordinate.


Put, y = 0 ⇒ x = 6 (6,0) - - - - second coordinate


x + 4y ≥ 12 → x + 4y = 12 (corresponding equation)


Two coordinates required to plot the equation are obtained as:


Put, x = 0 ⇒ y = 3 (0,3) - - - - first coordinate.


Put, y = 0 ⇒ x = 12 (12,0) - - - - second coordinate


x = 0 is the y - axis and y = 0 is the x - axis.


Hence, we have the following plot:



The shaded region in the above figure represents the region of a feasible solution.


Now to minimize our objective function, we need to find the coordinates of the corner points of the shaded region.


We can determine the coordinates graphically our by solving equations. But choose only those equations to solve which gives one of the corner coordinates of the feasible region.


Solving x + 4y = 12 and x + y = 6 gives (4,2)


Similarly solve other combinations by observing graph to get other coordinates.


From the figure we have obtained coordinates of corners as:


(4,2),(12,0),(1,5) and (0,10)


Now we have coordinates of the corner points so we will put them one by one to our objective function and will find at which point it is maximum.


∵ Z = 3x + 2y


∴ Z at (4,2) = 4×3 + 2×2 = 16


Z at (12,0) = 3×12 + 2×0 = 36


Z at (1,5) = 3× 1 + 2× 5 = 13


Z at (0,10) = 3× 0 + 2×10 = 20


As the region is unbounded as we can’t say blindly that Z = 13 is minimum because there might be other points in feasible region that may Make Z even lesser.


So we need to check whether Z is minimum or not or Z lesser than 13 or not.


For this we define inequation using the optimal function if the solution region of the inequation does not coincide with the feasible region, it means it has a maxima


Inequation: 3x + 2y < 13


It will be towards origin side, so it will not overlap with the feasible region.


∴ Minima are possible.


Minimum is possible and minimum occurs at x = 1 and y = 5 and value is Z = 13.



Question 23.

Solve each of the following linear programming problems by graphical method.

Find the maximum and minimum value of 2x + y subject to the constraints :

x + 3y ≥ 6,

x – 3y ≤ 3,

3x + 4y ≤ 24,

- 3x + 2y ≤ 6,

5x + y ≥ 5,

x, y ≥ 0.


Answer:

Given,


Z = 2x + y


Constraints:


x + 3y ≥ 6


x – 3y ≤ 3


3x + 4y ≤ 24


- 3x + 2y ≤ 6,


5x + y ≥ 5,


x, y ≥ 0.


First convert the given inequations into corresponding equations and plot them:


x + 3y ≥ 6 → x + 3y = 6 (corresponding equation)


Two coordinates required to plot the equation are obtained as:


Put, x = 0 ⇒ y = 2 (0,2) - - - - first coordinate.


Put, y = 0 ⇒ x = 6 (6,0) - - - - second coordinate


Join them to get the line.


As we know, Linear inequation will be a region in the plane, and we observe that the equation divides the XY plane into 2 halves only, so we need to check which region represents the given inequation,


If the given line does not pass through origin then just put (0,0) to check whether inequation is satisfied or not. If it satisfies the inequation origin side is the required region else the other side is the solution.


Similarly, we repeat the steps for other inequation also and find the common region.


x – 3y ≤ 3 → x – 3y = 3 (corresponding equation)


Two coordinates required to plot the equation are obtained as:


Put, x = 0 ⇒ y = - 1 (0, - 1) - - - - first coordinate.


Put, y = 0 ⇒ x = 3 (3,0) - - - - second coordinate


3x + 4y ≤ 24 → 3x + 4y = 24 (corresponding equation)


Two coordinates required to plot the equation are obtained as:


Put, x = 0 ⇒ y = 6 (0,6) - - - - first coordinate.


Put, y = 0 ⇒ x = 8 (8,0) - - - - second coordinate


- 3x + 2y ≤ 6 → - 3x + 2y = 6 (corresponding equation)


Two coordinates required to plot the equation are obtained as:


Put, x = 0 ⇒ y = 3 (0,3) - - - - first coordinate.


Put, y = 0 ⇒ x = - 2 ( - 2,0) - - - - second coordinate


5x + y ≥ 5 → 5x + y = 5, (corresponding equation)


Two coordinates required to plot the equation are obtained as:


Put, x = 0 ⇒ y = 5 (0,5) - - - - first coordinate.


Put, y = 0 ⇒ x = 1 (1,0) - - - - second coordinate


x = 0 is the y - axis and y = 0 is the x - axis.


Hence, we have the following plot:



The shaded region in the above figure represents the region of a feasible solution.


Now to minimize our objective function, we need to find the coordinates of the corner points of the shaded region.


We can determine the coordinates graphically our by solving equations. But choose only those equations to solve which gives one of the corner coordinates of the feasible region.


Solving 5x + y = 5 and - 3x + 2y = 6 gives (4/13,45/13)


Similarly solve other combinations by observing graph to get other coordinates.


From the figure we have obtained coordinates of corners as:


(4/13, 45/13) ,(4.5, 0.5), (9/14, 25/14), (4/3, 5), (84/13, 15/13)


Now we have coordinates of the corner points so we will put them one by one to our objective function and will find at which point it is maximum.


∵ Z = 2x + y


∴ Z at = 2×(4/13) + 45/13 = 57/13 = 4.38


Z at = 2×(9/2) + 1/2 = 19/2 = 9.5


Z at (9/14,25/14) = 2× (9/14) + (25/14) = 43/14 = 3.07


Z at = 2×(4/3) + 5 = 23/3 = 7.67


Z at 183/13 = 14.07


Clearly, Z is maximum at x = 84/13 and y = 15/13 and maximum value is 14.07


Z is minimum at x = 9/14 and y = 25/14 and minimum value is 3.07



Question 24.

Solve each of the following linear programming problems by graphical method.

Find the minimum value of 3x + 5y subject to the constraints

- 2x + y ≤ 4

x + y ≥ 3

x – 2y ≤ 2

x, y ≤0


Answer:

Given,


Z = 3x + 5y


Constraints:


- 2x + y ≤ 4


x + y ≥ 3


x – 2y ≤ 2


x, y ≤ 0


First convert the given inequations into corresponding equations and plot them:


- 2x + y ≤ 4 → - 2x + y = 4 (corresponding equation)


Two coordinates required to plot the equation are obtained as:


Put, x = 0 ⇒ y = 4 (0,4) - - - - first coordinate.


Put, y = 0 ⇒ x = - 2 ( - 2,0) - - - - second coordinate


Join them to get the line.


As we know, Linear inequation will be a region in the plane, and we observe that the equation divides the XY plane into 2 halves only, so we need to check which region represents the given inequation,


If the given line does not pass through origin then just put (0,0) to check whether inequation is satisfied or not. If it satisfies the inequation origin side is the required region else the other side is the solution.


Similarly, we repeat the steps for other inequation also and find the common region.


x + y ≥ 3 → x + y = 3 (corresponding equation)


Two coordinates required to plot the equation are obtained as:


Put, x = 0 ⇒ y = 3 (0,3) - - - - first coordinate.


Put, y = 0 ⇒ x = 3 (3,0) - - - - second coordinate


x – 2y ≤ 2 → x – 2y = 2 (corresponding equation)


Two coordinates required to plot the equation are obtained as:


Put, x = 0 ⇒ y = - 1 (0, - 1) - - - - first coordinate.


Put, y = 0 ⇒ x = 2 ( 2,0) - - - - second coordinate


x = 0 is the y - axis and y = 0 is the x - axis.


Hence, we have the following plot:



There is no shaded region in the above figure represents that there is no region of a feasible solution.


x + y ≥ 3 will not be bounded by x, y ≤ 0. Thus, no feasible region is there.


∴ There is no possible minimum value Z.



Question 25.

Solve each of the following linear programming problems by graphical method.

Solved the following linear programming problem graphically :

Maximize Z = 60x + 15y

Subject to constraints

x + y ≤ 50

3x + y ≤ 90

x, y ≥ 0


Answer:

Given,


Z = 60x + 15y


Constraints:


x + y ≤ 50


3x + y ≤ 90


x, y ≥ 0


First convert the given inequations into corresponding equations and plot them:


x + y ≤ 50 → x + y = 50 (corresponding equation)


Two coordinates required to plot the equation are obtained as:


Put, x = 0 ⇒ y = 50 (0,50) - - - - first coordinate.


Put, y = 0 ⇒ x = 50 (50,0) - - - - second coordinate


Join them to get the line.


As we know, Linear inequation will be a region in the plane, and we observe that the equation divides the XY plane into 2 halves only, so we need to check which region represents the given inequation,


If the given line does not pass through origin then just put (0,0) to check whether inequation is satisfied or not. If it satisfies the inequation origin side is the required region else the other side is the solution.


Similarly, we repeat the steps for other inequation also and find the common region.


3x + y ≤ 90 → 3x + y = 90 (corresponding equation)


Two coordinates required to plot the equation are obtained as:


Put, x = 0 ⇒ y = 90 (0,90) - - - - first coordinate.


Put, y = 0 ⇒ x = 30 (30,0) - - - - second coordinate


x = 0 is the y - axis and y = 0 is the x - axis.



Hence, we have the following plot:


The shaded region in the above figure represents the region of a feasible solution.


Now to minimize our objective function, we need to find the coordinates of the corner points of the shaded region.


We can determine the coordinates graphically our by solving equations. But choose only those equations to solve which gives one of the corner coordinates of the feasible region.


Solving x + y = 50 and 3x + y = 90 gives (20,30)


Similarly solve other combinations by observing graph to get other coordinates.


From the figure we have obtained coordinates of corners as:


(0,0),(30,0),(0,50) and (20,30)


Now we have coordinates of the corner points so we will put them one by one to our objective function and will find at which point it is maximum.


∵ Z = 60x + 15y


∴ Z at (0,0) = 0


Z at (30,0) = 60×30 + 15×0 = 1800


Z at (0,50) = 60×0 + 15×50 = 750


Z at (20,30) = 60×20 + 15×30 = 1650


Clearly Z is maximum at x = 30 and y = 0 and maximum value is 1800



Question 26.

Solve each of the following linear programming problems by graphical method.

Find graphically, the maximum value of Z = 2x + 5y, subject to constraints given below :

2x + 4y ≤ 8

3x + y ≤ 6

x + y ≤ 4

x ≥ 0, y ≥ 0


Answer:

Given,


Z = 2x + 5y


Constraints:


2x + 4y ≤ 8


3x + y ≤ 6


x + y ≤ 4


x ≥ 0, y ≥ 0


First convert the given inequations into corresponding equations and plot them:


2x + 4y ≤ 8 → 2x + 4y = 8 (corresponding equation)


Two coordinates required to plot the equation are obtained as:


Put, x = 0 ⇒ y = 2 (0,2) - - - - first coordinate.


Put, y = 0 ⇒ x = 4 (4,0) - - - - second coordinate


Join them to get the line.


As we know, Linear inequation will be a region in the plane, and we observe that the equation divides the XY plane into 2 halves only, so we need to check which region represents the given inequation,


If the given line does not pass through origin then just put (0,0) to check whether inequation is satisfied or not. If it satisfies the inequation origin side is the required region else the other side is the solution.


Similarly, we repeat the steps for other inequation also and find the common region.


3x + y ≤ 6 → 3x + y = 6 (corresponding equation)


Two coordinates required to plot the equation are obtained as:


Put, x = 0 ⇒ y = 6 (0,6) - - - - first coordinate.


Put, y = 0 ⇒ x = 2 (2,0) - - - - second coordinate


x + y ≤ 4 → x + y = 4 (corresponding equation)


Two coordinates required to plot the equation are obtained as:


Put, x = 0 ⇒ y = 4 (0,4) - - - - first coordinate.


Put, y = 0 ⇒ x = 4 (4,0) - - - - second coordinate


x = 0 is the y - axis and y = 0 is the x - axis.


Hence, we have the following plot:



The shaded region in the above figure represents the region of a feasible solution.


Now to minimize our objective function, we need to find the coordinates of the corner points of the shaded region.


We can determine the coordinates graphically our by solving equations. But choose only those equations to solve which gives one of the corner coordinates of the feasible region.


Solving 2x + 4y = 8 and 3x + y = 6 gives (8/5,6/5)


Similarly solve other combinations by observing graph to get other coordinates.


From the figure we have obtained coordinates of corners as:


(0,0),(2,0),(0,2) and (8/5,6/5)


Now we have coordinates of the corner points so we will put them one by one to our objective function and will find at which point it is maximum.


∵ Z = 2x + 5y


∴ Z at (0,0) = 0


Z at (2,0) = 2×2 + 5×0 = 4


Z at (0,2) = 2×0 + 5×2 = 10


Z at (8/5,6/5) = 2×(8/5) + 5×(6/5) = 46/5 = 9.2


Clearly Z is maximum at x = 0 and y = 2 and maximum value is 10



Question 27.

Solve each of the following linear programming problems by graphical method.

Solve the following LPP graphically :

Maximize Z = 20x + 10y

Subject to the following constraints

x + 2y ≤ 28

3x + y ≤ 24

x ≥ 2


Answer:

Given,


Z = 20x + 10y


Constraints:


x + 2y ≤ 28


3x + y ≤ 24


x ≥ 2


First convert the given inequations into corresponding equations and plot them:


x + 2y ≤ 28 → x + 2y = 28 (corresponding equation)


Two coordinates required to plot the equation are obtained as:


Put, x = 0 ⇒ y = 14 (0,14) - - - - first coordinate.


Put, y = 0 ⇒ x = 28 (28,0) - - - - second coordinate


Join them to get the line.


As we know, Linear inequation will be a region in the plane, and we observe that the equation divides the XY plane into 2 halves only, so we need to check which region represents the given inequation,


If the given line does not pass through origin then just put (0,0) to check whether inequation is satisfied or not. If it satisfies the inequation origin side is the required region else the other side is the solution.


Similarly, we repeat the steps for other inequation also and find the common region.


3x + y ≤ 24 → 3x + y = 24 (corresponding equation)


Two coordinates required to plot the equation are obtained as:


Put, x = 0 ⇒ y = 24 (0,24) - - - - first coordinate.


Put, y = 0 ⇒ x = 8 (8,0) - - - - second coordinate


x = 2 is the line parallel to y-axis passing through (2,0)


Hence, we have the following plot:



The shaded region in the above figure represents the region of a feasible solution.


Now to minimize our objective function, we need to find the coordinates of the corner points of the shaded region.


We can determine the coordinates graphically our by solving equations. But choose only those equations to solve which gives one of the corner coordinates of the feasible region.


Solving x + 2y = 28 and 3x + y = 24 gives (4,12)


Similarly solve other combinations by observing graph to get other coordinates.


From the figure we have obtained coordinates of corners as:


(2,0),(8,0),(2,13) and (4,12)


Now we have coordinates of the corner points so we will put them one by one to our objective function and will find at which point it is maximum.


∵ Z = 20x + 10y


∴ Z at (2,0) = 20× 2 + 10× 0 = 40


Z at (8,0) = 20×8 + 10×0 = 160


Z at (2,13) = 20×2 + 10×13 = 170


Z at (4,12) = 20×(4) + 10×(12) = 200


Clearly Z is maximum at x = 4 and y = 12 and maximum value is 200



Question 28.

Solve each of the following linear programming problems by graphical method.

Solve the following linear programming problem graphically :

Minimize z = 6x + 3y

Subject to the constraint

4x + y ≥ 80

x + 5y ≥ 115

3x + 2y ≤ 150

x ≥ 0, y ≥ 0


Answer:

Given,


z = 6x + 3y


Constraints:


4x + y ≥ 80


x + 5y ≥ 115


3x + 2y ≤ 150


x ≥ 0, y ≥ 0


First convert the given inequations into corresponding equations and plot them:


4x + y ≥ 80 → 4x + y = 80 (corresponding equation)


Two coordinates required to plot the equation are obtained as:


Put, x = 0 ⇒ y = 80 (0,80) - - - - first coordinate.


Put, y = 0 ⇒ x = 20 (20,0) - - - - second coordinate


Join them to get the line.


As we know, Linear inequation will be a region in the plane, and we observe that the equation divides the XY plane into 2 halves only, so we need to check which region represents the given inequation,


If the given line does not pass through origin then just put (0,0) to check whether inequation is satisfied or not. If it satisfies the inequation origin side is the required region else the other side is the solution.


Similarly, we repeat the steps for other inequation also and find the common region.


x + 5y ≥ 115 → x + 5y = 115 (corresponding equation)


Two coordinates required to plot the equation are obtained as:


Put, x = 0 ⇒ y = 23 (0,23) - - - - first coordinate.


Put, y = 0 ⇒ x = 115 (115,0) - - - - second coordinate


3x + 2y ≤ 150 3x + 2y = 150 (corresponding equation)


Two coordinates required to plot the equation are obtained as:


Put, x = 0 ⇒ y = 75 (0,75) - - - - first coordinate.


Put, y = 0 ⇒ x = 50 (50,0) - - - - second coordinate


x = 0 is y - axis and y = 0 is the x – axis.



Hence, we have the following plot:


The shaded region in the above figure represents the region of a feasible solution.


Now to minimize our objective function, we need to find the coordinates of the corner points of the shaded region.


We can determine the coordinates graphically our by solving equations. But choose only those equations to solve which gives one of the corner coordinates of the feasible region.


Solving x + 5y = 115 and 3x + 2y = 150 gives (40,15)


Similarly solve other combinations by observing graph to get other coordinates.


From the figure we have obtained coordinates of corners as:


(15,20),(40,15) and (2,72)


Now we have coordinates of the corner points so we will put them one by one to our objective function and will find at which point it is maximum.


∵ Z = 6x + 3y


∴ Z at (15,20) = 6× 15 + 3×20 = 150


Z at (40,15) = 6×40 + 3×15 = 285


Z at (2,72) = 6×2 + 10×72 = 732


Z is minimum at x = 15, and y = 20 and the minimum value is 150




Exercise 30.3
Question 1.

A diet of two foods F1 and F2 contains nutrients thiamine, phosphorous and iron. The amount of each nutrient in each of the food (in milligrams per 25 gms) is given in the following table :



The minimum requirement of the nutrients in the diet is 1.00 mg of thiamine, 7.50 mg of phosphorous and 10.00 mg of iron. The cost of F1 is 20 paise per 25 gms while the cost of F2 is 15 paise per 25 gms. Find the minimum cost of diet.


Answer:

The information above can be expressed in the following table:



Let the amount of food F1 and F2 required to be ‘x’ and ‘y’ units.


Cost of F1 = 0.20x


Cost of F2 = 0.15y


So, Cost of diet = 0.20x + 0.15y


Now,


⟹ 0.25x + 0.10y ≥ 1.00


i.e. the minimum requirement of Thiamine should be 1.00mg, from both the foods F1 and F2, each of which have 0.25mg and 0.10mg of Thiamine respectively. So, this is the first constraint.


⟹ 0.75x +1.50y ≥ 7.50


i.e. the minimum requirement of Phosphorous should be 7.50mg, from both the foods F1 and F2, each of which have 0.75mg and 1.50mg of Phosphorous respectively. This is the second constraint.


⟹ 1.60x + 0.80y ≥ 10.00


i.e. the minimum requirement of Iron should be 10.00mg, from both the foods F1 and F2, each of which have 1.60mg and 0.8mg of Iron respectively.


Hence, mathematical formulation of LPP is as follows:


Find ‘x’ and ‘y’ which


Minimise Z = 0.20x + 0.15y


Subject to the following constraints:


(i) 0.25x + 0.10y ≥ 1.00


(ii) 0.75x +1.50y ≥ 7.50


(iii) 1.60x + 0.80y ≥ 10.00


(iv) x,y ≥0 (∵ quantity cant be negative)



The feasible region is unbounded


The end points of the feasible region are as follows:



So, Z is smallest at B(5,2.5)


Let us consider the inequation 0.20x + 0.15y ≤ 1.375


As this has no intersection with the feasible region, the smallest value is the minimum value.


So, the minimum cost of diet is ₹1.375



Question 2.

A diet for a sick person must contain at least 4000 units of vitamins, 50 units of minerals and 1400 of calories. Two foods A and B, are available at the cost of ₹ 4 and ₹ 3 per unit respectively. If one unit of A contains 200 units of vitamin, 1 unit of mineral and 40 calories and one unit of food B contains 100 units of vitamin, 2 units of minerals and 40 calories, find what combination of foods should be used to have the least cost?


Answer:

The above information can be expressed in the form of the following table:



Let the quantity of the foods be ‘x’ and ‘y’ respectively.


Cost of food A = 4x


Cost of food B = 3y


Total cost of the combination = 4x + 3y


Now,


⟹ 200x + 100y ≥ 4000


i.e. the minimum requirement of vitamins from the two foods should be 4000.


⟹ x + 2y ≥ 50


i.e. the minimum requirement of minerals from the two foods should be 50.


⟹ 40x + 40y ≥ 1400


i.e. the minimum requirement of calories from the two foods should be 1400


Hence, mathematical formulation of LPP is as follows:


Find ‘x’ and ‘y’ which


Minimize Z = 4x + 3y


Subject to the following constraints:


(i) 200x + 100y ≥ 4000


i.e. 2x + y ≥ 40


(ii) x + 2y ≥ 50


(iii) 40x + 40y ≥ 1400


i.e. x + y ≥ 35


(iv) x,y ≥ 0 (∵ quantity cant be negative)



The feasible region is unbounded.


The corner points of the feasible region is as follows:



Z is smallest at B(5,30)


Let us consider 4x + 3y ≤ 110


As it has no intersection with the feasible region, the smallest value is the minimum value.


The minimum cost of foods is ₹110



Question 3.

To maintain one’s health, a person must fulfill certain minimum daily requirement for the following three nutrients : calcium, protein and calories. The diet consists of only items I and II whose prices and nutrient contents are shown below :



Answer:

Let the quantity of foods chosen be ‘x’ and ‘y’


Cost of food X = 0.6x


Cost of food Y = y


Cost of diet = 0.6x + y


Now,


⟹ 10x + 4y ≥ 20


i.e. the minimum daily requirement of calcium in the diet is 20 units.


⟹ 5x + 6y ≥ 20


i.e. the minimum daily requirement of protein in the diet is 20 units.


⟹ 2x + 6y ≥ 12


i.e. the minimum daily requirement of calories in the diet is 12 units.


Hence, mathematical formulation of the LPP is as follows:


Find ‘x’ and y’ such that


Minimises Z = 0.6x + y


Subject to the following constraints:


(i) 10x + 4y ≥ 20


(ii) 5x + 6y ≥ 20


(iii) 2x + 6y ≥ 12


(iv) x,y ≥ 0 (∵ quantity cant be negative)



The feasible region is unbounded


The corner points of the feasible region is as follows:



Z is smallest at


Let us consider 0.6x + y ≤ 2.712.


As it has no intersection with the feasible region, the smallest value is the minimum value.


The minimum value of Z is ₹2.712



Question 4.

A hospital dietician wishes to find the cheapest combination of two foods, A and B, that contains at least 0.5 milligram of thiamine and at least 600 calories. Each unit of A contains 0.12 milligram of thiamine and 100 calories, while each unit of B contains 0.10 milligram of thiamine and 150 calories. If each food costs 10 paise per unit, how many units of each should be combined at a minimum cost?


Answer:

The above information can be expressed using the following table:



Let the quantity of the foods A and B be ‘x’ and ‘y’ respectively.


Cost of food A = 0.10x


Cost of food B = 0.10y


Cost of diet = 0.10x + 0.10y


Now,


⟹ 0.12x + 0.10y ≥ 0.5


i.e. the minimum requirement of thiamine in the foods is 0.5mg


⟹ 100x + 150y ≥ 600


i.e. the minimum requirement of calories in the foods is 600.


Hence, mathematical formulation of the LPP is as follows:


Find ‘x’ and ‘y’ that:


Minimises Z = 0.10x + 0.10y


Subject to the following constraints:


(i) 0.12x + 0.10y ≥ 0.5


(ii) 100x + 150y ≥ 600


i.e. 2x + 3y ≥ 12


(iii) x,y ≥ 0 (∵ quantity cant be negative)



The feasible region is unbounded.


The corner points of the feasible region is as follows:



Z is smallest at A(1.875,2.75)


Let us consider 0.1x+0.1y ≤ 0.4625


As it has no intersection with the feasible region, the smallest value is the minimum value.


The minimum cost of the foods is ₹0.4625



Question 5.

A dietician mixes together two kinds of food in such a way that the mixture contains at least 6 units of vitamin A, 7 units of vitamin B, 11 units of vitamin C and 9 units of vitamin D. The vitamin contents of 1 kg of food X and 1 kg of food Y are given below :



One kg of food X costs ₹ 5, whereas one kg of food Y costs ₹ 8. Find the least cost of the mixture which will produce the desired diet.


Answer:

The above information can be expressed with the help of the following table:



Let the quantity of foods X and Y be ‘x’ and ‘y’.


Cost of food X = 5x


Cost of food Y = 8y


Cost of the meal 5x+8y


Now,


⟹ x + 2y ≥ 6


i.e. the minimum requirement of Vitamin A in the foods X and Y is 6units, each of which has 1unit and 2 unit of Vitamin A.


⟹ x + y ≥ 7


i.e. the minimum requirement of Vitamin B in the two foods is 7units, each of which has 1 unit of Vitamin B.


⟹ x + 3y ≥ 11


i.e. the minimum requirement of vitamin C in the two foods is 11units, each of which has 1 unit and 3 units of vitamin C.


⟹ 2x + y ≥ 9


i.e. the minimum requirement of Vitamin D in the foods is 9units, each of which has 2 units and 1 unit of Vitamin D.


Hence, mathematical formulation of the LPP is as follows:


Find ‘x’ and ‘y’ that


Minimises Z = 5x + 8y


Subject to the following constraints:


(i) x + 2y ≥ 6


(ii) x + y ≥ 7


(iii) x + 3y ≥ 11


(iv) 2x + y ≥ 9


(v) x,y ≥ 0 (∵ quantity cant be negative)



The feasible region is unbounded.


The corner points of the feasible region are as follows:



Z is smallest at C(5,2)


Let us consider 5x + 8y ≤ 41.


As it has no intersection with the feasible region, the smallest value is the minimum value.


The minimum cost of the diet is ₹41



Question 6.

A diet is to contain at least 80 units of vitamin A and 100 units of minerals. Two foods F1 and F2 are available. Food F2 costs ₹ 4 per unit F2 costs ₹ 6 per unit one unit of food F1 contains 3 units of vitamin A and 4 units of minerals. One unit of food F2 contains 6 units of vitamin A and 3 units of minerals. Formulate this as a linear programming problem and find graphically the minimum cost for diet that consists of mixture of these foods and also meets the mineral nutritional requirements.


Answer:

The above information can be expressed in the following table:



Let the quantity of the foods F1 and F2 be ‘x’ and ‘y’ respectively.


Cost of food F1 = 4x


Cost of food F2 = 6y


Cost of Diet = 4x + 6y


Now,


⟹ 3x + 6y ≥ 80


i.e. the minimum requirement of Vitamins from the two foods is 80units, each of which contains 3units and 6units of Vitamins.


⟹ 4x + 3y ≥ 100


i.e. the minimum requirement of minerals firm the two foods is 100units, each of which contains 4unit and 3 units of vitamins.


Hence, mathematical formulation of the LPP is as follows:


Find ‘x’ and ‘y’ that:


Minimise Z = 4x + 6y


Subject to the following constraints:


(i) 3x + 6y ≥ 80


(ii) 4x + 3y ≥ 100


(iii) x,y ≥ 0 (∵ quantity cant be negative)



The feasible region is unbounded.


The corner points of the feasible region is as follows:



Z is smallest at


Let us consider 4x + 6y ≤ 104


As it has no intersection with the feasible region, the smallest value is the minimum value.


The minimum cost of diet is ₹104.



Question 7.

Kellogg is a new cereal formed of a mixture of bran and rice that contains at least 88 grams of protein and at least 36 milligrams of iron. Knowing that bran contains 80 grams of protein and 40 milligrams of iron per kilogram, and that rice contains 100 grams of protein and 30 milligrams of iron per kilogram, find the minimum cost of producing this new cereal if bran costs ₹ 5 per kg and rice costs ₹ 4 per kg.


Answer:

The above information can be expressed using the following table:



Let the amount of Bran and Rice required be ‘x’ and ‘y’ kgs respectively.


Cost of Bran = 5x


Cost of Rice = 4y


Cost of the cereal = 5x + 4y


Now,


⟹ 80x + 100y ≥ 88


i.e. the minimum requirement of protein in the cereal, from Bran and Rice combined, is 88g, each of which have 80g and 100g of proteins respectively.


⟹ 40x + 30y ≥ 36


i.e. the minimum requirement of iron in the cereal, from Bran and Rice combined, is 36mg, each of which have 40mg and 30mg of iron.


Hence, mathematical formulation of LPP is as follows:


Find ‘x’ and ‘y’ that:


Minimises Z = 5x + 4y


Subject to the following constraints:


(i) 80x + 100y ≥ 88


(ii) 40x + 30y ≥ 36


(iii) x,y ≥ 0 (∵ quantity cant be negative)



The feasible region is unbounded.


The corner points of the feasible region is as follows:



Z is smallest at B(0.6,0.4)


Let us consider 5x + 4y ≤ 4.6


As it has no intersection with the feasible region, the smallest value is the minimum value.


The minimum cost of the cereal is ₹4.6



Question 8.

A wholesale dealer deals in two kinds, A and B (say) of mixture of nuts. Each kg of mixture A contains 60 grams of almonds, 30 grams of cashew nuts and 30 grams of hazel nuts. Each kg of mixture B contains 30 grams of almonds, 60 grams of cashew nuts and 180 grams of hazel nuts. The remainder of both mixtures is per nuts. The dealer is contemplating to use of cashew nuts and 540 grams of hazel nuts. Mixture A costs ₹ 8 per kg. and mixture B costs ₹ 12 per kg. Assuming that mixtures A and B are uniform, use graphical method to determine the number of kg. of each mixture he should use to minimize the cost of the bag.


Answer:

The above information can be expressed in the form of the following table:



Let the number of bags chosen of A and B be ‘x’ and ‘y’ respectively.


Cost of Bag A = 8x


Cost of Bag B = 12y


Total Cost of Bags = 8x + 12 y


Now,


⟹ 60x + 30y ≥ 240


i.e. the minimum requirement of almonds from both the bags is 240g, each of which contains 60g and 30g of almonds respectively.


⟹ 30x + 60y ≥ 300


i.e. the minimum requirement of Cashew Nuts from both the bags is 300g, each of which contains 30g and 60g of cashew nuts respectively.


⟹ 30x + 180y ≥ 540


i.e. the minimum requirement of Hazel Nuts from both the bags is 540g, each of which contains 30g and 180g of hazelnut respectively.


Hence, mathematical formulation of the LPP is as follows:


Find ‘x’ and ‘y’ that


Minimises Z = 8x + 12y


Subject to the following constraints:


(i) 60x + 30y ≥ 240


i.e. 2x + y ≥ 8


(ii) 30x + 60y ≥ 300


i.e. x + 2y ≥ 10


(iii) 30x + 180y ≥ 540


i.e. x + 6y ≥ 18


(iv) x,y ≥ 0 (∵ quantity cant be negative)



The feasible region is unbounded.


The corner points of the feasible region are as follows:



Z is smallest at C(2,4)


Let us consider 8x + 12y ≤ 64


As this has no intersection with the feasible region, the smallest value is the minimum value.


The minimum cost of the bags is ₹64



Question 9.

One kind of cake requires 300 gm of flour and 15 gm of fat, another kind of cake requires 150 gm of flour and 30 gm of fat. Find the maximum number of cakes which can be made from 7.5 kg of flour and 600 gm of fat, assuming that there is no shortage of the other ingredients used in making the cakes. Make it as an LPP and solve it graphically.


Answer:

The above information can be expressed in the form of the following table



Let ‘x’ and ‘y’ units of cake 1 and cake 2 be made.


Number of cakes made = x + y


Now,


300x + 150y ≤ 7500


i.e. the maximum availability of flour is 7500g for both cakes, each of which requires 300g and 150g of flour respectively


⟹ 15x + 30y ≥ 600


i.e. the maximum availability of fat is 600g for both the cakes, each of which requires 15g and 30g of fat.


Hence , mathematical formulation of the LPP is as follows :


Find ‘x’ and ‘y’ that,


Maximises Z = x + y


Subject to the following constraints:


(i) 300x + 150y ≤ 7500


i.e. 2x + y ≤ 50


(ii) 15x + 30y ≥ 600


i.e. x + 2y ≥ 40


(iii) x,y ≥0 (∵ quantity cant be negative)



The feasible region is bounded (ABO)


The corner points of the feasible region is as follows:



Z is maximised at B(20,10)


The maximum number of cakes that can be made are 20 and 10 of each kind i.e. 30 in total.



Question 10.

Reshma wishes to mix two types of food P and Q in such a way that the vitamin contents of the mixture contain at least 8 units of vitamin A and 11 units of vitamin B. Food P costs ₹ 60 kg and Food Q costs ₹ 80 kg. Food P contains 3 units / kg of Vitamin A and 5 units / kg of Vitamin B while food Q contains 4 units / kg of Vitamin A and 2 units / kg of vitamin B. Determine the minimum cost of the mixture.


Answer:

The above information can be expressed in the form of the following table:



Let the mixture contain ‘x’ kgs and ‘y’ kgs of food P and Q respectively.


Cost of food P = 60x


Cost of food Q = 80y


Cost of mixture = 60x + 80y


Now,


⟹ 3x + 4y ≥ 8


i.e. the minimum requirement of vitamin A from the mixture of P and Q is 8units, each of which contains 3units and 4units respectively.


⟹ 5x + 2y ≥ 11


i.e. the minimum requirement of vitamin B from the mixture of P and Q is 11 units, each of which contains 5units and 2units respectively.


Hence, mathematical formulation of the LPP is as follows:



The feasible region is Unbounded.


The corner points of the feasible region are as follows:



Z is minimised on the line joining points B(2,0.5) and .


The minimum cost of mixture is ₹160



Question 11.

One kind of cake requires 200 g of flour and 25 g of fat, and another kind of cake requires 100 g of flour and 50 g of fat. Find the maximum number of cakes which can be made from 5 kg of flour and 1 kg of fat assuming that there is no shortage of the other ingredients used in making the cakes.


Answer:

The above information can be expressed in the form of the following table:



Let the number of Cake 1 and Cake 2 be made be ‘x’ and ‘y’


Number of cakes made = x + y


Now,


⟹ 200x + 100y ≤ 5000


i.e. the maximum flour available for both the cakes combined is 5000g, each of which requires 200g and 100g of flour respectively.


⟹ 25x + 50y ≤ 1000


i.e. the maximum fat available for the two cakes combined is 1000g, each of which requires 25g and 50g of fat respectively.


Hence, the mathematical formulation of the LPP is as follows:


Find ‘x’ and ‘y’ that:


Maximises Z = x + y


Subject to the following constraints:


(i) 200x + 100y ≤ 5000


i.e. 2x + y ≤ 50


(ii) 25x + 50y ≤ 1000


i.e. x + 2y ≤ 40


(iii) x,y ≥ 0 (∵ quantity cant be negative)



The feasible region is bounded (OBAC)


The corner points of the feasible region is as follows:



Z is maximised at A(20,10)


The maximum number of cakes that can be made are 30.



Question 12.

A dietician has to develop a special diet using two foods P and Q. Each packet (containing 30 g) of food P contains 12 units of calcium, 4 units of iron, 6 units of cholesterol and 6 units of vitamin A. Each packet of the same quantity of food Q contains 3 units of calcium, 20 units of iron, 4 units of cholesterol and 3 units of vitamin A. The diet requires at least 240 units of calcium, at least 460 units of iron and at most 300 units of cholesterol. How many packets of each food should be used to minimize the amount of vitamin A in the diet? What is the minimum amount of vitamin A?


Answer:

The above information can be expressed with the help of the following table



Let the number of packets bought, of P and Q, be ‘x’ and ‘y’


Vitamin A from P = 6x


Vitamin A from Q = 3y


Vitamin A in the diet = 6x + 3y


Now,


⟹ 12x + 3y ≥ 240


i.e. the minimum requirement of Calcium in the diet, form both the foods combined, is 240units, each of which has 12units and 3units of calcium respectively.


⟹ 4x + 20y ≥ 460


i.e. the minimum requirement of Iron from P and Q combined is 460 units, each of which has 4units and 20units of iron respectively.


⟹ 6x + 4y ≤ 300


i.e. the maximum requirement of Cholesterol from P and Q combined is 300 units, each of which contains 6units and 4units of cholesterol respectively.


Hence, the mathematical formulation of the LPP is as follows:


Find ‘x’ and ‘y’ that:


Minimises Z = 6x + 3y


Subject to the following constraints:


(i) 12x + 3y ≥ 240


i.e. 4x + y ≥ 80


(ii) 4x + 20y ≥ 460


i.e. x + 5y ≥ 115


(iii) 6x + 4y ≤ 300


i.e. 3x + 2y ≤ 150


(iv) x,y ≥0 (∵ quantity cant be negative)



The feasible region is bounded (ABC)


The corner points of the feasible region are as follows:



Z is minimised at A(15,20) i.e. 15 packets of P and 20 packets of Q should be used to minimise the amount of vitamin A.


The minimum amount of vitamin A is 150 units.



Question 13.

A farmer mixes two brands P and Q of cattle feed. Brand P, costing ₹ 250 per bag, contains 3 units of nutritional element A, 2.5 units of element B and 2 units of element C. Brand Q costing ₹ 200 per bag contains 1.5 units of nutritional element A, 11.25 units of element B, and 3 units of element C. The minimum requirements of nutrients A, B and C are 18 units, 45 units and 24 units respectively. Determine the number of bags of each brand which should be mixed in order to produce a mixture having a minimum cost per bag? What is the minimum cost of the mixture per bag?


Answer:

The above information can be expressed with the help of the following table:



Let ‘x’ bags of P and ‘y’ bags of Q be bought.


Cost of P = 250x


Cost of Q = 200y


Cost of mixture = 250x + 200y


Now,


⟹ 3x + 1.5y ≥18


i.e. the minimum requirement of element A from both P and Q combined is 18units, each of which has 3units and 1.5units of element A.


⟹ 2.5x + 11.25y ≥ 45


i.e. the minimum value of element B from both P and Q combined is 45units, each of which contains 2.5units and 11,25units of element B.


⟹ 2x + 3y ≥ 24


i.e. the minimum value of element C from both P and Q combined is 24units, each of which contains 2units and 3 units of element C.


Hence, mathematical formulation of the above LPP is as follows :


Find ‘x’ and ‘y’ that:


Minimises Z = 250x + 200y


Subject to the following constraints:


(i) 3x + 1.5y ≥18


(ii) 2.5x + 11.25y ≥ 45


(iii) 2x + 3y ≥ 24


(iv) x,y ≥0 (∵ quantity cant be negative)



The feasible region is unbounded


The corner points of the feasible region are as follows:



Z is minimised at B(3,6) i.e. 3 bags of P and 6 bags of Q should be purchased to achieve the minimum cost of the mixture per bag.


The minimum cost of the mixture is ₹1950.



Question 14.

A dietician wishes to mix together two kinds of food X and Y in such a way that the mixture contains at least 10 units of vitamin A, 12 units of vitamin B and 8 units of vitamin C. The vitamin contents of one kg food is given below :



One kg of food X costs ₹16 and one kg of food Y costs ₹ 20. Find the least cost of the mixture which will produce the required diet?


Answer:

The above information can be expressed in the form of the following table:



Let the quantity of X and Y purchased be ‘x’ and ‘y’ kgs


Cost of X = 16x


Cost of Y = 20y


Cost of the mixture = 16x + 20y


Now,


⟹ x + 2y ≥ 10


i.e. the minimum requirement of Vitamin A from the mixture of X and Y is 10 units, each of which contains 1 unit and 2 units of Vitamin A respectively.


⟹ 2x + 2y ≥ 12


i.e. the minimum requirement of Vitamin B from the mixture of X and Y is 12 units, each of which contains 2units of vitamin B each.


⟹ 3x + y ≥ 8


i.e. the minimum requirement of vitamin C from the mixture of X and Y is 8 units, each of which contains 3 units and 1 unit of vitamin C respectively.


Hence, the mathematical formulation of the LPP is as follows :


Find ‘x’ and ‘y’ that:


Minimises Z = 16x + 20y


Subject to the following constraints:


(i) x + 2y ≥ 10


(ii) 2x + 2y ≥ 12


i.e. x + y ≥ 6


(iii) 3x + y ≥ 8


(iv) x,y ≥0 (∵ quantity cant be negative)



The feasible region is unbounded


The corner points of the feasible region is as follows:



Z is smallest at C(2,4)


Let us consider 16x + 20y ≤ 112


As it has no intersection with the feasible region, the smallest value is the minimum value.


The minimum cost of the mixture is ₹112.



Question 15.

A fruit grower can use two types of fertilizer in his garden, brand P and Q. The amounts (in kg) of nitrogen, phosphoric acid, potash, and chlorine in a bag of each brand are given in the table. Tests indicate that the garden needs at least 240 kg of phosphoric acid, at least 270 kg of potash and at most 310 kg of chlorine.



If the grower wants to minimize the amount of nitrogen added to the garden, how many bags of each brand should be used? What is the minimum amount of nitrogen added in the garden?


Answer:

The above information can be expressed with the help of the following table:



Let the number of bags of P and Q chosen be ‘x’ and ‘y’ units.


Nitrogen from P = 3x


Nitrogen from Q = 3.5y


Nitrogen form the mixture = 3x + 3.5y


Now,


⟹ x + 2y ≥ 240


i.e. the minimum requirement of phosphoric acid in the mixture of P and Q is 240 kgs, each of which contains 1kg and 2kgs of phosphoric acid respectively


⟹ 3x + 1.5y ≥ 270


i.e. the minimum requirement of Potash in the mixture of P and Q is 270 kgs, each of which contains 3kgs and 1.5kgs of Potash respectively.


⟹ 1.5x + 2y ≤ 310


i.e. the maximum requirement of Chlorine in the mixture of P and Q is 310 kgs, each of which contains 1.5kgs and 2kgs of Chlorine respectively.


Hence, mathematical formulation of the LPP is as follows:


Find ‘x’ and ‘y’ that


Minimises Z = 3x + 3.5y


Subject to the following constraints:


(i) x + 2y ≥ 240


(ii) 3x + 1.5y ≥ 270


(iii) 1.5x + 2y ≤ 310


(iv) x,y ≥0 (∵ quantity cant be negative)



The feasible region is bounded (ABC)


The corner points of the feasible region is as follows:



Z is minimised at B(40,100)


The minimum amount of Nitrogen in the mixture is 470kgs




Exercise 30.4
Question 1.

If a young man drives his scooter at a speed of 25 km/hr, he has to spend Rs2 per km on petrol. If he drives the scooter at a speed of 40 km/hour, it produces air pollution and increases his expenditure on petrol to Rs 5 per km. He has a maximum of Rs100 to spend on petrol and travel a maximum distance in one hour time with less pollution. Express this problem as an LPP and solve it graphically. What value do you find here?


Answer:

Let young man drives x km at a speed of 25 km/hr and y km at a speed of 40 km/hr. Clearly,


x, y 0


It is given that, he spends Rs 2 per km if he drives at a speed of 25 km/hr and Rs 5 per km if he drives at a speed of 40 km/hr. Therefore, money spent by him when he travelled x km and y km are Rs 2x and Rs 5y respectively.


It is given that he has a maximum of Rs 100 to spend.


Thus, 2x + 5y 100


Time spent by him when travelling with a speed of 25 km/hr = hr


Time spent by him when travelling with a speed of 40km/hr = hr


Also, the available time is 1 hour.



Or, 40x + 25y1000


The distance covered is Z = x + y which is to be maximized.


Thus, the mathematical formulation of the given linear programming problem is Max Z = x + y subject to


2x + 5y 100


40x + 25y1000


x, y 0


First we will convert inequations as follows:


2x + 5y = 100


40x + 25y = 1000


x = 0 and y = 0.


The region represented by 2x + 5y 100


The line 2x + 5y = 100 meets the coordinate axes at A(50,0) and B(0,20) respectively. By joining these points, we obtain the line 2x + 5y = 100. Clearly (0, 0) satisfies the 2x + 5y = 100. So, the region which contains the origin represents the solution set of the inequation 2x + 5y 100


The region represented by 40x + 25y 1000


The line 40x + 25y = 1000 meets the coordinate axes at C(25,0) and D(0,40) respectively. By joining these points, we obtain the line 2x + y = 12. Clearly (0, 0) satisfies the 40x + 25y = 1000. So, the region which contains the origin represents the solution set of the inequation 40x + 25y 1000


The region represented by x 0, y 0 :


Since every point in the first quadrant satisfies these inequations. So, the first quadrant is the region represented by the inequations x 0 and y 0.


The feasible region determined by the system of constraints


2x + 5y 100, 40x + 25y1000, x 0 and y 0 are as follows



The corner points are O(0,0), B(0,20), E, and C(25,0). The value of Z at these corner points are as follows:



The maximum value of Z is 30 which is attained at E.


Thus, the maximum distance travelled by the young man is 30 kms, if he drives km at a speed of 25 km/hr and km at a speed of 40 km/hr.



Question 2.

A manufacturer has three machines installed in his factory. Machines I and II are capable of being operated for at most 12 hours whereas Machine III must operate at least for 5 hours a day. He produces only two items, each requiring the use of three machines. The number of hours required for producing one unit each of the items on the three machines is given in the following table :



He makes a profit of Rs 6.00 on item A and Rs 4.00 on item B. Assuming that he can sell all that he produces, how many of each item should he produce to maximize his profit? Determine his maximum profit. Formulate this LPP mathematically and then solve it.


Answer:

Let x units of item A and y units of item B be manufactured. Therefore, x, y 0.


As we are given,



Machines I and II are capable of being operated for at most 12 hours whereas Machine III must operate at least for 5 hours a day.


According to the question, the constraints are


x + 2y 12


2x + y 12


x + y 5


He makes a profit of Rs 6.00 on item A and Rs. 4.00 on item B. Profit made by him in producing x items of A and y items of B is 6x + 4y.


Total profit Z = 6x + 4y which is to be maximized


Thus, the mathematical formulation of the given linear programming problem is


Max Z = 6x + 4y, subject to


x + 2y 12


2x + y 12


x + y 5


x, y 0


First, we will convert the inequations into equations as follows:


x + 2y = 12, 2x + y = 12, x + y = 5, x = 0 and y = 0.


The region represented by x + 2y 12


The line x + 2y = 12 meets the coordinate axes at A(12,0) and B(0,6) respectively. By joining these points, we obtain the line x + y = 12. Clearly (0, 0) satisfies the x + 2y = 12. So, the region which contains the origin represents the solution set of the inequation x + 2y 12


The region represented by 2x + y 12


The line 2x + y = 12 meets the coordinate axes at C(6,0) and D(0,12) respectively. By joining these points, we obtain the line 2x + y = 12. Clearly (0, 0) satisfies the 2x + y = 12. So, the region which contains the origin represents the solution set of the inequation 2x + y 12


The region represented by x + y 5


The line x + y 5 meets the coordinate axes at E(5,0) and F(0,4) respectively. By joining these points, we obtain the line x + y = 5. Clearly (0, 0) satisfies the x + y 5. So, the region which does not contain the origin represents the solution set of the inequation x + y 5


The region represented by x 0, y 0 :


Since every point in the first quadrant satisfies these inequations. So, the first quadrant is the region represented by the inequations x 0 and y 0.


The feasible region determined by the system of constraints


x + 2y 12, 2x + y 12, x + y 5, x, y 0 are as follows.



Thus the maximum profit is of Rs 40 obtained when 4 units each of item A and B are manufactured.


The corner points are D(0,6), I(4,4), C(6,0), G(5,0), and H(0,4). The values of Z at these corner points are as follows:



The maximum value of Z is 40 which is attained at I(4, 4).



Question 3.

Two tailors, A and B earn ₹ 15 and ₹ 20 per day respectively. A can stitch 6 shirts and 4 pants while B can stitch 10 shirts and 4 pants per day. How many days shall each work if it is desired to produce (at least) 60 shirts and 32 pants at a minimum labour cost?


Answer:

Let tailor A work for x days and tailor B work for y days.


In one day, A can stitch 6 shirts and 4 pants whereas B can stitch 10 shirts and 4 pants.


Thus in x days, A can stitch 6x shirts and 4x pants whereas in y days B can stitch 10y shirts and 4y pants.


It is given that the minimum requirement of the shirt and pants are respectively 60 and 32.


Thus,


6x + 10y 60


4x + 4y 32


Further it is given that A and B earn Rs 15 and Rs 20 per day respectively. Thus, A earn Rs 15x and B earns Rs 20y.


Let Z denotes the total cost


Z = 15x + 20y


Days cannot be negative.


x,y 0.


MIN Z = 15x + 20y


Subject to


6x + 10y 60


4x + 4y 32


x,y 0


First we will convert inequations into equations as follows:


6x + 10y = 60, 4x + 4y = 32, x = 0, y = 0


Region represented by 6x + 10y 60


The line 6x + 10y = 60 meets the coordinate axes at A(10,0) and B(0,6) respectively. By joining these points we obtain the line 6x + 10y = 60. Clearly (0, 0) satisfies the 6x + 10y 60. So, the region which does not contains the origin represents the solution set of the inequation 6x + 10y 60


Region represented by 4x + 4y 32


The line 4x + 4y = 32 meets the coordinate axes at C(8,0) and D(0,8) respectively. By joining these points we obtain the line 4x + 4y = 32. Clearly (0, 0) satisfies the 4x + 4y 32. So, the region which does not contains the origin represents the solution set of the inequation 4x + 4y 32.


Region represented by x 0, y 0 :


Since, every point in the first quadrant satisfies these inequations. So, the first quadrant is the region represented by the inequations x 0 and y 0.


The feasible region determined by the system of constraints 6x + 10y 60, 4x + 4y 32


x,y 0 are as follows.



The corner points are D(0,8), E(5,3), A(10,00. The values of Z at these corner points are as follows:



The minimum value of Z is 135 which is attained at E(5,3).


Thus, for minimum labour cost, A should work for 5 days and B should work for 3 days.



Question 4.

A factory manufactures two types of screws, A and B, each type requiring the use of two machines - an automatic and a hand - operated. It takes 4 minute on the automatic and 6 minutes on the hand - operated machines to manufacture a package of screws ‘A’, while it takes 6 minutes on the automatic and 3 minutes on the hand - operated machine to manufacture a package of screws ‘B’. Each machine is available for at most 4 hours on any day. The manufacturer can sell a package of screws ‘A’ at a profit of 70 P and screws ‘B’ at a profit of ₹ 1. Assuming that he can sell all the screws he can manufacture, how many packages of each type should the factory owner produce in a day in order to maximize his profit? Determine the maximum profit.


Answer:

Let the factory manufacture x screws of type A and y screws of type B on each day,


Therefore, x 0 and y 0.


The given information can be compiled in a table as follows



4x + 6y 240


6x + 3y 240


The manufacturer can sell a package of screws ‘A’ at a profit of Rs 0.7 and screws ‘B’ at a profit of Re 1.


Total profit, Z = 0.7x + 1y


The mathematical formulation of the given problem is


Maximize Z = 0.7x + 1y


subject to the constraints,


4x + 6y 240


6x + 3y 240


x, y 0


First we will convert the inequations into equations as follows:


4x + 6y = 240, 6x + 3y = 240, x = 0, y = 0.


Region represented by 4x + 6y 240


The line 4x + 6y = 240 meets the coordinate axes at A(60,0) and B(0,40) respectively. By joining these points we obtain the line 4x + 6y = 240. Clearly (0, 0) satisfies the 4x + 6y 240. So, the region which contains the origin represents the solution set of the inequation 4x + 6y 240.


Region represented by 6x + 3y 240


The line 6x + 3y = 240 meets the coordinate axes at C(40,0) and d(0,80) respectively. By joining these points we obtain the line 6x + 3y = 240. Clearly (0, 0) satisfies the 6x + 3y 240. So, the region which contains the origin represents the solution set of the inequation 6x + 3y 240.


Region represented by x 0, y 0 :


Since, every point in the first quadrant satisfies these inequations. So, the first quadrant is the region represented by the inequations x 0 and y 0.


The feasible region determined by the system of constraints 4x + 6y 240, 6x + 3y 240, x 0,


y 0 are as follows.



The corner points are C(40,0), E(30,20), B(0,40). The values of Z at these corner points are as follows



The maximum value of Z is 410 at (30, 20).


Thus, the factory should produce 30 packages of screws A and 20 packages of screws b to get the maximum profit of Rs 410.



Question 5.

A company produces two types of leather belts, say type A and B. Belt A is a superior quality and belt B is of a lower quality. Profits on each type of belt are 2 and 1.50 per belt, respectively. Each belt of type A requires twice as much time as required by a belt of type B. If all belts were of type B, the company could produce 1000 belts per day. But the supply of leather is sufficient only for 800 belts per day (both A and B combined). Belt A requires a fancy buckle and only 400 fancy buckles are available for this per day. For belt of type B, only 700 buckles are available per day.

How should the company manufacture the two types of belts in order to have a maximum overall profit?


Answer:

Let the company produces x belts of types A and y belts of type B. Number of belts cannot be negative. Therefore, x,y 0.


It is given that leather is sufficient only for 800 belts per day (both A and B combined).


Therefore,


x + y 800


It is given that the rate of production of belts of type B is 1000 per day. Hence the time taken to produce y belts of type B is .


And, since each belt of type A requires twice as much time as a belt of type B, the rate of production of belts of type A is 500 per day and therefore, total time taken to produce x belts of type A is


Thus, we have,



Or, 2x + y 1000


Belt A requires fancy buckle and only 400 fancy buckles are available for this per day.


x 400


For Belt of type B only 700 buckles are available per day.


y 700


profits on each type of belt are Rs 2 and Rs 1.50 per belt, respectively. Therefore, profit gained on x belts of type A and y belts of type B is Rs 2x and Rs 1.50y respectively. Hence, the total profit would be Rs(2x + 1.50y). Let Z denote the total profit


Z = 2x + 1.50y


Thus, the mathematical formulation of the given linear programming problem is;


Max Z = 2x + 1.50y subject to


x + y 800


2x + y 1000


x 400


y 700


First we will convert these inequations into equations as follows:


x + y = 800


2x + y = 1000


x = 400


y = 700


Region represented by x + y = 800


The line x + y = 800 meets the coordinate axes at A(800,0) and B(0,800) respectively. By joining these points we obtain the line x + y = 800. Clearly (0, 0) satisfies the x + y 800. So, the region which contains the origin represents the solution set of the inequation x + y 800.


Region represented by 2x + y 1000


The line 2x + y = 1000 meets the coordinate axes at C(500,0) and D(0,1000) respectively. By joining these points we obtain the line 2x + y = 1000. Clearly (0, 0) satisfies the 2x + y 1000. So, the region which contains the origin represents the solution set of the inequation 2x + y 1000.


Region represented by x 400


The line x = 400 will pass through (400,0). The region to the left of the line x = 400 will satisfy the inequation x 400


Region represented by y 700


The line y = 700 will pass through (0,700). The region to the left of the line y = 700


will satisfy the inequation y 700.


Region represented by x 0, y 0 :


Since, every point in the first quadrant satisfies these inequations. So, the first quadrant is the region represented by the inequations x 0 and y 0.


The feasible region determined by the system of constraints x + y 800, 2x + y 1000, x 400,


y 700



The corner points are F(0,700), G(200,600), H(400,200), E(400,0). The values of Z at these corner points are as follows



The maximum value of Z is 1300 which is attained at G(200,600).


Thus, the maximum profit obtained is Rs 1300 when 200 belts of type A and 600 belts of type B are produced.



Question 6.

A small manufacturer has employed 5 skilled men and 10 semi - skilled men and makes an article in two qualities deluxe model and an ordinary model. The making of a deluxe model requires 2 hrs. work by a skilled man and 2 hrs. work by a semi - skilled man. The ordinary model requires 1 hr by a skilled man and 3 hrs. by a semi - skilled man By union rules no man may work more than 8 hrs per day. The manufacturers clear profit on deluxe model is Rs 15 and on an ordinary model is Rs 10. How many of each type should be made in order to maximize his total daily profit.


Answer:

Let x articles of deluxe model and y articles of an ordinary model be made.


Numbers cannot be negative.


Therefore,


x, y 0


According to the question, the profit on each model of deluxe and ordinary type model are Rs 15 and Rs 10 respectively.


So, profits on x deluxe model and y ordinary models are 15x and 10y.


Let Z be total profit, then,


Z = 15x + 10y


Since, the making of a deluxe and ordinary model requires 2 hrs. and 1 hr work by skilled men, so, x deluxe and y ordinary models require 2x and y hours of skilled men but time available by skilled men is 58 = 40 hours.


So,


2x + y 40 { First Constraint}


Since, the making of a deluxe and ordinary model requires 2 hrs. and 3 hrs work by semi skilled men, so, x deluxe and y ordinary models require 2x and 3y hours of skilled men but time available by skilled men is 108 = 80 hours.


So,


2x + 3y 80 {Second constraint}


Hence the mathematical formulation of LPP is,


Max Z = 15x + 10y


subject to constraints,


2x + y 40


2x + 3y 80


x, y 0


Region 2x + y 40: line 2x + 4y = 40 meets axes at (20,0), (0,40) respectively. Region containing origin represents 2x + 3y 40 as (0,0) satisfies 2x + y 40


Region 2x + 3y 80: line 2x + 3y = 80 meets axes at (40,0), (0,) respectively. Region containing origin represents 2x + 3y 80.



The corner points are (20,0), P(10,20), (0,).


The value of Z = 15x + 10y at these corner points are



The maximum value of Z is 300 which is attained at P(10,20).


Thus, maximum profit is obtained when 10 units of deluxe model and 20 units of ordinary model is produced.



Question 7.

A manufacturer makes two types A and B of tea - cups. Three machines are needed for the manufacture and the time in minutes required for each cup on the machines is given below :



Each machine is available for a maximum of 6 hours per day. If the profit on each cup A is 75 paise and that on each cup B is 50 paise, show that 15 tea - cups of type A and 30 of type B should be manufactured in a day to get the maximum profit.


Answer:

Let the required number of tea cups of Type A and B are x and y respectively.


Since, the profit on each cup A is 75 paise and that on each cup B is 50 paise. So, the profit on x tea cup of type A and y tea cup of type B are 75x and 50y respectively.


Let total profit on tea cups be Z, so


Z = 75x + 50y


Since, each tea cup of type A and B require to work machine I for 12 and 6 minutes respectively so, x tea cups of Type A and y tea cups of Type B require to work on machine I for 12x and 6y minutes respectively.


Total time available on machine I is 660 = 360 minutes. So,


12x + 6y 360 {First Constraint}


Since, each tea cup of type A and B require to work machine II for 18 and 0 minutes respectively so, x tea cups of Type A and y tea cups of Type B require to work on machine IIII for 18x and 0y minutes respectively.


Total time available on machine I is 660 = 360 minutes. So,


18x + 0y 360


x 20 {Second Constraint}


Since, each tea cup of type A and B require to work machine III for 6 and 9 minutes respectively so, x tea cups of Type A and y tea cups of Type B require to work on machine I for 6x and 9y minutes respectively.


Total time available on machine I is 660 = 360 minutes. So,


6x + 9y 360 {Third Constraint}


Hence mathematical formulation of LPP is,


Max Z = 75x + 50y


subject to constraints,


12x + 6y 360


x 20


6x + 9y 360


x,y 0 [Since production of tea cups can not be less than zero]


Region 12x + 6y 360: line 12x + 6y = 360 meets axes at A(30,0), B(0,60) respectively. Region containing origin represents 12x + 6y 360 as (0,0) satisfies 12x + 6y 360


Region x 20: line x = 20 is parallel to y axis and meets x - axes at C(20,0). Region containing origin represents x 20


as (0,0) satisfies x 20.


Region 6x + 9y 360: line 6x + 9y = 360 meets axes at E(60,0), F(0,40) respectively. Region containing origin represents 6x + 9y 360 as (0,0) satisfies 6x + 9y 360.


Region x,y 0: it represents the first quadrant.



The shaded region is the feasible region determined by the constraints,


12x + 6y 360


x 20


6x + 9y 360


x,y 0


The corner points are F(0,40), G(15,30), H(20,20), C(20,0).


The values of Z at these corner points are as follows



Here Z is maximum at G(15,30).


Therefore, 15 teacups of Type A and 30 tea cups of Type B are needed to maximize the profit.



Question 8.

A factory owner purchases two types of machines, A and B, for his factory. The requirements and limitations for the machines are as follows :



He has an area of 7600 sq.m available and 72 skilled men who can operate the machines. How many machines of each type should he buy to maximize the daily output?


Answer:

Let required number of machine A and B are x and y respectively.


Since, products of each machine A and B are 60 and 40 units daily respectively. So, production by by x number of machine A and y number of machine B are 60x and 40y respectively.


Let Z denotes total output daily, so,


Z = 60x + 40y


Since, each machine of type A and B requires 1000sq. m and 1200 s. m area so, x machine of type A and y machine of type B require 1000x and 1200y sq. m area but,


Total available area for machine is 7600 sq. m. So,


1000x + 1200y 7600


or, 5x + 6y 38. {First Constraint}


Since each machine of type A and B requires 12 men and 8 men to work respectively. So, x machine of type A and y machine of type B require 12x and 8y men to work respectively.


But total men available for work are 72.


So,


12x + 8y 72


3x + 2y 18 {Second Constraint}


Hence mathematical formulation of the given LPP is,


Max Z = 50x + 40y


Subject to constraints,


5x + 6y 38


3x + 2y 18


x,y 0 [Since number of machines can not be less than zero]


Region 5x + 6y 38: line 5x + 6y = 38 meets the axes at A(,0), B(0,) respectively.


Region containing the origin represents 5x + 6y 38 as origin satisfies 5x + 6y 38


Region 3x + 2y 18: line 3x + 2y = 18 meets the axes at C(6,0), D(0,9) respectively.


Region containing the origin represents 3x + 2y 18 as origin satisfies 3x + 2y 18.


Region x,y 0: it represents the first quadrant.



Shaded region represents the feasible region.


The corner points are O(0,0), B(0,), E(4,3), C(6,0).


Thus the values of Z at these corner points are as follows:



The maximum value of Z is 360 which is attained at E(4,3), C(6,0).


Thus,the maximum output is Rs 360 obtained when 4 units of type A and 3 units of type B or 6 units of type A and 0 units of type B are manufactured.



Question 9.

A company produces two types of goods, A and B, that require gold and silver. Each unit of type A requires 3 gm of silver and 1 gm of gold while that of type B requires 1 gm of silver and 2 gm of gold. The company can produce 9 gm of silver and 8 gm of gold. If each unit of type A brings a profit of ₹ 40 and that of type B ₹ 50, find the number of units of each type that the company should produce to maximize the profit. What is the maximum profit?


Answer:

Let required number of goods A and B are x and y respectively.


Since, profits of each A and B are Rs. 40 and Rs. 50 respectively. So, profits on x number of type A and y number of type B are 40x and 50y respectively.


Let Z denotes total output daily, so,


Z = 40x + 50y


Since, each A and B requires 3 grams and 1 gram of silver respectively. So, x of type A and y of type B require 3x and y of silver respectively. But,


Total silver available is 9 grams. So,


3x + y 9 {First Constraint}


Since each A and B requires 1 gram and 2 grams of gold respectively. So, x of type A and y of type B require x and 2y respectively.


But total gold available is 8 grams.


So,


x + 2y 8 {Second Constraint}


Hence mathematical formulation of the given LPP is,


Max Z = 40x + 50y


Subject to constraints,


3x + y 9


x + 2y 8


x,y 0 [Since production of A and B can not be less than zero]


Region 3x + y 9: line 3x + y = 9 meets the axes at A(3,0), B(0,9) respectively.


Region containing the origin represents 3x + y 9


as origin satisfies 3x + y 9.


Region x + 2y 8: line x + 2y = 8 meets the axes at C(8,0), D(0,4) respectively.


Region containing the origin represents x + 2y 8 as origin satisfies x + 2y 8.


Region x,y 0: it represents the first quadrant.



The corner points are O(0,0), D(0,4), E(2,3), A(3,0)


The values of Z at these corner points are as follows



The maximum value of Z is 230 which is attained at E(2,3).


Thus the maximum profit is of Rs 230 when 2 units of Type A 3 uniits of Type B are produced.



Question 10.

A manufacturer of Furniture makes two products : chairs and tables. Processing of these products is done on two machines A and B. A chair requires 2 hrs on machine A and 6 hrs on machine B. A table requires 4 hrs on machine A and 2 hrs on machine B . There are 16 hrs of time per day available on machine A and 30 hrs on machine B. profit gained by the manufacturer from a chair and a table is ₹ 3 and ₹ 5 respectively. Find with the help of graph what should be the daily production of each of the two products so as to maximize his profit.


Answer:

Let daily production of chairs and tables be x and y respectively.


Since, profits of each chair and table is Rs. 3 and Rs. 5 respectively. So, profits on x number of type A and y number of type B are 3x and 5y respectively.


Let Z denotes total output daily, so,


Z = 3x + 5y


Since, each chair and table requires 2 hrs and 3 hrs on machine A respectively. So, x number of chair and y number of table require 2x and 4y hrs on machine A respectively. But,


Total time available on Machine A is 16 hours. So,


2x + 3y 16


x + 2y 8 {First Constraint}


Since, each chair and table requires 6 hrs and 2 hrs on machine B respectively. So, x number of chair and y number of table require 6x and 2y hrs on machine B respectively. But,


Total time available on Machine B is 30 hours. So,


6x + 2y 30


3x + y 15 {Second Constraint}


Hence mathematical formulation of the given LPP is,


Max Z = 3x + 5y


Subject to constraints,


x + 2y 8


3x + y 15


x,y 0 [Since production of chairs and tables can not be less than zero]


Region x + 2y 8: line x + 2y = 8 meets the axes at A(8,0), B(0,4) respectively.


Region containing the origin represents x + 2y 8


as origin satisfies x + 2y 8.


Region 3x + y 15: line 3x + y = 15 meets the axes at C(5,0), D(0,15) respectively.


Region containing the origin represents 3x + y 15 as origin satisfies 3x + y 15


Region x,y 0: it represents the first quadrant.



The corner points are O(0,0), B(0,4), E(), and C(5,0).


The values of Z at these corner points are as follows,



The maximum value of Z is 22.2 which is attained at E().


Thus the maximum profit of Rs 22.2 when units of chair and units of table are produced.



Question 11.

A furniture manufacturing company plans to make two products : chairs and tables. From its available resources which consists of 400 square feet of teak wood and 450 man hours. It is known that to make a chair requires 5 square feet of wood and 10 man - hours and yields a profit of ₹ 45, while each table uses 20 square feet of wood and 25 man - hours and yields a profit of ₹ 80. How many items of each product should be produced by the company so that the profit is maximum?


Answer:

Let required production of chairs and tables be x and y respectively.


Since, profits of each chair and table is Rs. 45 and Rs. 80 respectively. So, profits on x number of type A and y number of type B are 45x and 80y respectively.


Let Z denotes total output daily, so,


Z = 45x + 80y


Since, each chair and table requires 5 sq. ft and 80 sq. ft of wood respectively. So, x number of chair and y number of table require 5x and 80y sq. ft of wood respectively. But,


But 400 sq. ft of wood is available. So,


5x + 80y 400


x + 4y 80 {First Constraint}


Since, each chair and table requires 10 and 25 men - hours respectively. So, x number of chair and y number of table require 10x and 25y men - hours respectively. But, only 450 hours are available . So,


10x + 25y 450


2x + 5y 90 {Second Constraint}


Hence mathematical formulation of the given LPP is,


Max Z = 45x + 80y


Subject to constraints,


x + 4y 80


2x + 5y 90


x,y 0 [Since production of chairs and tables can not be less than zero]


Region x + 4y 80: line x + 4y = 80 meets the axes at A(80,0), B(0,20) respectively.


Region containing the origin represents x + 4y 80 as origin satisfies x + 4y 80


Region 2x + 5y 90: line 2x + 5y = 90 meets the axes at C(45,0), D(0,20) respectively.


Region containing the origin represents 2x + 5y 90


as origin satisfies 2x + 5y 90


Region x,y 0: it represents the first quadrant.



The corner points are O(0,0), D(0,18), C(45,0).


The values of Z at these corner points are as follows:



The maximum value of Z is 2025 which is attained at C(45,0).


Thus maximum profit of Rs 2025 is obtained when 45 units of chairs and no units of tables are produced.



Question 12.

A firm manufactures two products A and B. Each product is processed on two machines M1 and M2. Product A requires 4 minutes of processing time on M1 and 8 min. on M2; product B requires 4 minutes on M1 and 4 min. on M2. The machine M1 is available for not more than 8 hrs 20 min. while machine M2 is available for 10 hrs. during any working day. The products A and B are sold at a profit of ₹ 3 and ₹ 4 respectively.Formulate the problem as a linear programming problem and find how many products of each type should be produced by the firm each day in order to get maximum profit.


Answer:

Let required production of product A and B be x and y respectively.


Since profit on each product A and B are Rs. 3 and Rs. 4 respectively. So, profits on x number of type A and y number of type B are 3x and 4y respectively.


Let Z denotes total output daily, so,


Z = 3x + 4y


Since, each A and B requires 4 minutes each on machine . So, x of type A and y of type B require 4x and 4y minutes respectively. But,


Total time available on machine is 8 hours 20 minutes = 500 minutes.


So,


4x + 4y 500


x + y 125 {First Constraint}


Since, each A and B requires 8 minutes and 4 minutes on machine respectively. So, x of type A and y of type B require 8x and 4y minutes respectively. But,


Total time available on machine is 10 hours = 600 minutes.


So,


8x + 4y 600


2x + y 150 {Second Constraint}


Hence mathematical formulation of the given LPP is,


Max Z = 3x + 4y


Subject to constraints,


x + y 125


2x + y 150


x,y 0 [Since production of A and B can not be less than zero]


Region x + y 125: line x + y = 125 meets the axes at A(125,0), B(0,125) respectively.


Region containing the origin represents x + y 125 as origin satisfies x + y 125.


Region 2x + y 150: line 2x + y = 150 meets the axes at C(75,0), D(0,150) respectively.


Region containing the origin represents 2x + y 150 as origin satisfies 2x + y 150.


Region x,y 0: it represents the first quadrant.



The corner points are O(0,0), B(0,125), E(25,100), and C(75,0).


The vaues of Z at these corner points are as follows:



The maximum value of Z is 500 which is attained at B(0,125).


Thus, the maximum profit is Rs 500 obtained when no units of product A and 125 units of product B are manufactured.



Question 13.

A firm manufacturing two type of electric items, A and B, can make a profit of 20 per unit of A and ₹ 30 per unit of B. Each unit of A requires 3 motors and 4 transformers and each unit of B requires 2 motors and 4 transformers. The total supply of these per month is restricted to 210 motors and 300 transformers. Type B is an export model requiring a voltage stabilizer which has a supply restricted to 65 units per month. Formulate the linear programming problem for maximum profit and solve it graphically.


Answer:

Let x units of item A and y units of item B were manufactured.


Numbers of items cannot be negative. Therefore,


x, y 0


The given information can be tabulated as follows:



Further, it is given that type B is an export model, whose supply is restricted to 65 units per month.


Therefore, the constraints are


3x + 2y 210


4x + 4y 300


y 65


A and B can make profit of Rs 20 and Rs 30 per unit respectively.


Therefore, profit gained from x units of item A and y units of item B is Rs 20x and 30y respectively.


Total Profit = Z = 20x + 30y which according to question is to be maximised.


Thus the mathematical formulation of the given LPP is,


Max Z = 20x + 30y


Subject to constraints


3x + 2y 210


4x + 4y 300


y 65


x, y 0


Region represented by 3x + 2y 210: The line 3x + 2y = 210 meets the axes at A(70,0), B(0,105) respectively.


Region containing the origin represents 3x + 2y 210 as origin satisfies 3x + 2y 210.


Region represented by 4x + 4y 300: The line 4x + 4y = 300 meets the axes at C(75,0), D(0,75) respectively.


Region containing the origin represents 4x + 4y 300 as origin satisfies 4x + 4y 300


y = 65 is the line passing through the point E(0,65) and is parallel to X - axis.


Region x,y 0: it represents the first quadrant.



The corner points are O(0,0), E(0,65), G(10,65), F(60,15) and A(70,0).


The values of Z at these corner points are as follows:



The maximum value of Z is 2150 which is attained at G(10,65).


Thus, the maximum profit is Rs. 2150 obtained when 10 units of item A and 65 units of item B are manufactured.



Question 14.

A factory uses three different resources for the manufacture of two different products,20 units of the resources a, 12 units of B and 16 units of C being available 1 unit of the first product requires 2,2 and 4 units of the respective resources and 1 unit of the second product requires 4,2 and 0 units of respective resources. It is known that the first product gives a profit of 2 monetary units per unit and the second 3. Formulate the linear programming problem. How many units of each product should be manufactured for maximizing the profit? Solve it graphically.


Answer:

Let number of product I and product II are x and y respectively.


Since, profits on each product I and II are 2 and 3 monetary unit. So, profits on x number of Product I and y number of Product II are 2x and 3y respectively.


Let Z denotes total output daily, so,


Z = 2x + 3y


Since, each I and II requires 2 and 4 units of resources A. So, x units of product I and y units of product II requires 2x and 4y minutes respectively. But, maximum available quantity of resources A is 20 units.


So,


2x + 4y 20


x + 2y 10 {First Constraint}


Since, each I and II requires 2 and 2 units of resources B. So, x units of product I and y units of product II requires 2x and 2y minutes respectively. But, maximum available quantity of resources A is 12 units.


So,


2x + 2y 12


x + y 6 {Second Constraint}


Since, each units of product I requires 4 units of resources C. It is not required by product II. So, x units of product I require 4x units of resource C. But, maximum available quantity of resources C is 16 units.


So,


4x 16


x 4 {Third Constraint}


Hence mathematical formulation of LPP is,


Max Z = 2x + 3y


Subject to constraints,


x + 2y 10


x + y 6


x 4


x, y 0 [ Since production for I and II can not be less than zero]


Region represented by x + 2y 10: The line x + 2y = 10 meets the axes at A(10,0), B(0,5) respectively.


Region containing the origin represents x + 2y 10 as origin satisfies x + 2y 10.


Region represented by x + y 6: The line x + y = 6 meets the axes at C(6,0), D(0,6) respectively. Region containing the origin represents x + y 6 as origin satisfies x + y 6


Region x,y 0: it represents the first quadrant.



The corner points are O(0,0), B(0,5), G(2,4), F(4,2), and E(4,0).


The values of Z at these corner points are as follows:



The maximum value of Z is 16 which is attained at G (12,4).


Thus, the maximum profit is 16 monetary units obtained when 2 units of first product and 4 units of second product were manufactured.



Question 15.

A publisher sells a hard cover edition of a text book for ₹ 72.00 and a paperback edition of the same ext for ₹ 40.00. Costs to the publisher are ₹ 56.00 and ₹ 28.00 per book respectively in addition to weekly costs of ₹ 9600.00. Both types require 5 minutes of printing time, although hardcover requires 10 minutes binding time and the paperback requires only 2 minutes. Both the printing and binding operations have 4,800 minutes available each week. How many of each

type of book should be produced in order to maximize profit?


Answer:

Let the sale of hand cover edition be ‘h’ and that of paperback editions be ‘t’.


SP of a hard cover edition of the textbook = Rs 72


SP of a paperback edition of the textbook = Rs 40


Cost to the publisher for hard cover edition = Rs 56


Cost to the publisher for a paperback edition = Rs 28


Weekly cost to the publisher = Rs 9600


Profit to be maximized, Z = (72 - 56)h + (40 - 28)t – 9600


Z = 16h + 12t - 9600


5(h + t) 4800


10h + 2t 4800.



The corner Points are O(0,0), (0,960), (360,600) and (480,00).


The values of Z at these corner points are as follows:



The maximum value of Z is 3360 which is attained at (360,600).


The maximum profit is 3360 which is obtained by selling 360 copies of hardcover edition and 600 copies paperback edition.



Question 16.

A firm manufactures headache pills in two sizes A and b. Size A contains 2 grains of aspirin, 5 grains of bicarbonate and 1 grain of codeine ; size B contains 1 grain of aspirin, 8 grains of bicarbonate and 66 grains of codeine. It has been found by users that it requires at least 12 grains of aspirin, 7.4 grains of bicarbonate and 24 grains of codeine for providing immediate effects. Determine graphically the least number of pills a patient should have to get immediate relief. Determine also the quantity of codeine consumed by patient.


Answer:

The above LPP can be presented in a table below,



Hence mathematical formulation of LPP is,


Max Z = x + y


Subject to constraints,


2x + y 12


5x + 8y 7.4


x + 66y 24


x, y 0 [ Since production can not be less than zero]



The corner points are B(0,12), P(5.86, 0.27), E(24,0)


The values of Z at these corner points are as follows:



The minimum value of Z is 6.13 but the region is unbounded so check whether x + y 6.13


Clearly, it can be seen that it does not has any common region.


So, x = 5.86, y = 0.27


This is the least quantity of pill A and B.


Codeine quantity = x + 66y = 5.86 + (660.27) = 24(approx).



Question 17.

A chemical company produces two compounds, A and B. The following table gives the units of ingredients, C and D per kg of compounds A and B as well as minimum requirements of C and D and costs per kg of A and B. Find the quantities of A and B which would give a supply of C and D at a minimum cost.



Answer:

Let required quantity of compound A and B are x and y kg.


Since, cost of one kg of compound A and B are Rs 4 and Rs 6 per kg. So,


Cost of x kg of compound A and y kg of compound B are Rs 4x and Rs 6


respectively.


Let Z be the total cost of compounds, so,


Z = 4x + 6y


Since, compound A and B contain 1 and 2 units of ingredient C per kg


respectively, So x kg of compound A and y kg of compound B contain x and 2y


units of ingredient C respectively but minimum requirement of ingredient C is 80


units, so,


x + 2y 80 {first constraint}


Since, compound A and B contain 3 and 1 units of ingredient D per kg


respectively, So x kg of compound A and y kg of compound B contain 3x and y


units of ingredient D respectively but minimum requirement of ingredient C is 75


units, so,


3x + y 75 {second constraint}


Hence, mathematical formulation of LPP is,


Min Z = 4x + 6y


Subject to constraints,


x + 2y 80


3x + y 75


x, y 0 [Since production can not be less than zero]


Region x + 2y 80: line x + 2y = 80 meets axes at A(80,0), B(0,40)


respectively. Region not containing origin represents x + 2y 80 as (0,0) does


not satisfy x + 2y 80.


Region 3x + y 75: line 3x + y = 75 meets axes at C(25,0), D(0,75)


respectively. Region not containing origin represents 3x + y 75 as (0,0) does


not satisfy 3x + y 75.


Region x,y 0: it represents first quadrant.



The corner points are D(0,75), E(14,33), A(80,0).


The values at Z at these corner points are as follows:


Corner Point Z = 4x + 6y


D 450


E 254


A 320


The minimum value of Z is 254 which is attained at E(14,33).


Thus, the minimum cost is Rs 254 obtained when 14 units of compound A


and 33 units compound B are produced.



Question 18.

A company manufactures two type of novelty Souvenirs made of plywood. Souvenirs of type A require 5 minutes each for cutting and 10 minutes each for assembling. Souvenirs of type B require 8 minutes each for cutting and 8 minutes each for assembling. There are 3 hours 20 minutes available for cutting and 4 hours available for assembling. The profit is 50 paisa each for type A and 60 paisa each for type B souvenirs. How many souvenirs of each type should the company manufacture in order to maximize the profit?


Answer:

Let the company manufacture x souvenirs of Type A and y souvenirs of Type B.


Therefore, x 0, y 0


The given information can be compiled in a table as follows:



The profit on Type A souvenirs is 50 paisa and on Type B souvenirs is 60 paisa. Therefore, profit gained on x souvenirs of Type A and y


souvenirs of Type B is Rs 0.50x and Rs 0.60y respectively.


Total Profit, Z = 0.5x + 0.6y


The mathematical formulation of the given problem is,


Max Z = 0.5x + 0.6y


Subject to constraints,


5x + 8y 200


10x + 8y 240


x 0, y 0


Region 5x + 8y 200: line 5x + 8y = 200 meets axes at A(40,0), B(0,25) respectively. Region containing origin represents the solution of the inequation 5x + 8y 200 as (0,0) satisfies 5x + 8y 200.


Region 10x + 8y 240: line 10x + 8y = 240 meets axes at C(24,0), D(0,30) respectively. Region containing origin represents the solution of the inequation 10x + 8y 240 as (0,0) satisfies 10x + 8y 240.


Region x,y 0: it represents first quadrant.



The corner points of the feasible region are O(0,0), B(0,25), E(8,20), C(24,0).


The values of Z at these corner points are as follows:



The maximum value of Z is attained at E(8,20).


Thus, 8 souvenirs of Type A and 20 souvenirs of Type B should be produced each day to get the maximum profit of Rs 16.



Question 19.

A manufacturer makes two products A and B. Product A sells at 200 each and takes 1/2 hour to make. Product A sells at ₹ 300 each and takes 1 hours to make. There is a permanent order for 14 of product A and 16 of product B. A working week consists of 40 hours of production and weekly turnover must not be less than Rs 10000. If the profit on each of product A is ₹ 20 and on product B is Rs 30, then how many of each should be produced so that the profit is maximum. Also, find the maximum profit.


Answer:

Let x units of product A and y units of product B were manufactured.


Number of units cannot be negative.


Therefore, x,y 0.


According to question, the given information can be tabulated as:



Also, the availability of time is 40 hours and the revenue should be atleast Rs 10000.


Further, it is given that there is a permanent order for 14 units of Product A and 16 units of product B.


Therefore, the constraints are,


200x + 300y 10000,


0.5x + y 40


x 14


y 16.


If the profit on each of product A is Rs 20 and on product B is Rs 30. Therefore, profit gained on x units of product A and y units of product B is Rs 20x and Rs 30y respectively.


Total profit = 20x + 30y which is to be maximized.


Thus, the mathematical formulation of the given LPP is,


Max Z = 20x + 30y


Subject to constraints,


200x + 300y 10000,


0.5x + y 40


x 14


y 16


x,y 0.


Region 200x + 300y 10000: line 200x + 300y = 10000 meets the axes at A(50,0), B(0,) respectively.


Region not containing origin represents 200x + 300y 10000 as (0,0) does not satisfy 200x + 300y 10000.


Region 0.5x + y 40: line 0.5x + y = 40 meets the axes at C(80,0), D(0,40) respectively.


Region containing origin represents 0.5x + y 40 as (0,0) satisfies 0.5x + y 40.


Region represented by x 14,


x = 14 is the line passes through (14,0) and is parallel to the Y - axis. The region to the right of the line x = 14 will satisfy the inequation.


Region represented by y 16,


y = 14 is the line passes through (16,0) and is parallel to the X - axis. The region to the right of the line y = 14 will satisfy the inequation.


Region x,y 0: it represents first quadrant.



The corner points of the feasible region are E(26,16), F(48,16), G(14,33), H(14,24).


The values of Z at these corner points are as follow:



The maximum value of Z is Rs 1440 which is attained at F(48,16).


Thus, the maximum profit is Rs 1440 obtained when 48 units of product A and 16 units of product B are manufactured.



Question 20.

A manufacturer produces two type of steel trunks. He has two machines A and B. For completing, the first types of the trunk requires 3 hours on machine A and 3 hours on machine B, whereas the second type of the trunk requires 3 hours on machine A and 2 hours on machine B . Machines A and B can work at most for 18 hours and 15 hours per day respectively. He earns a profit of Rs 30 and Rs 25 per trunk of the first type and the second type respectively. How many trunks of each type must
he make each day to make maximum profit?


Answer:

Let x trunks of first type and y trunks of second type were manufactured. Number of trunks cannot be negative.


Therefore, x, y 0


According to the question, the given information can be tabulated as



Therefore, the constraints are,


3x + 3y 18


3x + 2y 15.


He earns a profit of Rs 30 and Rs 25 per trunk of the first type and the second type respectively. Therefore, profit gained by him from x trunks of first type and y trunks of second type is Rs 30x and Rs 25y respectively.


Total profit Z = 30x + 25y which is to be maximized.


Thus, the mathematical formulation of the given LPP is


Max Z = 30x + 25y


Subject to


3x + 3y 18


3x + 2y 15


x, y 0


Region 3x + 3y 18: line 3x + 3y = 18 meets axes at A(6,0), B(0,6) respectively. Region containing origin represents the solution of the inequation 3x + 3y 18 as (0,0) satisfies 3x + 3y 18.


Region 3x + 2y 15: line 3x + 2y = 15 meets axes at C(5,0), D(0,) respectively. Region containing origin represents the solution of the inequation 3x + 2y 15 as (0,0) satisfies 3x + 2y 15.


Region x,y 0: it represents first quadrant.



The corner points are O(0,0), B(0,6), E(3,3), and C(5,0).


The values of Z at these corner points are as follows:



The maximum value of Z is 165 which is attained at E(3,3).


Thus, the maximum profit is of Rs 165 obtained when 3 units of each type of trunk is manufactured.



Question 21.

A manufacturer of patent medicines is preparing a production plan on medicines, A and B. There are sufficient raw materials available to make 20000 bottles of A and 40000 bottles of B, but there are only 45000 bottles into which either of the medicines can be put. Further, it takes 3 hours to prepare enough material to fill 1000 bottles of A, it takes 1 hours to prepare enough material to fill 1000 bottles of B and there are 66 hours available for this operation. The profit is ₹ 8 per bottle for A and ₹ 7 per bottle for B. How should the manufacturer schedule his production in order to maximize his profit?


Answer:

Let production of each bottle of A and B are x and y respectively.


Since profits on each bottle of A and B are Rs 8 and Rs 7 per bottle respectively. So, profit on x bottles of A and y bottles of of B are 8x and 7y respectively. Let Z be total profit on bottles so,


Z = 8x + 7y


Since, it takes 3 hours and 1 hour to prepare enough material to fill 1000 bottles of Type A and Type B respectively, so x bottles of A and y bottles of B are preparing is hours and hours respectively, bout only 66 hours are available, so,



3x + y 66000


Since raw materials available to make 2000 bottles of A and 4000 bottles of B but there are 45000 bottles in which either of these medicines can be put so,


x 20000


y 40000


x + y 45000


x,y 0. [ Since production of bottles can not be negative]


Hence mathematical formulation of the given LPP is,


Max Z = 8x + 7y


Subject to constraints,


3x + y 66000


x 20000


y 40000


x + y 45000


x,y 0


Region 3x + y66000: line 3x + y = 66000 meets the axes at A(22000,0), B(0,66000) respectively.


Region containing origin represents 3x + y 10000 as (0,0) satisfy 3x + y 66000


Region x + y 45000: line x + y = 45000 meets the axes at C(45000,0), D(0,45000) respectively.


Region towards the origin will satisfy the inequation as (0,00 satisfies the inequation


Region represented by x 20000,


x = 20000 is the line passes through (20000, 0) and is parallel to the Y - axis. The region towards the origin will satisfy the inequation.


Region represented by y 40000,


y = 40000 is the line passes through (0,40000) and is parallel to the X - axis. The region towards the origin will satisfy the inequation.


Region x,y 0: it represents first quadrant.



The corner points are O(0,0), B(0,40000), G(10500,34500), H(20000,6000), A(20000,0).


The values of Z at these corner points are,



The maximum value of Z is 325500 which is attained at G(10500, 34500).


Thus the maximum profit is Rs 325500 obtained when 10500 bottles of A and 34500 bottles of B are manufactured.



Question 22.

An aeroplane can carry a maximum of 200 passengers. A profit of ₹ 400 is made on each first class ticket and a profit of ₹ 600 is made on each economy class ticket. The airline reserves at least 20 seats of first class. However, at least 4 times as many passengers prefer to travel by economy class to the first class. Determine how many each type of tickets must be sold in order to maximize the profit for the airline. What is the maximum profit.


Answer:

Let required number of first class and economy class tickets be x and y respectively.


Each ticket of first class and economy class make profit of Rs 400 and Rs 600 respectively.


So, x ticket of first class and y tickets of economy class make profit of Rs 400x and Rs 600y respectively.


Let total profit be Z = 400x + 600y


Given, aeroplane can carry a minimum of 200 passengers, so,


x + y 200


Given airline reserves at least 20 seats for first class, so,


x 20


Also, at least 4 times as many passengers prefer to travel by economy class to the first class, so


y 4x


Hence the mathematical formulation of the LPP is


Max Z = 400x + 600y


Subject to constraints


x + y 200


y 4x


x 20


x,y 0 [ since seats in both the classes can not be zero]


Region represented by x + y 200: the line x + y = 200 meets the axes at A(200,0), B(0,200). Region containing origin represents x + y 200 as (0,0) satisfies x + y 200.


Region represented by x 20: line x = 20 passes through (20,0) and is parallel to y axis. The region to the right of the line x = 20 will satisfy the inequation x 20


Region represented by y 4x: line y = 4x passes through (0,0). The region above the line y = 4x will satisfy the inequation y 4x


Region x,y 0: it represents the first quadrant.



The corner points are C(20,80), D(40,160), E(20,180).


The values of Z at these corner points are as follows:



The maximum value of Z is attained at E(20, 180).


Thus, the maximum profit is Rs 116000 obtained when 20 first class tickets and 180 economy class tickets are sold.



Question 23.

A gardener has a supply of fertilizer of type I which consists of 10% nitrogen and 6% phosphoric acid and type II fertilizer which consists of 5% nitrogen and 10% phosphoric acid. After testing the soil conditions, he finds that he needs at least 14 kg of nitrogen and 14 kg of phosphoric acid for his crop. If the type I fertilizer costs 60 paise per kg and type II fertilizer costs 40 paise per kg, determine how many kilograms of each fertilizer should be used, so that nutrient requirements are met at a minimum cost. What is the minimum cost?


Answer:

Let x kg of Type I fertilizer and y kg of Type II fertilizers are supplied.


The quantity of fertilizers can not be negative.


So, x,y 0


A gardener has a supply of fertilizer of type I which consists of 10% nitrogen and Type II consists of 5% nitrogen, and he needs at least 14 kg of nitrogen for his crop.


So,


(10100) + (5100) 14


Or, 10x + 5y 1400


A gardener has a supply of fertilizer of type I which consists of 6% phosphoric acid and Type II consists of 10% phosphoric acid, and he needs at least 14 kg of phosphoric acid for his crop.


So,


(6100) + (10100) 14


Or, 6x + 10y 1400


Therefore, A/Q, constraints are,


10x + 5y 1400


6x + 10y 1400


If the Type I fertilizer costs 60 paise per kg and Type II fertilizer costs 40 paise per kg. Therefore, the cost of x kg of Type I fertilizer and y kg of Type II fertilizer is Rs0.60x and Rs 0.40y respectively.


Total cost = Z(let) = 0.6x + 0.4y is to be minimized.


Thus the mathematical formulation of the given LPP is,


Min Z = 0.6x + 0.4y


Subject to the constraints,


10x + 5y 1400


6x + 10y 1400


x,y 0


The region represented by 6x + 10y 1400: line 6x + 10y = 1400 passes through A(,0) and B(0,140). The region which doesn’t contain the origin represents the solution of the inequation 6x + 10y 1400


As (0,0) doesn’t satisfy the inequation 6x + 10y 1400


Region represented by 10x + 5y 1400: line 10x + 5y = 1400 passes through C(140,0) and D(0,280). The region which doesn’t contain the origin represents the solution of the inequation 10x + 5y 1400


As (0,0) doesn’t satisfy the inequation 10x + 5y 1400


The region, x,y 0: represents the first quadrant.



The corner points are D(0,280), E(100,80), A(,0)


The values of Z at these points are as follows:



The minimum value of Z is Rs 92 which is attained at E(100,80)


Thus, the minimum cost is Rs92 obtained when 100 kg of Type I fertilizer and 80 kg of TypeII fertilizer is supplied.



Question 24.

Anil wants to invest at most ₹ 12000 in Saving Certificates and National Saving Bonds. According to rules, he has to invest at least ₹ 2000 in Saving Certificates and at least 4000 in National Saving Bonds. If the rate of interest on saving certificate is 8% per annum and the rate of interest on National Saving Bonds is 10% per annum, how much money should he invest to earn

maximum yearly income? Find also his maximum yearly income.


Answer:

Let Anil invests Rs x and Rs y in saving certificate (SC) and National saving bond (NSB) respectively.


Since, the rate of interest on SC is 8% annual and on NSB is 10% annual. So, interest on Rs x of SC is and Rs y of NSB is per annum.


Let Z be total interest earned so,


Z = +


Given he wants to invest Rs 12000 is total


x + y 12000


According to the rules he has to invest at least Rs 2000 in SC and at least Rs 4000 in NSB.


x 2000


y 4000


Hence the mathematical formulation of LPP is to find x and y which


Maximizes Z


Max Z = +


Subject to constraints


x 2000


y 4000


x + y 12000


x,y 0


The region represented by x 2000: line x = 2000 is parallel to the y - axis and passes through (2000,0).


The region not containing the origin represents x 2000


As (0,0) doesn’t satisfy the inequation x 2000


The region represented by y 4000: line y = 4000 is parallel to the x - axis and passes through (0,4000).


The region not containing the origin represents y 4000


As (0,0) doesn’t satisfy the inequation y 4000


Region represented by x + y 12000: line x + y = 12000 meets axes at A(12000,0) and B(0,12000) respectively. The region which contains the origin represents the solution set of x + y 12000


as (0,0) satisfies the inequality x + y 12000.


Region x,y 0 is represented by the first quadrant.



The corner points are E(2000,10000), C(2000,4000), D(8000,4000).


The values of Z at these corner points are as follows:



The maximum value of Z is Rs 1160 which is attained at E(2000,10000).


Thus the maximum earning is Rs1160 obtained when Rs 2000 were invested in SC and Rs 10000 in NSB.



Question 25.

A man owns a field of area 1000 sq.m. He wants to plant fruit trees in it. He has a sum of 1400 to purchase young trees. He has the choice of two type of trees. Type A requires 10 sq.m of ground per tree and costs ₹ 20 per tree and type B requires 20 sq.m of ground per tree and costs ₹ 25 per tree. When fully grown, type A produces an average of 20 kg of fruit which can be sold at a profit of ₹ 2.00 per kg and type B produces an average of 40 kg of fruit which can be sold at a profit of ₹ 1.50 per kg. How many of each type should be planted to achieve maximum profit when the trees are fully grown? What is the maximum profit?


Answer:

Let the required number of trees of Type A and B be Rs x and Rs y respectively.


Number of trees cannot be negative.


x,y 0.


To plant tree of Type A requires 10 sq. m and Type B requires 20 sq. m of ground per tree. And it is given that a man owns a field of area 1000 sq. m. Therefore,


10x + 20y 1000


x + 2y100


Type A costs Rs 20 per tree and Type B costs Rs 25 per tree. Therefore, x trees of type A and y trees of type B cost Rs 20x and Rs 25y respectively. A man has a sum of Rs 1400 to purchase young trees.


20x + 25y 1400


4x + 5y280


Thus the mathematical formulation of the given LPP is


Max Z = 40x - 20x + 60y - 25y = 20x + 35y


Subject to,


x + 2y100


4x + 5y280


x,y 0


Region 4x + 5y280: line 4x + 5y280 meets axes at (70,0), (0,56) respectively.


The region containing origin represents 4x + 5y280 as (0,0) satisfies 4x + 5y280.


Region x + 2y100: line x + 2y = 100 meets axes at (100,0), (0,50) respectively.


Region containing origin represents x + 2y100 as (0,0) satisfies x + 2y100


Region x,y 0: it represents the first quadrant.



The corner points are (70,0), P(20,40), (0,50)


The values of Z at these corner points are as follows:



The maximum value of Z is 1800 which is attained at P(20,40).


Thus the maximum profit is Rs 1800 obtained when Rs 20 were involved in Type A and Rs 40 were involved in Type II.




Exercise 30.5
Question 1.

Two godowns, A and B, have grain storage capacity of 100 quintals and 50 quintals respectively. They supply to 3 ration shops, D, E and F, whose requirements are 60, 50 and 40 quintals respectively. The cost of transportation per quintal from the godowns to the shops are given in the following table :



How should the supplies be transported in order that the transportation cost is minimum?


Answer:

Let godown A supply x and y quintals of grain to shop D and E respectively. Remaining grain storage of A is 100 – x – y. So, now it is supplied to shop F by godown A.

Now, remaining requirement of shop D is 60 – x which is supplied by godown B.


Remaining requirement of shop E is 50 – y which is supplied by godown B.


Remaining requirement of shop F is 40 – (100 – x – y)


= x + y - 60 which is supplied by godown B.


where,


x ≥ 0, y ≥ 0 and 100 – x – y ≥ 0


⇒ x ≥ 0, y ≥ 0 and x + y ≤ 100


60 – x ≥ 0, 50 – y ≥ 0 and x + y – 60 ≥ 0


⇒x ≤ 60, y ≤ 50 and x + y ≥ 60


This can be illustrated by this:




Cost = Number of quintals * Cost of transportation per quintal


Total transportation cost z is given by,


z = 6x + 3y + 2.5(100 – x – y) + 4(60 – x) + 2(50 – y) + 3(x + y – 60)


⇒ z = 6x + 3y + 250 – 2.5x – 2.5y + 240 – 4x + 100 – 2y + 3x + 3y – 180


⇒ z = 2.5x + 1.5y + 410


We need to minimize the cost


Hence, mathematical formulation of LPP is


Minimize z = 2.5x + 1.5y + 410


subject to the constraints,


x + y ≥ 60


y ≤ 50


x ≤ 60


x + y ≤ 100


x, y≥0


The feasible region determined by the system of constraints is as follows:



The corner points of enclosed region are A(60, 0) , B(60, 40), C(50, 50) and D(10,50)


The value of z at these corners points is as follows:


Case 1: A(60, 0)


z = 2.5x + 1.5y + 410


⇒ z = 2.5(60) + 1.5(0) + 410


⇒ z = 150 + 0 + 410


⇒ z = 560


Case 2: B(60, 40)


z = 2.5x + 1.5y + 410


⇒ z = 2.5(60) + 1.5(40) + 410


⇒ z = 150 + 60 + 410


⇒ z = 620


Case 3: C(50, 50)


z = 2.5x + 1.5y + 410


⇒ z = 2.5(50) + 1.5(50) + 410


⇒ z = 125 + 75 + 410


⇒ z = 610


Case 4: D(10, 50)


z = 2.5x + 1.5y + 410


⇒ z = 2.5(10) + 1.5(50) + 410


⇒ z = 25 + 75 + 410


⇒ z = 510


The value of z is minimum in fourth case at point D(10, 50)


As, x = 10, y = 50


Godown A supplies to:


Shop D = x = 10 quintals


Shop E = y = 50 quintals


Shop F = 100 – x – y = 100 – 10 – 50 = 40 quintals


Godown B supplies to:


Shop D = 60 – x = 60 – 10 = 50 quintals


Shop E = 50 – y = 50 – 50 = 0 quintals


Shop F = x + y – 60 = 10 + 50 – 60 = 0 quintals


Minimum cost for transportation of these quintals to their respective shops = Rs. 510




Question 2.

A medical company has factories at two places, A and B. From these places, supply is made to each of its three agencies situated at P, Q and R. The monthly requirements of the agencies are respectively, 40, 40 and 50 packets of the medicines, while the production capacity of the factories, A and B are 60 and 70 packet respectively. The transportation cost per packet from the factories to the agencies are given below :



How many packets from each factory be transported to each agency so that the cost of transportation is minimum? Also find the minimum cost?


Answer:

Let factory A supply x and y packets of medicines to agency P and Q respectively. Remaining packets of factory A are 60 – x – y. So, now they will be supplied to agency R by factory A.

Now, remaining packets requirement of agency P is 40 – x which will be supplied by factory B.


Remaining packets requirement of agency Q is 40 – y which will be supplied by factory B.


Remaining packets requirement of agency R is 50 – (60 – x – y)


= x + y – 10 which will be supplied by factory B.


where,


x ≥ 0, y ≥ 0 and 60 – x – y ≥ 0


⇒ x ≥ 0, y ≥ 0 and x + y ≤ 60


40 – x ≥ 0, 40 – y ≥ 0 and x + y – 10 ≥ 0


⇒x ≤ 40, y ≤ 40 and x + y ≥ 10


This can be illustrated by this:




Cost = Number of packets * Cost of transportation per packet


Total transportation cost z is given by,


z = 5x + 4y + 3(60 – x – y) + 4(40 – x) + 2(40 – y) + 5(x + y – 10)


⇒ z = 5x + 4y + 180 – 3x – 3y + 160 – 4x + 80 – 2y + 5x + 5y – 50


⇒ z = 3x + 4y + 370


We need to minimize the cost


Hence, mathematical formulation of LPP is


Minimize z = 3x + 4y + 370


subject to the constraints,


x+y≥10


y≤40


x≤40


x+y≤60


x,y≥0


The feasible region determined by the system of constraints is as follows:



The corner points of the enclosed region are A(10, 0) , B(40, 0), C(40, 20), D(20,40), E(0, 40) and F(0, 10)


The value of z at these corners points is as follows:


Case 1: A(10, 0)


z = 3x + 4y + 370


⇒ z = 3(10) + 4(0) + 370


⇒ z = 30 + 0 + 370


⇒ z = 400


Case 2: B(40, 0)


z = 3x + 4y + 370


⇒ z = 3(40) + 4(0) + 370


⇒ z = 120 + 0 + 370


⇒ z = 490


Case 3: C(40, 20)


z = 3x + 4y + 370


⇒ z = 3(40) + 4(20) + 370


⇒ z = 120 + 80 + 370


⇒ z = 570


Case 4: D(20, 40)


z = 3x + 4y + 370


⇒ z = 3(20) + 4(40) + 370


⇒ z = 60 + 160 + 370


⇒ z = 590


Case 5: E(0, 40)


z = 3x + 4y + 370


⇒ z = 3(0) + 4(40) + 370


⇒ z = 0 + 160 + 370


⇒ z = 530


Case 4: F(0, 10)


z = 3x + 4y + 370


⇒ z = 3(0) + 4(10) + 370


⇒ z = 0 + 40 + 370


⇒ z = 410


The value of z is minimum in first case at point A(10, 0)


As, x = 10, y = 0


Factory A supplies to:


Agency P = x = 10 packets


Agency Q = y = 0 packets


Agency R = 60 – x – y = 60 – 10 – 0 = 50 packets


Godown B supplies to:


Agency P = 40 – x = 40 – 10 = 30 packets


Agency Q = 40 – y = 40 – 0 = 40 packets


Agency R = x + y – 10 = 10 + 0 – 10 = 0 packets


Minimum cost for transportation of these packets to their respective agencies = Rs. 400





Mcq
Question 1.

The solution set of the inequation 2x + y > 5 is
A. half plane that contains the origin

B. open half plane not containing the origin

C. whole xy-plane not containing the origin

D. none of these


Answer:

Given inequation is 2x + y > 5.


Now, we convert the inequation into an intercept line equation form, we can clearly see the intercepts of the inequation on x-axis and y-axis.


2x + y > 5


[dividing the whole inequation by 5]





Therefore, from the above inequation, we can say that, 2.5 and 5 are the intercepts of the x-axis and y-axis respectively.


Now by plotting these on the graph, we can clearly see the graph of the inequation.



From the graph, it is clear that, the solution set of the inequation


2x + y > 5 is the open half plane not containing origin i.e. option B.


Question 2.

Objective function of a LPP is
A. a constraint

B. a function to be optimized

C. a relation between the variables

D. none of these


Answer:

Given,


To define the objective of a Linear programming Problem.


As per the definition of the Linear Programming Problem,


A Linear programming problem is a linear function (also known an objective function) subjected to certain constraints for which we need to find an optimal solution ( i.e. either a maximum/minimum value) depending on the requirement of the problem.


From the above definition, we can clearly say that, Linear programming problem’s objective is to either maximize/ minimize a given objective function, which means to optimize a function to get an optimum solution.


Hence the answer is option B.


Question 3.

Which of the following sets are convex?
A. {(x, y) : x2 + y2 ≥ 1}

B. {(x, y) :y2 ≥ x}

C. {(x, y) : 3x2 + 4y2 ≥ 5}

D. {(x, y) : y ≥ 2, y ≤ 4}


Answer:

Given sets are


• {(x,y) : x2 + y2≥ 1}


• {(x, y) :y2 ≥ x}


• {(x, y) : 3x2 + 4y2 ≥ 5}


• {(x, y) : y ≥ 2, y ≤ 4}


A convex set, is nothing but whose solution set is in the shape of a convex polygon.


If we map these functions on a graph, we can clearly find the set with a convex solution set.


• f : {(x,y) : x2 + y2≥ 1}



From the graph, it is evident that the solution set which is the shaded region is not convex.


• g:{(x, y) :y2 ≥ x}



From the graph, it is evident that the solution set which is the blue shaded region is not convex.


• h:{(x, y) : 3x2 + 4y2 ≥ 5}



From the graph, it is evident that the solution set which is the grey shaded region is not convex.


• p:{(x, y) : y ≥ 2, y ≤ 4}



From the graph, the dark blue shaded region between the two bright lines is a convex set.


Hence, the solution is option D.


Question 4.

Let X1 and X2 are optimal solutions of a LPP, then
A. X = λ X1 + (1 – λ)X2, λ ϵ R is also an optimal solution

B. X = λ X1 + (1 – λ)X2, 0 ≤ λ ≤ 1 gives an optimal solution

C. X = λ X1 + (1 – λ)X2, , 0 ≤ λ ≤ 1 give an optimal solution

D. X = λ X1 + (1 +λ)X2, λϵ R gives an optimal solution


Answer:

Given, X1 and X2 are optimal solutions of a Linear programming problem(LPP).


This means that, {X1 ,X2} C (a convex Set) as the optimal solution of a LPP is convex.


Now by using the definition of a Convex set,


A set of points C is called convex if, for all λ in the interval 0 ≤ λ ≤ 1, λy + (1 − λ)z is contained in C whenever y and z are contained in C.


By using this property of Convex set,


If {X1 ,X2} C (a convex set of optimal solutions), then


X = λX1 + (1 − λ) X2 where 0 ≤ λ ≤ 1, is also contained in C (the optimal solution set).


This proves that, also X C.


Hence the answer is option B.


Question 5.

The maximum value of Z = 4x + 2y subjected to the constraints 2x + 3y ≤ 18, x + y ≥ 10; x, y ≥ 0 is
A. 36

B. 40

C. 20

D. none of these


Answer:

Given,


Z = 4x + 2y


Subjected to constraints,


2x + 3y ≤ 18


x + y ≥ 10


x ≥ 0


y ≥ 0


Consider, the inequalities as equalities for some time,


2x + 3y = 18 and x + y = 10,


If we convert these into intercept line format equations, we get,


[Dividing the whole equation with the right hand side number of the equation]


and


and


From this form of line, we can say that the line 2x + 3y = 18 meets the x-axis at (9,0) and y-axis at (0,6).


This shows the inequality 2x + 3y ≤ 18 holds good in the below blue colored region.



Similarly from the intercept line format, we can say that the line x + y = 10 meets the x-axis at (10,0) and y-axis at (0,10).


This shows the inequality x + y ≥ 10 holds above in the green colored region.



Now considering the inequalities, x ≥ 0 and y ≥ 0, this clearly shows the region where both x and y are positive. This represents the 1st quadrant of the graph.


So, the solutions of the LPP are in the first quadrant where the inequalities meet.


Now by plotting both graphs 2x + 3y ≤ 18 and x + y ≥ 10 we get the below graph.



We can clearly see that, there is no area in the 1st quadrant where the two inequalities met.


This clearly says that there is no solution for the LPP with the given constraints.


Hence the option D, is the solution to the problem.


Question 6.

The optimal value of the objective function is attained at the points
A. given by intersection of inequations with the axes only

B. given by intersection of inequations with x-axis only

C. given by corner points of the feasible region

D. none of these


Answer:

Given that,


• There is an objective function


• There are optimal values


From the definition of optimal value of a Linear Programming Problem(LPP):


An optimal/ feasible solution is any point in the feasible region that gives a maximum or minimum value if substituted in the objective function.


Here feasible region of an LPP is defined as:


A feasible region is that common region determined by all the constraints including the non-negative constraints of the LPP.


So the Feasible region of a LPP is a convex polygon where, its vertices (or corner points) determine the optimal values (either maximum/minimum) of the objective function.


For Example,


5x + y ≤ 100 ; x + y ≤ 60 ; x ≥ 0 ; y ≥ 0


The feasible solution of the LPP is given by the convex polygon OADC.



Here, points O, A ,D and C will be optimal solutions of the taken LPP


Hence the answer is option C.


Question 7.

The maximum value of Z = 4x + 3y subjected to the constraints 3x + 2y ≥ 160, 5x + 2y ≥ 200, x + 2y ≥ 80; x, y ≥ 0 is
A. 320

B. 300

C. 230

D. none of these


Answer:

Given object function is


Z = 4x+3y


Constraints are


3x + 2y ≥ 160


5x + 2y ≥ 200


x + 2y ≥ 80


x ≥ 0


y≥ 0.


Consider, the inequalities as equalities for some time,


3x + 2y = 160 ; 5x + 2y = 200 and x + 2y = 80


If we convert these into intercept line format equations, we get,


[Dividing the whole equation with the right hand side number of the equation]


; and


; and


From this form of line, we can say that the line 3x + 2y = 160 meets the x-axis at (,0) and y-axis at (0,80).


This shows the inequality 3x + 2y ≥ 160 holds good in the below blue colored region.



Similarly, from the intercept line format, we can say that the line 5x + 2y = 200 meets the x-axis at (40,0) and y-axis at (0,100).


This shows the inequality 5x + 2y ≥ 200 holds above in the pink colored region.



Similarly from the intercept line format, we can say that the line x + 2y = 80 meets the x-axis at (80,0) and y-axis at (0,40).


This shows the inequality x + 2y ≥ 80 holds above in the green colored region.



Now considering the inequalities, x ≥ 0 and y ≥ 0, this clearly shows the region where both x and y are positive. This represents the 1st quadrant of the graph.


So, the solutions of the LPP are in the first quadrant where the inequalities meet.


Now by plotting all the graphs 3x + 2y ≥ 160 , 5x + 2y ≥ 200 and x + 2y ≥ 80 we get the below graph.



We can clearly see that, there is no area in the 1st quadrant where all the three inequalities met.


This clearly says that there is no solution for the LPP with the given constraints.


Hence the option D, is the solution to the problem.


Question 8.

Consider a LPP given by

Minimum Z = 6x + 10y

Subjected to x ≥ 6; y ≥ 2; 2x + y ≥ 10; x, y ≥ 0

Redundant constraints in this LPP are

A. x ≥ 0, y ≥ 0

B. x ≥ 6, 2x + y ≥ 10

C. 2x + y ≥ 10

D. none of these


Answer:

Given


Objective Function is Z = 6x + 10y


Constraints are:


x ≥ 6


y ≥ 2


2x + y ≥ 10


x, y ≥ 0


A redundant constraint is that, which doesn’t intersect with the feasible region of the out non-redundant constraints.


Here, the problem is a minimization problem and as per the constraints x≥ 0 and y ≥ 0 the feasible solution is located in the 1st quadrant.


Now, if we map all the three inequalities in a graph, we have



From the graph, it is very clear that, the graph of the inequality 2x + y ≥ 10 is not intersecting the feasible region formed by the constraints x ≥ 6; y ≥ 2; x≥ 0 and y ≥ 0.


Hence the inequality 2x + y ≥ 10 is not really making any difference to the feasible region from by x ≥ 6; y ≥ 2; x≥ 0 and y ≥ 0.


Therefore inequality 2x + y ≥ 10 remains redundant.


The answer of the question is option C.


Question 9.

The objective function Z = 4x + 3y can be maximized subjected to the constraints 3x + 4y ≤ 24, 8x + 6y ≤ 48, x ≤ 5, y ≤ 6; x, y ≥ 0
A. at only one point

B. at two points only

C. at an infinite number of points

D. none of these


Answer:

Given the objective function is Z = 4x + 3y


Constraints are:


3x + 4y ≤ 24


8x + 6y ≤ 48


x ≤ 5


y ≤ 6


x ≥ 0


y≥ 0


If we consider these inequalities as equalities for some time,


We will have


3x + 4y = 24


8x + 6y = 48


x = 5


y = 6


x = 0


y= 0


If we plot all these lines on a graph we will have optimal area formed by the vertices, OABCD.



Now, to find where the function Z has maximized, let us substitute all these points in the objective function Z.



Here, we can clearly see that, the function Z is maximized at two points B & C giving the value 24.


There will be infinite/multiple optimal solutions for a LPP if it has more than one set of optimal solutions that can maximize/ minimize a problem.


This will clear the fact that, the function Z will maximize at infinite number of points.


Hence the answer is option C.


Question 10.

If the constraints in a linear programming problem are changed
A. the problem is to be re-evaluated

B. solution is not defined

C. the objective function has to be modified

D. the change in constraints is ignored


Answer:

Given,


The constraints of a linear programming problem are changed.


Now, as per the definition of the Linear Programming Problem,


A Linear programming problem is a linear function (also known an objective function) subjected to certain constraints for which we need to find an optimal solution ( i.e. either a maximum/minimum value) depending on the requirement of the problem.


Here, the LPP is solved using the constraints, so, if the constraints are changed, the problem is to has to be re-calculated with the new constraints provided.


Hence the answer is option A.


Question 11.

Which of the following statements is correct?
A. Every LPP admits an optimal solution

B. A LPP admits unique optimal solution

C. If a LPP admits two optimal solutions it has an infinite number of optimal solutions

D. The set of all feasible solutions of a LPP is not a converse set convex set


Answer:

Given,


The statements:


• Every LPP admits an optimal solution


This need not be true as all LPPs need not have optimal solutions and such LPPs are called unbound.


• A LPP admits unique optimal solution


Every LLP need not have unique optimal solutions as if there are two optimal solutions to an LLP there will be infinite number or optimal solutions to the LLP problem.


• If a LPP admits two optimal solutions it has an infinite number of optimal solutions


As mentioned in the above point, if there are two optimal solutions to an LLP there will be infinite number or optimal solutions to the LLP problem.


• The set of all feasible solutions of a LPP is not a convex set.


As per a theorem of Convex Sets,


If {X1 ,X2} C (a convex set of optimal solutions), then


X = λX1 + (1 − λ) X2 where 0 ≤ λ ≤ 1, is also contained in C (the optimal solution set). This makes all the feasible solutions of a LPP also a convex set.


Hence, from the explanations, the answer is Option C.


Question 12.

Which of the following is not a convex set?
A. {(x, y) : 2x + 5y < 7}

B. {(x, y) : x2 + y2 ≤ 4}

C. {x : |x| = 5}

D. {(x, y) : 3x2 + 2y2 ≤ 6}


Answer:

Given 4 sets,


i. {(x, y) : 2x + 5y < 7}


ii. {(x, y) : x2 + y2 ≤ 4}


iii. {x : |x| = 5}


iv. {(x, y) : 3x2 + 2y2 ≤ 6}


By graphing them, we can clearly figure out the convex set.


A convex set, is nothing but whose solution set is in the shape of a convex polygon.


i. {(x, y) : 2x + 5y < 7}


This inequation can be converted into an equation and by applying the intercept line format, we get,



[dividing the whole by 7]


We get,



So the graph is



ii. {(x, y) : x2 + y2 ≤ 4}


So the graph for this inequality is given by



iii. {x : |x| = 5}


The graph for the set is as below:



iv. {(x, y) : 3x2 + 2y2 ≤ 6}


The graph for the inequality is given by



From t he above graphs, we can clearly say that, only the 3rd graph is a convex set.


Hence option C is the answer.


Question 13.

By graphical method, the solution of linear programming problem

Maximize Z = 3x1 + 5x2

Subject to 3x1 + 2x2 ≤ 18

x1 ≤ 4

x2 ≤ 6

x1 ≥ 0, x2 ≥ 0, is

A. x1 = 2, x2 = 0, Z = 6

B. x1 = 2, x2 = 6, Z = 36

C. x1 = 4, x2 = 3, Z = 27

D. x1 = 4, x2 = 6, Z = 12


Answer:

Given objective function is Z = 3x1 + 5x2


Subject to constraints:


3x1 + 2x2 ≤ 18


x1 ≤ 4


x2 ≤ 6


x1 ≥ 0, x2 ≥ 0


To solve this,


Consider the constraints as equations for a while, then we will have


3x1 + 2x2 = 18


x1 = 4


x2 = 6


x1 = 0, x2 = 0


Consider the 3x1 + 2x2 = 18, for graphing this, we can find the intercepts of this equation and then plot the line.


By dividing the whole equation with 18 we get,




From this equation we know that, x1 will intercept the line x1 = 0 at (6,0) and x2 will intercept the line x2 = 0 at (0,9)


Using these points to plot 3x1 + 2x2 = 18 in the graph, continue plotting the other equations, x1 = 4 , x2 = 6 , x1 = 0, x2 = 0 on he graph.



From the graph we can clearly see that the area under the polygon, OABCD is the feasible solution.


For determining the maximum optimum value from the polygon, the points at the vertices of OABCD are substituted in the Z function and see which point will maximize the value of Z = 3x1 + 5x2.



Among the above points, the Z value maximized at point B where x1 = 2 and x2 = 6.


Therefore the solution is, option B.


Question 14.

The region represented by the inequation system x, y ≥ 0, y ≤ 6, x + y ≤ 3 is
A. unbounded in first quadrant

B. unbounded in first and second quadrants

C. bounded in first quadrant

D. none of these


Answer:

Given inequations are


x, y ≥ 0, y ≤ 6, x + y ≤ 3


If we graph the inequalities y ≤ 6 and x + y ≤ 3 on the graph, we get the graph,



Here we can clearly see that, the region for the inequality is above the points A, B, C and D. But considering the constraints x , y ≥ 0, this clearly shows that the region can only be in the 1st quadrant.


So from the green color highlighted area, we can say that, the region of the inequalities is unbound in the first quadrant.


Hence, the answer is option A.


Question 15.

The point at which the maximum value of x + y, subject to the constraints x + 2y ≤ 70, 2x + y ≤ 95, x, y ≥ 0 is obtained, is
A. (30, 25)

B. (20, 35)

C. (35, 20)

D. (40, 15)


Answer:

Given objective function is Z = x + y


Constraints are:


x + 2y ≤ 70


2x + y ≤ 95


x, y ≥ 0


Let us consider these constraints as equations for a while, then we will have,


x + 2y = 70 ----- (1)


2x + y = 95 ----- (2)


Now, graph the equations, by transforming the equations to intercept form of line.


Equation (1) dividing throughout by 70




The line x + 2y = 70 can be plot in the graph as a line passing through the points, (70,0) and (0,35) as 70 and 35 are the intercepts of the line on the x-axis and y-axis respectively.


Similarly equation (2) can be divided 95 to get




The line 2x + y = 95 can be plot in the graph as a line passing through the points, (,0) and (0,95) as and 95 are the intercepts of the line on the x-axis and y-axis respectively.


By considering the constraints x , y ≥ 0, this clearly shows that the region can only be in the 1st quadrant.


The graph of the inequations will look like,



The points OABC is the feasible region of the LPP.


Now from the points O, A ,B and C the vertices of the polygon formed by the constraints, one of the points will provide the maximum solution to the function Z = x + y


Now checking the points, O,A,B and C by substituting in Z = x + y.



From the above values, it is clear that Z maximized at point B( 40,15).


Hence the answer is option D.


Question 16.

The value of objective function is maximum under linear constraints
A. at the centre of feasible region

B. at (0, 0)

C. at any vertex of feasible region

D. the vertex which is maximum distance from (0, 0)


Answer:

Given that,


The objective function will be maximum at ______


As per the steps while solving a Linear Programming Problem, we first determine the feasible region using the constraints and then, consider the vertices of the obtained convex polygon in the objective function to see the vertex at which the objective function obtains a maximum value. Then that vertex is determines the maximum value of the objective function.


Hence, the maximum value of an objective function under linear constraints is at any vertex of the feasible region.


Therefore the answer is C.


Question 17.

The corner points of the feasible region determined by the following system of linear inequalities:

2x + y ≤ 10, x + 3y ≤ 15, x, y ≥ 0 are (0, 0), (5, 0), (3, 4) and (0, 5). Let Z = px + qy, where p, q > 0. Condition on p and q so that the maximum of Z occurs at both (3, 4) and (0, 5) is

A. p = q B. p = 2q

C. p = 3q D. q = 3p


Answer:

Given the vertices of the feasible region are:


O (0,0)


A (5,0)


B (3,4)


C (0,5)


Also given the objective function is Z = px+qy


Now substituting O,A,B and C in Z



As per the condition on p and q so that the maximum of Z occurs at both (3, 4) and (0, 5)


Then we can equate Z values at B and C, this gives


3p + 4q = 5q


3p = 5q – 4q


3p = q


Therefore the answer is option D i.e. q = 3p.